Você está na página 1de 512
COMPLETE ADVANCED LEVEL MATHEMATICS for <1 SS, 6 Andy Martin * Kevin Brown © Paul Rigby « Simon Riley STANLEY THORNES © Andy Martin, Kevin Brown, Paul Rigby, Simon Riley 2000 Ilustrations © Stanley Thornes (Publishers) Ltd 2000, ‘The right of Andy Martin, Kevin Brown, Paul Righy and Simon Riley to be identified as authors of this work has been asserted by them in accordance with the Copyright, Designs and Patents Act 1988, All rights reserved. No part of this publication may be reproduced or transmitted in any form or by any means, electronic or mechanical, including photocopy, recording, or any information storage and retrieval system, wilhout permission in writing from the publisher or under licence from the Copyright Licensing Agency Limited. Further dotails of such licencos (for reprographic reproduction) may be obtained from the Copyright Licensing Agency Limited of 90 Tottenham Court Road, London W1P OLP. First published in 2000 by ‘Stanley Thornes (Publishers) Lid Ellanborough House Wellington Street (CHELTENHAM GLs01YW A calalogue record for this baok is available fram the British Library, ISBN 0 7487 3558.5 Book plus Advanced Level Mathematics Tutorials Pure Mathematics CD-ROM sample {Tracle Edition) ISBN 0 7487 4523 8 Advanced Level Mathematies Tutorials Pure Mathematics CD-ROM Multi-user version ISBN 0 7487 39149 Single-user version ISU 07487 3918 1 oo 0102.03 04/10987654321 Cover photograph, STP Archive Photograph, p106, STP Archive Evory effort has been mado to contact copyright holders. The authors and publishers apologise to anyone whose rights have been ‘overlooked and will be happy to rectify any errors ar omissions at the earliest opportunity. ‘Typosot by Tho Alden Group, Oxford Printed and bound in Italy by STIGE, Turin Contents Examination Papers 4 Algebra What you need to know Review 1.1 The Number System and Surds igindices 1.3 Polynomials 1.4 Factorisation 1.5 Solving Quadratic Equations 1.6 Simultaneous Equations Consolidation B Applications and Activities Summary 2 Coordinate Geometry: What you need to know: Review 2.1 Coordinate Geome! 2.2 The Equation of a Straight Line 2.3 More on the Straight Line 2.4 Inequalities 5 The Equation of a Circle Consolidation A Consolidation B Applications and Activities Summary 3 Trigonometry 1 hat you nes fo kuow 3.1 Trigonomotric Functions 3.2 Equations and Identities a Compound and Doublo: ompound and Double Angle Formulas 3.4 The Gosine Rule and the Sine Rule ‘The Area of a Triangle Consolidation A Consolidation B Applications and Activities Summary 4 Functions What you need to know: Review 4.1 Mappings and Functions 42 Inverse Function: 4.3 Composite Functions 4aTranslormations of Graphs and Functions 4.5 Even, Odd and Periodic Functions Consolidation A Consolidation B Applications and Activities ‘Summary & 1 1 1 3 a u 20 26 39 a4 a5, Contents 5 Differentiation 1 What you need to knaw Review 1 Finding the Grac Stationary Points: 5.3 Further Ap lications of Differentiation 4 The Chait Rule and Related Rates of Change 5 Differentiation of Trigonometric Functions lidation A. Consolidation B Applications and Activities Summary tof a Curve 6 Algebra I What you need to know Review, 6.1 Polynomial Division 2 Algebraic Fractions 3 Partial Fractions 6.4 Curve Sketching Consolidation A 190 190 190 191 208 27 226 236 249 251 253 254 255 255 255 256 265 270 281 289 Consolidation BO Applications and Activities 292 Summary 7 Exponentials and Logarithms What you need to know Roview 7.1 Tho Exponential Function 7.2 Logarithms 293 294 204 204 295 300 7.3 Laws of Logarithms 304 7.4 Solving a* = b 309 nsolidation A ‘316 Consolidation Be Applications and Activitios 319 Summary 320 B Sequences and Series What you need to know 8.1 Sequences and Series 8.2 Arithmetic Progression 8.3 Geometric Progression 8.4 Convorgenco, Divergence and Oscillation 8.5 The Binomial Theorem and Power Series Consolidation A Consolidation B Applications and Act Summary 9 Integration I What you need to know Review 9.1 Indefinite Integration The Area Under a Graph The Arca Botween 9.5 Volumes of Revolution 9.6 Integration of Sine and Gosine Consolidation A Consolidation B Applications and Activities Summary 321 321 B21 322 828 335 340 346 354 355 357 358 359 359 359 360 367 377 384 390 397 399 401 402 Contents 10 Trigonometry 1 403 What you need io know 403, Review 403 10.1 'The Factor Formulas 404 10.2 Functions of the Form f(x) = asinx + bcosx. 411 10.3 General Solutions of Trigonometric Equations er 427 Applications and Activities 420 ‘Summary 430 ifforontiati What you need to know 431 11.1 Differentiating Products and Quotients 433 11.2 Differentiating Exponentials and Logarithms 438 11.3 Further Triganomettic Differentiation 448 11/4 Using the Second Derivative 455 11.5 Implicit Differentiation 465 11.6 Parametric Differentiation. 472 11.7 Maclaurin Series 481 lation A 488 490 Applications and Activities 491 Summary 491 12 Integration I What you need fo know Review 12.1 Standard Integrals 12.2 Rational Functions 12.3 Integration by Parts 12.4 Integration by Substitution 12.5 The Area Under a Paramotrically Defined Curve 12.6 Differential Equations Consolidation A Consolidation B Applications and Act Summary vities 13 Numerical Methods What you need to know Review 13.1 Approximate Solutions of Equations 13.2 Numerical Integration Applications and Activities 587 ‘Summary 588 14 Vectors 590 What you need to know 590 Review 14.1 Vector Geomet 591 T4.2 Vectors in Two and ThFse Dimensions soo 44.3 The Sealar Product 614 14-4 The Vector Equation of a Straight Line 626 14.5 The Vector Equation of a Plane 643 Consolidation A 661 Consolidation B 663 Applications and Activ 665, Summary 665 Contents vi 15 Proof and Mathematical Argument What you need to know matical Argument 15.2 Proof by Exhaustion and Disproof by Counter Example 15.3 Proof by Contradiction 15.4 Proof by Induction Consolidation A Applications and Activities Summary Answers a7 Formulas 724 Index 727 Examination Papers We are grateful to the following Awarding Bodies for permission to reproduce questions from their past examination papers @ Assessment and Qualifications Alliance (AQA), including Northern Examinations and Assessment Board (NEAB), Associated Examining Board (AEB), Secondary Mathematics Project (SMP) and Joint Matriculation Board (JMB). @ The Edexcel Foundation, including University of London Examinations and Assessment Council (ULEAC), @ Oxford, Cambridge and RSA (OCR), including University of Cambridge Local Examinations Syndicate (UCLES), Oxford & Cambridge Schools Examination Board (OCSEB) and Mathematics in Education and Industry (MED), @ Northern Ireland Council for the Curriculum, Examin and Assessment (NICCEA). @ =Welsh Joint Education Committee (WJEC), All answers and worked solutions provided for examination questions are the responsibility of the authors. Graphical calculator support An icon ( gi) is shown in the margin feature of this book to denote areas where the work can be further developed or demonstrated with a graphical calculator. A Graphical Calculator Support Pack is: also available to use alongside this book (ISBN: 07487 4524 6), containing k ulator settings for Texas, Sharp and Casio macl nal questions that build on the activities are also provided. Where there is a link on the page of this book to the pack this is indicated next to the icon in the margin. Advanced Level Mathematics Tutorials Pure Mathematics Advanced Level Mathematics Tutorials Pure Mathematics is a new CD-ROM resource. It is designed to provide further and valuable extra practice and additional support to your Pure Mathematics work, There are two versions of this programme, each providing Live Authored™ worked solutions to exercises from this book. ‘The Multi-user version provides worked solutions to 120 exercises; the Single-user version provides worked salutions to 75 exercises. Each worked solution is supported by an additional two new exercises to try. nin book to denote where a worked solution to an exercise on that page is provided on the CD-ROM. The relevant exercise number(s) is shawn with the icon, shown in the margin feature of the pages in this For further details of this new resource, contact Stanley Thornes (Publishers) Ltd, Ellenborough House, Wellington Street, Cheltenham, Glos GL50 1YW. Tel: 01242 267272. Fax: 01242 253695. e-mail: cservice@thornes.co.uk Multi-user version: A sot of three identical disks containing 120 Live Authored™ worked solutions. Each disk is licensed to three users. ISBN 0 7467 3914 9 Single-user version: A single disk containing 75 Live Authored™ oxamples. ‘The disk is licensed to one user. ISBN 0 7487 39181 About the Authors @ Andy Martin is currently Head of Mathematics in a Doncaster school. He has also been a Pure Mathematics Examiner and Coursework Moderator since 1994 as well as a member of the Examination Board's subject committee. Andy is also the National Project Officer for a National Gurriculum Development project. @ Kevin Brown is currently Head of Mathematics at a school in St Helens. Kevin has many years’ teaching experience at a comprehensive school in Liverpool. He has also previously written material fora Mathematics multimedia package. @ Paul yy is currently Second in the Mathematics department of a high school in Bolton. Paul has worked previously at a comprehensive school in Oldham, @ Simon H Riley { is currently Head of Mathematics at a school in ant Examiner, vil Introduction Complete Advanced Level Mathematics is an exciting new series of mathematics books, Teacher Resource Files and other support materials (see page viii) fom Stanley Thornes for those studying at Advanced Level. It has been developed following an extensive period of research and consultation with a wide number of teachers, students and others. All the authors are experienced and practising teachers and, in some cases, Advanced Level Mathematics Examiners. Chapters have been trialed in schools and colleges. All the requirements for complete success in Advanced Level Mathematics are provided by this seri This book covers all the requirements for Pure Mathematics from all the latest Advanced Level specifications and course requirements for AS and A Level mathematics. It will provide you with: @ Material that builds on work done at GGSE level, where appropriate. Comprehensive coverage and clear explanations of all Pure Mathematics topics and skills. Numerous exercises and worked examples with questions and clear diagrams, Precise and comprehensive teaching text with clear progression. Margin notes that provide supporting commentary on key topics, formulas and other aspects of the work. In text highlighted ‘hints’ to assist with important areas, such as specific calculations in worked examples and key formulas. Margin icons for topics requiring the use of a graphical calculator or computer. Topics that can be developed with IT. are included throughout. © A comprehensive list of formulas that students need to know, with chapter references, and a full index. Chapters in this book contain a number of key features: @ What you need to know sections covering prerequisite knowledge for a chapter. © Review sections with practice questions on what you need to know. Worked Examples and supporting commentary. Technique and Contextual Exercises to give thorough practice in all Pure Mathematics concept areas and skills. © Consolidation A and B Exercises, which include actual examination questions, to build on the work in a chapter and provide practice in a variety of question types. Applications and Activities as a support to coursework. A Summary of all the key concepts covered. Companion volumes for Mechanics (0 7487 3559 3) and Statistics (0 7487 3560 7) and a Pure Mathematics Teacher Resource File (0 7487 3561 5) are also available in this series. vill COMPLETE ADVANCED LEVEL MATHEMATICS for <1 SS, 6 Andy Martin * Kevin Brown © Paul Rigby « Simon Riley STANLEY THORNES COMPLETE ADVANCED LEVEL MATHEMATICS Pure Mathematics Andy Martin * Kevin Brown © Paul Rigby * Simon Riley STANLEY THORNES Thais one } (N00 AN 2969-1XC-BEBR 1 Algebra | What you need to know How to use index form, How to evaluate Va x Va. How to collect like terms, How to expand brackets. How to solve linear equations. How to factorise algebraic expressions. How to use a division algorithm (method) without a calculator. Review Write the following in index form: a xxx do axaxbxbxb bo 7x7x7 @ xox © XXxXKX f{ Sax 2ax2a Find the exact value of the following: a V36 x V36 d vi6 x vi6 b Vix ve e vaxva © Vax Va Po Vex VE By collecting like terms simplify the following expressions: a 2v43x do ayy 43x Bb ox? + ax? - 2x? e 2% —avd4 © 6y—3y4+4e f 249% —ax ‘Multiply out, or expand, the following terms in brackets, simplifying the answer where possible: a ax +2) B (x+1)(x+2) b 2(x-3) ho (x—2)( +3) © ~2(x42) i (x+#4)(x=1) d -3(x-a) j (e483) +5) © —5(x+2y—3) k (x—2)(x-3) f 6(2x—2y — 12) 1 (x-4)? Urheberrechtlich geschiitztes Mat Complete Advanced Level Mathematics: Pure Mathematics Irrationals and surds ‘There are some numbers that cannot be written in this form. These numbers have a decimal expansion that doesn't terminate, but goes on forever without repeating. These numbers are known as irrational numbers, Some examples of irrational numbers are x, 2, v5 and V7. ‘They cannot be evaluated exactly; calculators simply give an approximation to 8, 10 or 12 decimal places. The ancient Grecks called numbers like /2, /5 and V7 incommensurables. Now they are often referred to as surds. Not all irrationals are surds; x is not a surd. Surds are used to give exact answers instead of an approximation to many decimal places (in much the same way as fractions are proferred to decimals). Surds can also be manipulated using the following properties. WBERYS Property 1 Learn these properties. i BREE 5 pens BEES pes Example 3 ‘Simplify: a vas b va c Ov7F+2v7 dd 5V9- VF Solution a V5 = VOx5 =Vax V5 4 Using property 1. =3v5 bo VH=Vaxe =Vax V6 <4 Using property 1. =2Vv6 © 6Y742V¥7=(642)V7 4 Using property 3. =8v7 d 5V3~- V27 =5v8-voxd Notice how 27 can be =5V8—(Jix V3) << Using property 1. written as a product involving a square =5V5—3V3=2V3 4 Using property 4. Humber! errechtlich geschitztes Material 1 Algebra | Example 4 Expand and simplify (v8 - ¥3)(V8 + V3). Solution (va — V8)(V8 + V3) = VB Va + v3) — V3(V8 + V5) = (V8)? + Vav3 - VaV8 - (v3)? = (vi)? - (v3)? =8-3=5 Example 4 demonstrates the algebraic result for difference of two squares: BEBE al=Ya This means ‘power }' can be thought of as a cube root. Investigate powers 1, },and so on in the same way. Notice how the fraction is related to the root, TEBE —Propesty6 © 4 Learn this property. Complete Advanced Level Mathematics: Pure Mathematics Property 6 can also be used to establish a meaning for fractional powers where the numerator isn't 1. For example, a? can be written in other forms, ali = (a)! = (aly? <4 Use property 2, ‘The first version, a} = (a°)}, allows the following interpretation. ai = (a?! = Va? Notice how the fractional power creates a root of a power. BBY etp Property 7 4 Learn this property. ‘These seven properties provide useful techniques for simplifying algebraic expressions. Example 1 Simplify: res axa i 1 cGy 4 Use property 1 on the numerator. <4 Use property 3. py texty’ _ oxty® arty ty =6x?-y8-)) 4 Use property 3 omx and y separately. = oxy" oy" =F << Use property 5. ce (8x8)F = 3? x (x8? = ox? <4 Use property 2. 10 Urheberrechtlich geschiitztes Material +1 Algebra Example 2 Write the following expressions in index form: aa bs © (t)! a bo Gackt 4 Use property 5. Notice in part b how the variable (x) is put into index form, but the fraction (}) is left as a multiplier. ce (tied 4 Use property 2. =% a -vae = Var =927 «Ye 4. Use surd property 1. =3x Example 3 Without using a calculator, evaluate: a ot b ayy <4 Use property 5. 4 Use property 2 Remove the mixed number. “a 4. Use property 5. Remember that division 4 Use property 6. bya a invert the fraction (and multiply). Urheberrechtlich geschitztes Merial Complete Advanced Level Mathematics: Pure Mathematics ‘The properties, or rules, of indices can also be used in equation solving. Example 4 Find the value of x when: a 2125 b a4 Solution a 5% 5125 ‘This type of problem can sometimes be solved using a trial and improvement approach, as follows, although this may be time consuming for very large numbers, and for non-integer solutions. sims Seon 25 S)=5x5x5=125 Sox=3 bo oxtsa <4 Use property 7. Ve aa ae x= 64 x= tVveE x=48 Urheberrechtlich geschdtztes Material 1 Algebra! 1.2 Indices Exercise Technique 1 | simplity the following expressions: a d b € c f 2.| Simplify the following expressions: a yt dai’)? box)? 2 xt} ce 2(x*)? f (3x7)8 3) Write the following expressions in index form: 3 SB x5 aa a 4 qaxéy* bo ° cary? aa . oy a oye [4] weite the folowing expressions in their simplest form: wt a tt at # ae S i ' 5 | Evaluate the following expressions without using a calculator: a 16 e (yt b oat r go e (-27yi 2 (3p a (-27)) ho (2h G | Using trial and improvement, or otherwise, solve (find the value of x for) these equations: a b a=9 c Fak do 12821 e £ x%=p g xtel ho e=343 Complete Advanced Level Mathematics: Pure Mathematics 4 1.3 Polynomials ‘An expression is a combination of numbers, variables (usually represented by the letters x and y) and mathematical operations (+, -, x, +). Some examples of expressions are 3x +2, 7x? + 3y, and x* — 5x +6. When two expressions are linked by the symbol of equality an equation is formed. Robert Recorde (1510-1558) For example, the following are all equations: Ox+255 Tx? + 3y =x+2 x -5x+6=0 ‘The general form of a simple equation can be written in various ways. yoax¢d, y=mete and y=a,x+ay are all forms of linear equations. Why are these equations linear? ‘Try drawing their graphs for some values ofa and b, mand ¢, or a, and ay. Notice how the graph is always a straight line. For example, y = 2x + 1 is one version ofy =ax +b, witha =2and b= 1. It is illustrated here. ‘An equation of the form y = x", where 1 is a positive integer, is known as a polynomial. The general form of this type is written y=a,x" +a, 4x" 4... +a, +0,x +a, where a, £0. This is called 4 polynomial equation of degree n. yox'43x-1 is a polynomial of degree 2, because its highest power is 2. x’+/e-Jisnota polynomial. This is yo2t 470 -Ox44 betaine fz =a and) isn't a positive intege is a polynomial of degroe 5, because its highest power is 5. What about the numbers in the polynomials? Notice how each term of the equation is distinct, separated by the operators + and ~. The number in front of each variable (letter) is called a coefficient. 1 Algebra! Consider the polynomial y = 2x° + 7x" — Gx +4. This is a quintic equation (order 5, because the highest power of x is 5). The numbers 2,7 and —8 are coefficients: 2 is the coefficient of the x° term; 7 is the coefficient of the x* term; and ~8 is the coefficient of the x term. The final term, +4, is called a constant because il remains the same {its value is constant), whatever the value of x. Addition and subtraction In order to simplify polynomials, like terms are collected together. Like terms can then be added or subtracted algebraically. Notice the distinction between like and unlike terms. @ Examples of like terms x*, 3x7, and 2x" aro like terms, bocause thoy all have the samo variable, x7 xvand 3yxcare like terms, because they both have the same variable, xy © Examples of unlike terms x and xy are unlike terms, because they contain different variables x“y and y’x are unlike terms, because they contain different variables Example 1 Simplify: a (3x? + 8x 2) + (Sx? + 3x +8) Bb (6x? + 8x ~ 2) — (5x? + 2x7) Solution a (3x7 + Bx ~ 2) + (5x7 + 3x + 8) = Bx" + BK— 24 x7 43x48 = 3x? +5x7 4 Ox+3N-248 = 8x 4 11N4+ 6 Bb (Gx? + ax — 2) — (5x7 + 2x—7) = Ox? + 8x —2- 5x? -2x+7 = GN" — Sx” + 8x — 2-247 =x 46x45 term inside the bracket, and collect together like Multiplication terms, Multiplication of polynomials is achieved by applying the distributive Jaw. In algebra this law can be stated, B= Verily this law for and yourself using numbers. Complete Advanced Level Mathematics: Pure Mathematics For example, 23 « 17 can be written (20+ 3) x (10 +7) = 20(10 +7) +3(10 +7) = 200 + 140+ 30 +21 =391 ‘What is the effect of this law? Notice that every term in the second bracket has been multiplied by each of the terms in the first bracket. This is the principle used in polynomial multiplication, Example 2 Expand and simplify: a (2x-+3)(4x" +3x? = 2x +5) Bb (3x” x7 + 7)(2x" + 2x? — ax +2) ce (x42) Solution a Recall the structure of the distributive law, Each term in the second bracket must be multiplied by each term in the first bracket. (2x + 3)(4x7 + 3x? — 2x +5) = 2x(ax" + 3x? — 2v +5) + 3(4x? + 3x? — 2x +5) = Bx" 4 6x" — ax? + 10x + 12x" + Ox? — Bx +15 = Ox 4 6x? + 120? — 4x? 4 ON? + LON — G4 15 = BN 4 18x? 45x? 4 an 4 15, b Recall the structure of the distributive law. (ax? — x7 + 7)(2x" + 2x? — 3x +2) = 3x¥(ax" + 2x? — 3x + 2) — x7 (2x? + 2x7 — 3x +2) + 7(2x3 + 2x7 - 3x +2) = 6x" + 6x9 — 9x! + 6x? — 2x9 — 2x4 + 3x? — 2n* + 14x" 4 14x? —21N4 14 = Gx" 4 Gx? = 2x* — ox* = 2x + Gx? + 3x" + 14x" = 2x7 + 14x? ~2ix+14 = 6x" + ax? — dix! + 23x" 4 12x" — 24x + 14 © (+2) = (x +2)(a+2)(6 +2) 4 Using the distributive law (x + 2)In(x +2)42(x42)) onthe second pair of ickets. = (x +2)" + 2x + 2N4- 4] = (x + 2)(x7 + ax #4) = x(x? + dx $4) + 20x? + ax +4) Notice how the faint crossing out can help collect and combine Ii terms quickly. =x) + 4x7 + 4y + dx? + Ox +8 =x) 6x7 +4 12x +8 Irheberrechtlich ges 41 Algebra Division Before attempting division of polynomials think back to division with integers (whole numbers). Numerical division can be represented by fractions. Fractions in the form 3, 44, 3. $are known as proper fractions. Hera the numerator (Lop number) is less than the denominator (bottom number). How are improper fractions written? Here the reverse is true. The numerator is greater than the denominator. Some examples are 3, and #2. How else could these be written? Improper fractions can also be written as mixed numbers, that is a mixture of an integer (whole number) and a propor fraction. f=13, a1, Hath: Fa3} ond os Similarly algebraic fractions (fractions involving polynomials) can be both proper and improper. 3 2 x44 _ et n 5 SET SOE PH Meal! proper algebraic fractions X42 x8413 x*45N+6 x45) N-4) PP 7KF12 are all impropor algebraic fractions. What's the difference between proper and improper algebraic fractions? Look at the degree, or order, of the polynomials in the numerator and denominator. For an algebraic fraction to be proper the order of the numerator must be less than the order of the denominator. If not, the fraction is improper. Example 3 Make the expression + 3 a proper algebraic fraction, Solution Notice that “+3 is improper (the numerator and denominator are both Here the x's cannot be order't,orlinen].‘Therw are two techaiqués hat could be used tomicke © (egameneeeestn! this a proper algebraic fraction, ste nek ta rty SE aot numerator and denominator. Method 1: The division algorithm Write ay asa division problem, and try a division by x. theberrechilich geschiitztesq gpuerial ‘Complete Advanced Level Mathematics: Pure Mathematics a X45)x$3 1 x+5)XE3 xt =2 So ‘Notice the positions of the remainder and the divisor in the result, Method 2: Algebraic manipulation In this technique the numerator is written as a multiple of the denominator and a remainder. x+3_ (x+5)-2 X45 X45 Then the numerator is split into two distinct parts. E+3_x4+5 2 2 x+8 x+8 +5) x48 Notice how both techniques give the same result, The fraction in the answer is proper. Example 4 ‘Write as proper fractions: 3x+6 1-x x+3 Solution a Using the division algorithm, 3x46 _ 3(x+3)=3 x+3 xX+3 = Met) 3 x+3 xX+3 3 “8-33 18 Because x in (x +5) v divide into the x in {x +3) once, Subtract 1 x (x++ 5) from (x +3). The remainder is 3-5=-2, eS Ps Finally, 0 ~(—4) =4. ‘The coefficient of x in the numerator is 3, so (x+ 3) is multiplied | 3. Multiplying out the bracket, the constant term in the numerator 9, We require it to be +6, s0 we subtract 3, 1 Algebra! 1.3 Polynomials Exercise Technique For each of the following polynomials write down (i) the order (degree) and (ii) the coefficient of x*: a ax? 42x? -2 do axt-7 b sxt-ax? +x e 3x -6x4+9 c 7x8 =3x'- 7x +9 £ ax -2x43 [2] ada: 3x74 7x = Sand Sx? = 2x48 6x? 4 Gx 4 9 and Gx? + 9x — 2 4x34 ax? 434 Gand 3x? + 7x —3 1x8 + 7x7 — ax + 9 and —an¥ + 2x8 + 7x7 + 7x [3] rinay, —y, when: a oy; =5x* +12x4 Sandy, = 3x7 +7x=4 boy, = 7x? + 12K — Zand y, = 2x* — 5x47 cy; =5xt 43x" — 2x7 + Ox and yy = 7x4 — 2x) — 2x? 42x —5, doy) =x" — 2x7 + 7x + 13 and y = Sx? — an? + Gx +12 ao oe [4] Muttipty and simplify the following: a (2x+1) and (3x+2) d= (3x -1) and (x? - 2x +1) b (2x43) and (2x7 +1) e@ (3 —x)and (x? + 3x —2) «= © (2x—1) and (x* + 3x +4) £ (4—x)and (2x* 5x +7) Expand and simplify the following: (2x + 8)(3x? — 2x + 8) + (x + 1)(x? + Bx 42) (5x — 1)(2x? + 3x + 2) + (x + 3)(2a7 + 4x — 3) (ax — 2)(3 + 2x —x*) + (x — 5)(x - 2x + 1) (4x = 3)(2x +7 — 2x?) + (x = 1)(a? + Sx = 1) (Sx + 1)(x? + 2x + 2) = (x + 1)(x? + 3x 41) (3x — 2)(2x* + 7x — 5) — (x — 1)(x" - 3x42) mene ee | Expand and simplify the following: a (2x+3)(2x+3) bb (2x+3)(2x4+3)? © (ax+3)* d= (1+2x)* e (x+1)' f (x-1)* [7] ‘Write the following as proper fractions: a #42 bt? eto x45 x+5 x1 6x47 ae 2 2x43 4 yea ° xre 0 et heberre es INPorial Complete Advanced Level Mathematics: Pure Mathematics 20 1.4 Factorisation ‘The distributive law a(b + c) =ab-+ ac has been used to demonstrate multiplication of polynomials, The same law can be used in reverse, soa sum of terms can be written as a product. Doing this often introduces brackets into the algebra. The process is called factorisation, Factorisation can be shown with natural numbers. The integers that divide exactly into @ are 1, 2, 4 and 6. These are called the factors of 6. Notice that 6 can be written as a product of some of these factors. =1xB B=2K4 Polynomials too can sometimes be factorised. Consider first polynomials of degree 2. These are more commonly known as quadratics. One such quadratic expression is x? + 3x — 18, It can be factorised as follows. x" 4 3x — 18 = (x +6)(x — 3) Chock this by multiplying out the brackets. We call (x + 6) and (x — 3) the factors of the quadratic x? + 3x — 18. Quadratics can be factorised using one of three basic techniques: @ extracting a common factor @ trial and improvement @ standard results ~ difference of two squares. Extracting a common factor Example 1 Factorise 4x"y* = 8x7y", Solution Here it is possible to extract common factors, 4-and 8 have the common factor 4 (4 is the largest factor of both numbers). x and x? have the common factor x? (x* is the largest factor of both terms). y? and y* have the common factor y* (v* is the largest factor of both terms). So 4x*y* = ax?y* = 4x¢y* x (some other term). Notice how the common factors are extracted from both terms in the expression and appear outside the bracket, These form one part of the product. The bracket must contain the terms necessary to combine with the common factor to create the original expression. axly? — arty? = axty* (x — ay) Check that 4x@y* = x = 4x9y? and that 4x*y? x (27) = —Bx¢y". 1 Algebra! In order to factorise quadratics, first check the coefficients in the expression. Consider again the quadratic expression, x* + 3x — 18. The coefficient of x* is 1, the coefficient of x is 3 and the constant term is ~18, So the three distinct numbers in this expression are 1, 3 and —18, Now look at the factors. of that expression, (x + 6) and (x ~ 3). These contain the two distinct numbers 6 and —3. How are these two sets of numbers connected? Notice that adding the numbers in the factors and multiplying the numbers in the factors creates the two larger numbers given by the coefficients. 6 + (~3) =3,which is the coefficient of x 6 x (—3) = -18, which is the constant term. ‘The general rule, for a quadratic expression where the coefficient of x? is 4, is that the expression can be factorised if two numbers can be found that add to give the coefficient of x and multiply to give the constant term. Trial and improvement Example 2 Factorise: a 4 7e +12 bo Bx +15 Solution a The coefficient of x* is 1, so we know that we need to find two numbers that add to make 7 and multiply to make 12. Notice that 4 and 3 work. xP 47x 412 = (x +4)(x +3) b The coefficient of x" is 1, so we need to find two numbers that add to —8 and multiply to 15. We find that —5 and —3 work -5+(-3)=-8 (5) x (-3) = 15 Sox? = Bx +15 = (x 5)(x-3). By finding the factors of the constant term first, much of the trial and improvement in these examples can be dane quickly. What happens when the coefficient of x* is greater than 1? This trial and improvement technique can then be modified, The method, or proces: sometimes known as 'PAFF’, the letters P, A, F and F representing the four stages of the process: Product, Addition, Factors and Factorise. The idea is to change the algebra into smaller numerical problems leading to soma less complicated factorisation. Follow each stage of the technique in Example 3, 221 Complete Advanced Level Mathematics: Pure Mathematics Example 3 Factorise 12x* 4 17x = 14, Solution Notice that the coefficient of x* is 12 so the technique used in Example 2 won't work. Try PAFF, the stages of which are as follows. 1, P— Product Multiply the coefficient of x* by the constant term. Here P= 12 x (-14) P=-168 2, A- Addition ‘This is the coefficient of x, something that factors need to add to. Here, A =17 3. -F- Factors Using the same technique as boforo, but this timo find two numbers that multiply to give P and add to give A. In this example ~7 and 24 work. (-7) x 24= -168 —7424=17 4, F—Factorise Use the factors identified in Step 3 to help factorise the expression. ‘These allow the coefficient of x to be split. 12x? + 17x — 14 = 12x? — 7x4 24 14 ‘This new expression can now be factorised by extracting common factors. Imagine factorising the first pair of terms and the second pair of terms separately. 12x? — 7x + 24x — 14 = x(12x — 7) + 2(12x —7) Notice that (12x — 7) is a new common factor, 12x? — 7x4 24x = 14 = (12x —7)(x + 2) Check by multiplying So 12x? +. 17K — 14 = (12x —7)(x +2) out the factors. This process looks complicated and time consuming, but with practice can be a very effective algorithm for factorising quadratics where the coofficient of x* is groater than 1. 22 1 Algebra Example 4 Factorise 4x? = 2x ~ 30. Analternative method here would be to extract Solution adi the common factor first ‘The coefficient o! is 4, so use PAFF, and we PARFona P: 4x (-30) =-120 simpler equation, At the coefficient of x is —2 F: the factors need to multiply to 120, and add to ~2. Check that —12 and 10 work. Fi 4x* — 2x — 30 = 4x? — 12x + 10x — 30 = 4x(x— 3) + 10(x-9) = (x —8)(4x +10) So 4x* — 2x — 30 = (x — 3)(4x + 10) What do you notice about the second factor? The numbers 4 and 10 have 2 asa common factor, so this bracket can be factorised further. Ax +10 = 2(x+5) So the original quadratic expression has three distinat factors. ax? — 2x — 30 = 2(x + 5)(x — 3) Difference of two squares Sometimes the coefficient of x can be zero, In this case the quadratic will contain an x* term and a constant term only. Factorisation of these expressions can often be achieved by extracting a common factor or using another standard result: the difference of two squares. Example 5 Factorise x’ — 49. Solution Notice that 49 is a square number; that is, 49 = 7*. The expression can therofore be rewritten, xt 49-57 7? ‘The right-hand side is now the difference of two squares. This factorises ina particular way, 7 -49=x-7? = (x+7)(x-7) Urheberrechtlich geschotztesggnierial Complete Advanced Level Mathematies: Pure Mathematics The result demonstrated in Example 5 can be generalised as HEHE h(0-» This result can be used as an aid to computation, Some “difficult” problems can be done quickly without using a calculator. Example 6 a Factorise 12x*— 3, b Factorise 5 tan’ @— 5. ¢ Evaluate 1017 — 100°. Sofution a 12x7-3=3(4x? -1) = a(x = 14} = 3(2x + 1)(2x — 1) b Stan?0~5 = 5(tan?@—1) = 5(tan?@— 17) = 5(tan 0 + 1)(tan 0 — 1) c 1017 = 100" = (101 + 100)(101 — 100) =201«1 =201 24 Urheberrechtlich 1 Algebra | 1.4 Factorisation Technique Factorise the following quadratic expressions: a x4ox42 e xt4ax—18 bo oxt+7x+10 £ x 4+x-12 c -x-20 g x +6x-16 dx 7x18 ho x +x-6 Factorise the following quadratic expressions using the difference of two squares: a x -16 e@ cost —1 b y-a9 fo sin?O-1 © ont 1 g 4x? —25y7 d 167-1 ho Bix? = 36) Factorise the following expressions completely: a 27-32 d 50y* — 200 bo ay*7—27 e 2 — 4501 © 20x75 £ 2e08*0-2 [4] Factorise the following expressions: a axt4x-2 e 10x" ~4Ix—45 bo oat-5x+3 £ 8x’ -21x-9 © 7x? 422x4+3 g 8x -17N49 do 4x? 12x +5 ho 6x? 7x-3 [5 | Factorise the following expressions: a 4x? 10x 46 e 36x? 33x46 box? + 2xy -@y* £ 16x? — 100x + 150 c x7 + 5xy —36)7 g axty— 7M +3 dd 10x7 — 18x —4 bh G0x*y ~ S5xy — 25y Utheberrechilich geschtitztes Mal Complete Advanced Level Mathematics: Pure Mathematics 1.5 Solving Quadratic : Equations ‘Gitniton algebraic ‘As in a quadratic expression, a quadratic equation in one unknown has ete the variable occurring at least once raised to the second power. The ‘ole al quate variable doesn't occur to any higher powers, Some examples of quadratic “equations that had re equations arex* — 4 = 0," + 4a— 5 = 0, 27 — 16+ 36=0,p? +p=2. frumber solutions. See Book X of the ‘Element Quadratic equations often occur in the solution of real-life problems, such as in echo sounding, calculating depths of wells and hardness testing, Can all quadratic equations be solved? The main techniques used to solve quadratic equations are: @ [actorising @ completing the square @ using the quadratic formula @ graphical methods. Factorising quadratic equations Example 1 Solve (x ~5)(x +2) = 0. Solution Notice that the left-hand side of this equation is a product of two factors, The result of multiplying these factors is zero. If two quantities multiply to zero then one of them rust be zero. Since (x — 5)(x +2) = 0, then (x —5) = 0 or (x +2) =0. These linear equations can now be solved. x-5=0 > x=5 andx+2=0 + x=-2 Check that these are solutions by substituting them back into the original equation. Notice that both of these values of x are solutions; there are two. solutions. Example 2 Solve: a x -av-5=0 b 2’ -32=0 e x -av=0 1 Algebra Solution ‘The coefficient of x’ is 1. a First factorise the quadratic expression. (+1) +(-5)=—4, (41) (-5)=-5 a ax=5=0 (x+1)@=5) =0 Sa (x+1)=0ar(x—5)=0 x=-lorx=5 b 2x? 3250 2x? — 32 = 2(x7 — 16) x? — 4?) = 2(x + 4)(x —4) (x + 4)(x 4) = 0 So 2x” — 32 = Now there aro three factors multiplying to give a zero result, Since 2 4 0,(x+4)=0 or (x—4)=0 Sox=-4orx=4 © x?—3x=x(x-3)=0 Sox =0 or (x—3)=0 x=Oorx=3 Example 3 Solve the quadratic equation 12x* + 17x — 14 =0. Solution Iax? + 17K-14=0 ‘This particular quadratic expression was factorised in Example 3 of Section 1.4. Tax? + 17K — 14 = (12x —7)(x +2) =0 So (12x —7)=Oor(x+2)=0 x=Bore=-2 Completing the square representation is Sometimes the quadratic expression cannot be factorised easily. The recurring. Check this equation may then be solved using a technique called completing the using a calculator. square, ‘To complete the square we need to write the quadratic in the form (x +a)? = b, where a and 6 are real numbers. Then the value of x can be found by taking the square root of both sides of the equation. Writing the quadratic in this form requires some skill in algebraic manipulation. e PF aierial Urheberrechttich geschitt Complete Advanced Level Mathematics: Pure Mathematics Example 4 Solve the equation x* — 6x —5 = 0. Solution ‘Try factorisation, What happens? Notice that no pair of integers add to We need the LHS to b give —6 and multiply to give —5. Don't despair: complete the square. First rewrite the equation, separating the variables from the constant term, x —6x-5=0 So. 3 -6x=5 x» -6x+9=5+9=14 So (x-3)(x-3)=14 Thatis, (x3)? =24 x-3=4V4 The solutions are therefore x = 3 4 V/14. Notice that the steps in the process of completing the square are: Stop @ Separate the constant term from the variable terms, Stop @ Adda value to cach side of the equation to force one side to be a perfect square. How did you know what value to add? There is a simple rule. Provided the coefficient of x* is 1, simply halve the coefficient of x and then square this value, Example 5 Solve x? —3x-—5 = 0. Solution x? —3x—5=0 + x°-3x=5 4 @ Separate the terms. ‘The coefficient of x is ~3. Half this is —3. Squaring that, we have §. So add ¥ to both sides of the equation. x -3x+]=54] 2 Force one side to be a perfect square, ‘The left-hand side is now a perfeet square, so we ean factorise it, oe a+y=9 ‘Take the square root of both sides. Urhaberrechilich geschtitztes Material 1 Algebra! Recall the properties of surds. Notice that written in this form (a surd), we have exact solutions and not docimal approximations. The solutions have also been written concisoly with a common denominator. Remember that this technique only works when the coefficient of x” is 1. ‘When it isn't, divide each term in the equation by the coefficient of x7. ‘This often creates equations with fractions as coefficients. Example 6 Solve 3x? + 4x —5 =0. Solution ‘The coefficient of x? is 3, so divide each term by 3. ax? +4x-5=0 > x +4x-$=0 Now the algorithm can be used as before. xt44x=8 4 @ Separate the terms. Add 4 to both sides of the equation (recall that this number is reached by halving the coefficient of x, and then squaring the result). 4 Force one side to be a perfect square. Somotimes the quadratic equation doesn’t have to be solved. It may be sufficient to write it in the form of a perfect square. Suppose that y= ax? +5x-+ccan be written in the form y = a(x + p)* +g, where a, b, ,p and qare all real numbers, ‘Then ax + bx +c =a(x +p) +q = a(x" + 2px +p") +4 ax* + bx +6 = ax" + 2apx + ap?+q Complete Advanced Level Mathematics: Pure Mathematics: By comparing coefficients you should see a relationship between a, b, ¢ and p,q. Compare the coefficients of x: that is, see how many ‘x’s there are on each side af the equation. b= 2ap Comparing the constant terms in the same way, cap +q Since a, b and care already known (directly from the quadratic) these two results can sometimes be used to establish p and g quickly. Example 7 Express the following in the form a(x + p)* + q: a x -ax-5 bo -Sx7- 2x49 Solution a Notice that a= 1,b =—3 and c= -5, We require x” — 3x = 5 = a(x +p)? +q. ‘Since a = 1, this can be simplified to xt -an-5 = (x+p)'+q = (x74 2pxtp)+q =ext+aptpitg Now comparing coefficients of.x and constant terms, -3=2p and -—5=pt+q ‘Then p=~3 and q=-5—p So x?-ae—5=(e-pt-2 has a least value of —: b Let —5x*—2x4+3=a(x +p +q =a(x* + 2px +p*)+q Sax" +2apx+ap*+q Comparing coefficients of x*, x and the constant terms, .-2=20p and 3=ap'+q 30 theberrechtlich geschiitztes Material 1 Algebral Since a = 5, -2 = 2ap becomes ~2 = 2 x (~5) x p= —10p Sop= bx? 2x43 = -5(e+ fh +q Now 3 = ap’ +q So3=-5(2) +q q=3+g=ati=# So —5x* ~2n+3 5 —5(n +H) +8 ‘The quadratic formula By completing the square on the general quadratic expression ax’ +bx +c, we can create a formula that can be used to solve quadratic equations simply by substituting values for a, b and c. Suppose ax” + by +6 = 0 for some real values of a,b and ¢. thens? + 48-0, asa Now complete the square on this expression in the same way as before. c 4.0 Separate the terms, wy a ‘The value to add to both sides of the equation is found, as before, by halving the coefficient of the x term, and then squaring the result. 2, bx (2) -(#)-2 4 @ Force one side to be a perfect 2a/ a square. ety a (2 2a 2a bY b-4ac ( +z) Gat b ‘b? — 4a Mg ta 4ac 4 Learn this result. Onee a, b and ¢ have been identified, this formula can be used to solve quadratic equations. Urheberrechtich geschotztes Material Complete Advanced Level Mathematics: Pure Mathematics 32 Example 8 Solvex-7=4, x Solution This may not at first appear to be a quadratic equation, but multiplying both sides by x gives x(x — 7) = 4. Notice how the denominator has multiplied the whole of the left-hand side of the equation, Now multiply x(x = 7) out to give x” — 7x = 4, Moving the constant term, the original ‘equation has been rearranged to the form for which the formula works. 7x40 Now, a=1,b=—7ande ~b+ VbF = 4ac 2a a9 axIx 2x1 _7 + ya9— (716) _ 7+ vos, — ee 4) So the equation x ~ 7 = $ has two distinct solutions, PAVE gay fa, 2 2 Evaluate these results using a calculator. What do you notice? Both answers are irrational so the calculator screen should give decimal expansions that do not recur. The numerical values correct to two decimal places are 7,53 and -0.53, Example 9 It is proposed that a new tunnel be built under the English Channel. This tunnel will be for cars to drive through. The road will be built ona ad concrete base inside the circular tunnel. concrete base If the radius of the tunnel is 5.2m and the width of the road surface is to be 9.2m, what dopth of concrete should be used? oO = (32-¢) A 46 TD: Lot d metres be the depth of concrete. From the diagram identify the right-angled triangle with the road surface as base. Using Pythagoras’ theorem, OA? = AD* + OD* 5.2? = 4.67 + (5.2 —d)* 5.2% = 4.67 + (5.2* — 10.4d 4 d*) So 0 = 4.6? — 10.4d +d* ‘That is, d? — 10.4d + 21.16 =0, ‘This isa quadratic equation in d where a = 1, b = ~10.4 and ¢ = 21.16. Now use the formula, b+ VBP aac 2a 0.4)°—4x 1x 21.16 =5.242.425 (3 dp) Notice that there are two solutions, The first, 2.78m, has the road in the Jower half of the tunnel. The second, 7.63 m, has the road in the upper half of the tunnel. one use of the quadratic formula in a problem solving context. Urheberrechttich ‘Complete Advanced Level Mathematics: Pure Mathematics Remember that although in many applications both solutions can be interpreted in the context of the problem, one solution will usually be preferable, Notice also that this quadratic equation had decimals as coefficients. The quadratic formula has given solutions that have been rounded, to give answers correct to three significant figures. Graphical methods A graphical calculator can be used to solve quadratic equations. This is also a good method to use if you simply want to check solutions from factorisation, completing the square or the quadratic formula, Example 10 Using a graphical calculator, or graph plotting software on a computer, draw the graphs of the fotlowing. a y=x?-4x-5 b y=x?-3x G y=xt-6x—5 do y=3x? +4x-5 Using the trace facility, find the coordinates of the points of intersection with the x axis. Now compare these results with the results from Examples 2a, 2e, 4 and 6. What do you notice? You should find that the calculator gives either an exact answer or a decimal approximation to the solutions calculated by other methods. Remember that the accuracy of these solutions will depend upon your calculator. Many will not state irrational results exactly; instead a decimal is given to 8, 10 or 12 places. Notice that the graphs of these quadratic equations all have the same basic shape, This curve is known as a parabola, but can be transformed by changing the values of the coefficients a, b and cin the expression ax’ +bx+e. Notice also that all the graphs are symmetrical. Is this line of symmetry ‘The equation related to the coefficients a, b and c? Think back to the quadratic formula. y= ax* + bx + ccross ‘This gives the solutions to the equation ax” + bx +¢ = 0 ina form that the x-axis at yy = 0. helps answer this question. ‘The points of intersection with the x-axis are written as (2+ a square-root term, 0). This suggests that the line of symmetry for the quadratic is x = —£. What about the square-root term? What does Vb? = 4ac represent? The expression b* — 4ac is known as the discriminant. It can be used to give an indication of how many times the graph will cross the x-axis, as follows. @ If b" —4ac > 0, it has two real square roots and ax” + bx +c =0 has two distinct solutions: the graph will cross the x-axis twice ~ at CS 10). @ = If b? — 4ac = 0, ax* + bx += 0 has one (repeated) solution: the graph will touch the x-axis at (— 44,0). @ fb? — 4ac < 0, ax® + bx +¢ =O has no (real) solutions: the graph will not cross the x-axis, Example 11 ‘Write down the equation of the line of symmetry of the graphs of the following quadratics, and predict the number of times the graph will cross the x-axis. a yax*4xt3 bo ysx?+5x+6 GC pox*+2xd1 Solution a yar 4xd3 When y = 0 (on the x-axis), x* +x +3=0,anda=1,b=1ande=3 in the quadratic formula. ‘The line of symmetry,x =~ isx=—}. To check to see if (and how many times) the graph crosses the x-axis, check the discriminant, BY — 4ac = 17 — (4x 1x 3) = 1-12 = -11 < 0, so the graph doesn't cross the x-axis. Urhe paptora ‘Complete Advanced Level Mathematics: Pure Mathematics b y=x45r+6 ‘When y =0,a=1,b =5 and c= 6 in the quadratic formula. ‘The line of symmetry, x = — &, is x = — 3. The discriminant b* — 4ac = 5? — (4 x 1 x 6) = 25—24=1>0,s0 the graph will cross the x-axis at two places. yas or+6 © yoxt+axrt1 ‘Here a = 1, b= Zand c= 1. The line of symmetry, x = — 4. is x= —}.Thatis,x=—1. ‘The discriminant b* — 4ac = 2* — (4 x 1 x 1) =4—4=0, so the graph will touch the x-axis at one point. Check that x? + 2x +1=0 wh 1 by factorisation (so the graph touches at (—1,0)). Urheberrechilich 1.5 Solving Quadratic Equations Exercise Technique [1] sotve the fotlowing: (x= 3)(x+2)=0 (x-3)(x-4)=0 (x-1)(x+3) =0 a b © ds (x+5)(x+1)=0 ao x45e44=0 bo oxt+2x-4i © -Gx+5=0 a 6x16 = bo xt+2x-8=0 © x -2x-3=0 a ax? 2x-8=0 b o3x*+10x-8=0 © wt+x-4=0 quadratic and the x-axis: a ax? 24x +6=0 b ax(y—a)+5 © 3{x? — 2) = 2(ax - 2) a Y-2x43 bo ox 44axdl © -x8 4 2e42 e © 8 h d e r d e iE d e f d e if d e £ (2x +5)(2x+5)=0 x(x+2)=0 12(x +3)(2x+1) =0 3(x —7)(3x +4) =0 2_| Solve, by factorising, the following equations: x-x-6=0 ax? — 198 +12 =0 9x’ + 24N+16=0 3) solve, by completing the square, the following equations: xt 6x +4 ax’ —2x-1=0 —ax*+8x+7=0 4 | Solve, using the quadratic formula, the following equations: xt-12x-5=0 2x + 15N+6=0 3x* — 18 + 10= 0 5 | Solve the following equations. In each case check the solutions by using a graphical calculator to find the points of intersection between the x e6xt4=0 ax? +6x+2=0 x43 wt 6 | Write the following expressions in the form a(x + p)' + q: =x" + Gx -19 =2x" + 5x -3 ax —3x-2 1 Algebra | Urheberrechtlich gesch 37, Complete Advanced Level Mathematies: Pure Mathematics Contextual [11] othe tormuta h = ut ~ 4gr gives the height h a body will reach after tin hon itis thrown vertically upwards with velocity u, where g is a constant, Calculate t when g = 9.8, u = 16 and h = 6. Why are thore two answers? [2] What is the shaded area ofthe washer || 42em illustrated here, where the diameter of — ' 1 ' the washer is 4.2 cm and the diameter ' —- of the hole is 1.8cm? i u [3B] rhe sum of the first n natural numbers (1-+2.43-+...-+) is given by the formula S = 4n(n-+ 1). If the sum of the numbers is 78, how many numbers have been added? [4] tthe formuta }n(n~ 3) defines the umber of diagonals in a polygon where n is the number of sides. A chef cuts a cake along its 65 diagonals. ‘How many sides does the cake have? [5] by completing the square, find the minimum value of 3a? — 12x +13. [6] How deop isthe water in this oil drum, given that the radius is 30cm and ACis 20cm? X B Urheberrechilich geschiitztes > 1 Algebra 1.6 Simultaneous Equations Polynomial equations can be graphed. If two or more polynomials are graphed on the same axes then the graphs may crass. The coordinates of the point (or points) where the graphs cross satisfy both polynomial equations at the same time; that is, simultaneously. Simultaneous linear equations: Linear equations may be put in the form y = a,x +09 where a, and ay are real numbers, The highest power of the variable x is 1, so they are of degree 1. When graphed, these equations produce straight lines, which is why they are called linear. A system of two linear equations can sometimes be solved simultaneously, using: @ substitution @ climination @ graphical methods. Example 1 Solve the equations x + 2y = 7 and 2x + 3y = 10 simullaneausly, Solution 1. Using the substitution technique gives a solution as follows. xtays7 call this equation [1] 2x +3y = 10 call this equation [2] Make x the subject of equation [1]. © Make a oeeeecee| x=7-y ies — Substitute forx in equation [2], 4) Substitute. 2x + ay = 10,s02(7 — 2y) + 3y = 10 call this equation [3] Equation [3] now has only one variable (letter). Solve this (find the variable) by multiplying out the bracket and collecting like terms. 1d-4y+3y=10 4 @ Solve the new equation. 14-y=10 14-10=y 4=y Substitute this value of y back into equation {1}. x=7-2y 4 @ Substitute this value into the first x=7-(2%4)=7-6 equation to find the value of the other xml variable. 39. Complete Advanced Level Mathematics: Pure Mathematics 40 ‘The values of x and y that simultaneously satisfy both equations are x= = 5 b Gradient of a line perpendicular to AB is ~ } Urheberrechilich gaschitztesspaterial Complete Advanced Level Mathematics: Pure Mathematics 2.1 Coordinate Geometry Exercise Technique 1 | Find the shortest distance between the following pairs of points: a (3,5) and (1, 4) e (-2,-3) and (—7, -1) b (3,5) and (5, 6) f (8, ~4) and (~7, -4) © (1,7) and (—2, 3) 8 (2,1, 5)and (5, 13, 9) d= (1,7) and (0, -1) h (7, -2, 18) and (—3, 3, 6) 2 | Find the length of the straight line joining the following pairs of points: A(?, -1) and B(~2, 5) A(4, ~8) and B (0, 6) A(-1, ~2) and B(—2, -5) A(4, 4) and B (1, 2) A (3, 2) and B (=1, =5) A(6, 11, =3) and B (1, 1,7) A(-2, —1) and B (0, 3) A(2,3, 4) and B (4, 6, 10) maoe sane 3 | Find the gradiont of the straight line formed by joining the following pairs of points: a (3,2) and (5, 12) d= (0,7) and (—2, 9) b (2,1) and (4,9) e (-2,-1) and (6, ~1) « (5,3) and (7,1) f (3,2) and (5, -8) | 4 | For the following pairs of points, A and B, find: i the gradient of the line AB ii the gradient of a line perpendicular to AB. a A(0,6) and B (2,7) d-A(-3,6)and B (~1,-3) b A(5, 2) and B (3, ~3) e A(-3, =2) and B(6, -6) e A(-3, 0) and B(2,—5) f A(-2,0)andB(7, 2) 5 | Rearrange the following equations to make y the subject. State whether the pairs of lines are parallel or perpendicular to each other. Y= W+3and y= y=3ax4+7andy y= 2x—Sandy—2x=3 ay +6x+8=Oandax+y=-7 ax +2y =6and 4y =9 +x 3y = O(x — 1) and 6y +.2x =6 memnoe 2 Coordinate Geometry Contextual 1 | Katie moves her position from the point (2, 6) to the point (5, 3) on the park map. The map is drawn toa scale of 1:10 000. Find a__ the shortest distance in centimetres thal Katie covers on the map b the actual distance she moves in km. 2 | Twins Peter and David radio their respective coordinates to each other. Peter is at position (3, 9) and David is at (—2, —3). How far apart are they? 3 | The night before military manoeuvres, an army troop are given starting coordinates, (965, 386), and finishing coordinates, (943, 379). By considering only the final two digits of each coordinate, find the distance on the Ordnance Survey map moved by the troop, in centimetres. If the scale of their map is 1:25 000, find the actual distance covered in kilometres. | According toa garden plan, the cottage (7, 9) is the same distance from the ash tree (5, 2) and the beech tree (9, 2), Investigate this statement. A parallelogram is formed by lines joining the points P, Q, Rand S. Given the coordinates of points P (~2, 3), Q (3, 4) and R (2, —1): a Find the coordinates of S. b Show that PQRS is a rhombus, 6 | The diagram shows a sketch of a cuboid. Given the coordinates of A(1, 2,1) and F (4, 5,5), find the shortest distance between A and F. F(4,5,5) A(t 2.1) ‘Complete Advanced Level Mathematics: Pure Mathematics 2.2 The Equation of a Straight Line Linear aaah can be written in many forms. The general form is ‘whore a, b and care real numbers. Example 1 Rearrange the following equations into the general form ax + by + c= 0: a y=-3"-8 b y+2=-1(x-1) Solution a ax+y+8=0 b y4+2=-1(x-1) 3y +6 = -(x-1) 3y +6 =-x41 x+3v+5=0 ‘The general equation can be rearranged to make y the subject. It is then writ ten [ames ¢ When written in this form it instantly highlights two important properties; the gradient of the line and its intercept with the y-axis, Consider the line y = Ly — 3, This is in the form y = mx +c, with m = }andc = —3, Notice that the gradient is } and the y-intercept is (0, —3), ¥ P74 The point where a line | erosses the y-axis is | known as the intercept ‘The equation y = $x ~ 3 can be written in other forms: yokx-3 Multiplying througho Qve=x-6 by 2 removes the x-2y-6=0 2 Coordinate Geometry In the last version, the equation is in the more general form ax + by +c = 0, with a= 1,b = —2 and c= ~6. Notice that y= 3x —3 and x ~ 2y — 6 = 0 both represent the same straight line. Example 2 Find the gradient and y-intercept of the straight lines represented by the following equations and sketch their graphs: a ox+ay-7=0 b 2x-5y+1=0 Solution a Ox+3y-7=0 + By=-3x47 ~ox 7 z yesptgeety So the gradient is ~1 and the y-intercept is (0. 3). n 4 Check this sketch using a graphical calculator. ax+ay=7=0 bo o2w-sy+1=0 + axd1=5y Reedley So the gradient is ? and the y-intercept is (0, }). ¥ 4 Chock this sketch using a graphical calculator. ‘The equations can be entered into the calculator in the form . ‘The technique of rearranging equations into the form y = mx + cis particularly useful when solving simultaneous equations with a graphical method. Complete Advanced Level Mathematics: Pure Mathematics Example 3 Solve the simultaneous equations x+y =4 and 2y — 3x = 3 usinga graphical method. Solution We could rearrange each equation into the form y = mx +c to identify the gradion! and intercept for each graph. Hawaver, itis often quicker to draw each graph by calculating where they eross the axes. ‘To find where the lines cross the y-axis, put x = 0-into each equation. To find where the lines cross the x-axis, substitute y = 0 instead. Forx+y=4, when x=0,y and when 5 =4 ‘This line crosses the axes at (0, 4) and (4, 0). Foray —ax=3, when x=0, 2y=3 y=t and when y =0,-3x=3 xed ‘This line crosses the axes at (0,3) and (~1,0). ‘The graphical solution of the simultaneous equations is given by the coordinates of the point of intersection of these two lines, You should find that the coordinates of this point are.x = 1 and y = 3. So the point (1, 3) lies on both lines and x = 1,,y = 3 is the solution because it satisfies both equations simultaneously. chilich geschitztes Material 2 Coordinate Geometry 2.2 The Equation of a Straight Line Exercise Technique [41] Rearrange the following equations into the form y = mx +e: Bety+7=0 e xtyed=0 b ax+y-a=0 € ax-y-5=0 2 | state the gradient and the y-intercept of the straight-line graphs produced by the following equations: a y=5y-3 b © y=7-2x d For the straight lines produced by following equations, find the gradient and the coordinates of the y-intercept: a wry+a=o do aw-ys7=4 b -2x+ay-2=0 e -3x+7y=14 © 5x+1y—2=8 £ axt+by+e=0 [4] the equation of the line shown is given by 9x +y -6= 0, Find the gradient and the coordinates of A and B. ion 2” [B| Rearrange the equation S$ = 3 the graph of this equation. into the form y= mx +e. Now sketch [6] soive the simultaneous equations x +y = 6 and 4x ~2y +6 = 0 usinga graphical method. Urheberrechtlich geschiitzte Complete Advanced Level Mathematics: Pure Mathematics 2.3 More on the Straight Line Think about the information needed to describe a particular straight line. How can we write the equation of a line by looking only at the graph of the line? We find that we can write the equation if we know: @ the gradient of the line and the coordinates of a point on it © | thocoordinates of two points on the line. The equation of a line given its gradient and the coordinates of one point on the line Suppose the gradient m and the coordinates of point P (x,,.¥%) on the line are known. A general point (x,.7) on the line can then be used to find the equation of the line. The known gradient, m, can be expressed by the sides of a right-angled triangle drawn on P and the general point (x, 9). General point wy) gradient yn xox ‘This can be rearranged into the very useful result, GREE x) < Learn this resuit. Example 1 a Find the equation of the straight line with gradient 2 that passes through the point (3, 4). b Find the equation of the straight line with gradient } that passes through the point (—2, —6). Solution a Use yyy = m(x— x4) Substitute the known values of m, x; and 3 ‘Then y — 4 = 2(x—3) yoda dv-6 2 Coordinate Geometry This equation can now be rearranged into either form of the equation ax + by +¢ =0, or ofa straight line. vemx+c, x-y=2=Oory=2r—2 b Use y=y, =m(x—x,) ‘Take care when manipulating the vtG=hxe1 yodxe1-6=4x-5 So the equation of the line is y = x — 5. The equation of a straight line passing through two known points Suppose that two points P (x,,.¥;) and Q (xs, ys) are known to lie on the straight line, The gradient of this line can be found by drawing in a right-angled triangle. 2M Now that we have the gradient, the problem is simple. We can use the resull y —y, = mw — x4) on either Por Q. Example 2 Find the equation of the straight line that passes through the points A (2, 3) and B (4, 4). ich ges chitztes torial Urheberr Complete Advanced Level Mathematics: Pure Mathematics: Solution The gradient of the straight line passing through A (2, 3) and B (4, 4) is Now use y —y, = m(x —;), with m = Sand B (4, 4) as the known point. Vas, =m(x-x,) + y-4= 3-4) yoda dx-2 v= 4xt2 Check that you arrive at the same equation using A (2. 3) instead of B(4, 4). ‘The expression for the gradient, m= *2—**, ean be substituted directly 2 into the equation for a straight line, y — y; = m(x — x,). At first the algebra might appear quite daunting, but it provides a very useful result. iy ae EAMG FVt mom ™) This equation can now be rearranged so that the x terms and y terms are separated, producing the equation of the straight line directly once the valuos of (x;,y;) and (xz,y2) are known, x x 4 Learn this result. Example 3 Find the equation of the straight line joining P (5, ~6) and Q(~3, 2). Solution ‘Using the result yrv v2 ‘Simplify the numeric components. 8 presses) V+6= —(x—5)=-x45 Yo -x+5-6=-x-1 Make y the subject. Check that this equation could also be written x +y-+1=0. Urheberrechtlich geschotztes Material 2 Coordinate Geometry Finding the mid-point of a line Given a line joining two known points the mid-point can be established Recall that the mean using the mean of the x and y coordinates. average is the sum of the numbers divided by how many of them there Consider the line joining A (1, 2) and B (7, 10). IfM is the mid-point of this line then M is half-way between A and B, both horizontally and vertically. In this case the coordinates of Mare (4,6). Example 4 ‘The vertices of an isosceles triangle are A (2, 7), B (5, 8) and C (4, 5). a State the coordinates of the mid-point of AC. b Find the equation of the straight line through B and the mid-point c oie equation of the perpendicular bisector of BC. Solution y ol x a Let Mbe the mid-point of AC. Then the coordinates of M are Ce. 7). (3,6). Utheberrechtlich geschitt 2 Coordinate Geometry 2.3 More on the Straight Line Exercise Technique [1] Find the equation of the straight line with the givon gradiont passing through the stated point in each of the following: a gradient 3, point (3, 2) d_ gradient —3, point (0, 4) b gradient 6, point (—1, 2) e gradient }, point (2, —3) © gradient 5, point (3, -2) f gradient —4, point (1, 4) [2] Find the equation ofthe straight line joining the following pairs of points: a A(2,4) and B (3, 6) dA (0, -2) and B (3, 4) Bb R(-3,4)and § (1, 2) e P(8,6) and Q(2, 12) ¢ T(-1, 1jand V (0, 6) f R(-1,-1) ands (5, 2) nd the mid-point and the equation of the perpendicular bisector of AB in each of the following cases: a A(1,—1and B (3,7) ce A(-2,5)and B (0, 3) b A(4,1)and BIS, 0) ad A(-1,-2)and B (1, 6) Contextual A straight line passing through the points A (—1, 1) and B (p, 13), has gradient 2. Determine the value of p and find the equation of the straight Tine. [2] Consider two points, P (2. 7) and Q (4, 13). Find the mid-point of PQ. Find the gradient of PQ. Write down the gradiont of the line perpendicular to the line PQ. Find the equation of the perpendicular bisector of PQ. Write it in the formax +by +¢=0. ane [3] Sketch a diagram to show the points A (0, —1), B (4, 3) and C (4, 5). Let M be the mid-point of AB. Find the coordinates of M and write down the ‘equation of the straight line that passes through M and C. [4] Ais the point (6, 6) and B (8, 2) lies on the straight line x — 2y — 4 = 0, a Find the equation of the straight line parallel tox — 2y — 4 = 0 that passes through A. Write it in the form ax + by + b Show that the straight line joining A and B is perpendicular to the linex=2y -4=0. ¢ Find tho perpendicular distance between the two parallel lines. Complete Advanced Level Mathematics: Pure Mathematics 70 2.4 Inequalities ‘There are four inequality symbols. > means ‘is greater than’ Learn the mathematic > means ‘is greater than or equal to! meaning of each of < means ‘is less than’ these symbols. <__means tis less than or equal to! Inequalities produce a range of acceptable answers. They can be represented on number lines using arrows. The base of the arrow is circular, and is shaded when the value is to be included in the range, and clear when the value is not to be included in the range. x23 —_—> 2 a 0 t & & 4 © xP oe 2H o ot @ 3 4 6 Inequalities can be categorised into two types; those without variables. (letters) and those with variables. Inequalities without variables are called. propositions. Statements such as 2 < 7,4 > },and —3 < —2 are propositions and are either true or false. Inequalities with variables can be solved using similar techniques to those used when solving equations. However, a solution sot is often produced showing a range of acceptable answers. Example 1 Solve the inequalities: a Ww+1>7 b 1-227 © Sxt2<3x+10 d St¥ eer Solution a WHS? Ww>6 x23 bo 1-2x27 + -236 Notice that the coofficiont of x is —2. To find x we need to divide both Check this result by sides of the inequality by ~2. When dividing by a negative number substituting value o the inequality needs to be reversed. smaller than —3 in th é original inequality. D ntr26 xs the original inequalit work? x<-3 2 Coordinate Geometry ¢ The process of solving linear inequalities is similar to solving linear equations. Collect the like terms together, with numbers on one side of the inequality and variables on the other. Sx+2 Sx-3x< 10-2 Sede ae sf value of x = Sst smaller than 4 in the => x<4 orig ua Again use a similar process to that used when solving linear equations. Eliminate the fractions (by mulliplying by a common multiple), collect like terms and find a condition on x by using division, Gtx Set? > G4+x23(K+7) = G+x>3x421 = 6-212 9x—x = 15> ax = -BEx ‘That is, x < —48 Notice how the following rules were used: @ Any term can be added to, or subtracted from, both sides of the inequality and the symbol doesn’t change. @ Both sides of an inequality can be multiplied, or divided, by the same positive number and the symbol doesn’t change, @ Whon both sides of an inequality are multiplied, or divided, by the same negative number then the symbol is reversed. Quadratic inequalities One example of a quadratic inequality is x* > 4, Il has two sets of solutions, Consider x” = 4. This has two solutions; x = 2 and.x = —2. Why? Because to solve the equation x? = 4 we take the square root of both sides of the equation. 4 xs4va=42 So what do we know about x when x* > 4? We can see that x > 2 works, Check this result, What is the other solution? Is there a condition involving —2? We find that x < ~2 is also a condition that works. chilich geschitztes Mahia Complete Advanced Level Mathematics: Pure Mathematics To see why the solutions are x > 2 and x < —2, think about the proposition x* > 4. Draw two graphs; y =x* and y = 4. Where is the parabola above the line y = 4? This is the same as asking for which values ‘of x the graph of y = x“ is above the line y = 4, or for which values x? > 4. ‘The curve is above the line forx > 2 and for x < —2, yea An alternative is to rearrange the original inequality. Then x* > 4 becomes x* — 45 0. This may not look simpler, but the new statement has a quadratic expression and a zero separated by an inequality symbol. A graph of y = x? — 4 can now be drawn, and we are looking for the points ‘where y > 0 (that is, for points of the curve above the x-axis), » Find the points where the curve crosses the x-axis (by solving x? — 4 = 0, or using your graphical calculator), Notice that the curve is above (greater than) the x-axis (y = 0) for x > 2 and x < —2. ‘This technique of sketching the graph is a useful way of checking that no solutions have been lost. Example 2 Solve: a x? 7x <-10 b x*-3x-5>0 ce xtix4+1<0 Solution a This is a quadratic inequality. It can be rewritten as a quadratie expression and a zero separated by an inequality symbol. xt -7e<-10 + x? - 74 10<0 2 Coordinate Geometry ‘The quadratic expression can be factorised. Recall ‘PAFF’ from Recall that since the Chapter 1. coefficient of x? is 1 the expression can be P: 10 A:-7 F:-2,-5 factorised from this step, ‘Then x” — 7x +10 < 0 becomes (x ~ 2)(x —5) < 0. What does this expression suggest about x? If the inequality symbol was an equality (that is, (x — 2)(x— 5) = 0) then x = 2 or x= 5 would be the solution. Since we have an. inequality, these critical values should be examined more closely. Begin by sketching the curve y = x" — 7x + 10. Notice how it crosses the x-axis atx = 2 and x =5. yt +10 ‘The parabola is below the x-axis for all values of x between x = 2 and x =5. This means x* — 7x + 10 is negative for all these values of x. So. x? —7x +10 < Owhenx > 2andx <5. Another way of writing this set of inequalities is as a ‘sandwich’: 2 0) when x <}(3 = V29) and x > 4(3 + v29). These inequalities are separate and cannot be condensed into a ‘sandwich’. ¢ Remember that the technique has been to identify critical values. By sketching the graph a set of inequalities has been identified where the graph is above or below the axis. Sketch the graph of y = x? + +1. What happens? ‘The graph doesn't cross the axis. The expression has no eritical values. Check this by trying to factorise x* +x +1, or use the quadratic formula, Since the curve is always above the x-axis, xt +x +1 is never negative, and x? +x +1 <0 has no solutions. In summary the technique for solving quadratic inequalities is as follows: ® Establish zero on one side of the inequality symbol. This means the quadratic expression can then be tested for being positive (>0) or nogative (<0). ® Establish the critical values. These are the values of x that make the quadratic expression equal zero. Identify the set of inequality solutions. Often this can be done by sketching a suitable graph. @ Decide which sides of the critical points form the solution set. 2 Coordinate Geometry An alternative method is to check the sign of the expression below the smallest critical value, between the critical values and above the largest critical value, Example 3 Solve 2x7 — gx 4.9 > 0. Solution ‘Notice that the first step has been done. This is a quadratic expression that needs to be positive (20). Now use PAFF to factorise the expression 2x? 9x+9, <4 2) Identify the critical values. P:2x9=18 tie) FF: 2x*-9x+9=2x"-6x-3x+9 = 2x(x — 3) - 3(x - 3) = (x —3)(2x — 3) So the critical values are 3, when x — 3 = 0, and 3, when 2x — 3 =0. Notice that the problem has changed from solving 2x* — 9x + 9 > 0 to salving (x ~9)(2v— 3) 20. ‘We will use the method where we check the sign of the expression against the critical values. Look at the sign of (x —3)(2x — 4) by comparing the signs of the separate factors. x<} po (x-3) | H (2-3) - { + ' + (x=3) (2x-3) Ox) Gx Hx) positive negative positive So (x ~ 3)(2v ~ 3) is positive when values of x are smaller than the least critical value, and larger than the highest critical value. ‘The set of solutions for 2x* — 9x + 9 > 0 isx < $and.x > 3. Check this result by sketching the graph of y = 2x*—9x +9. Urheberrechilich gasch 8 Complete Advanced Level Mathematics: Pure Mathematics 2.4 Inequalities Exercise Technique 1 | Solve these linear inequalities: a x+3>8 b ax+3s7 fa 24x © OxF22x+8 d SFcxte 2| Solve these quadratic inequalities: a x59 b x25 ce x 4950 d xt-64<0 3 | Solve these quadratic inequalities: a eq a x 48x+15>0 b x*+8y-6>0 c¢ x7 +7v4+10<0 do x?-2x-15<0 e x*-Sx+6<0 ff x*+3v-4>0 4| Transform these statements into quadratic inequalities invalving zero. Solve the inequality in each case. a x*—10n> 24 b x+x56 e x? -9 e x <4x+77 { xt+4<4x 5 | Solve these quadratic inequalities: a 3 +ax+2>0 b Ix? +22x+3<0 ¢ ax +5x4+2<0 do axt+1>4x e 2x? > x43 f axt+x>2 Contextual 1 | Solve the inequality x +3 > x*, leaving your answer in the form a+ bVé where a, b and c are rational, 2 | Find an inequality represented ' by the highlighted section of this graph. 3 | Find the set of values of x for which (ax - 1)? < 3x7 +13. 4 | Solve the equation x7 — 5 vZx + 12 = 0, writing your answer using surds. Hence, or otherwise, solve x* — 5V2x + 12 < 0. 76 2 Coordinate Geometry 2.5 The Equation of a Circle Consider a circle of radius r whose centre is at the origin, and let P (x,y) be any point on the circle. This means the distance OP must always be equal to the radius of the circle Pythagoras’ theorem can be used to find the relationship between x, and ‘The equation of a circle of radius r whose centre is at the origin (0, 0) is: aye What happens if the centre of the circle is moved to a new position Q (a,b)? Again, let P (x.y) be some point an the circumference of the circle, Notice that PQ is a radius of length r. A now right-angled triangle can be drawn on PQ so that the shorter sides are parallel to the x-axis and. y-axis respectively. x ‘The lengths of these sides are (x — a) and (y"~ b). Check this from the diagram. Now use Pythagoras’ theorem on this triangle. What happens? es -- ‘This is the equation of a circle of radius r whose centre is the point (a,b). rrechtlich geschiitzted fatarial Complete Advanced Level Mathematics: Pure Mathematics Example 1 State the radius and centre of the following circles: a (x-7)? +(y42)*=36 b (xta)?t(y—s)?=23 Solution a Compare (x ~ 7)" + (y+ 2)* = 36 with (x— a)? +(y—b)* =r. The equation is in the same form with a = 7, b = ~2 and r= So (x ~ 7)! + (y +2)* = 36 is the equation of a circle of radius 6, centre (7, —2). b (x +1)? +(y— 5)? = 23 can be compared with (x — a)? + (y —b)? =P, togive a = -1,b =Sandr = 23. So (x + 1)° + (vy —5)? = 23 is the equation of a circle of radius 23 whose centre is (~1,5). Example 2 a The point (k,2) lies om the circle x* + y* = 13. Find the values of k. b The point (k,0) lies on the circle with centre (7,2) and radius v8. Find the possible values of k. Solution a Substitute (k, 2) into the equation of the circle. Then x? +y7=13 > Ke 42% = 13 > B+a=2B = K=9 ed k=2V9=43 So there are two possible values of k. These can be interpreted geometrically by sketching a diagram of the circle. vk (3.2) Urheberrechilich geschtitztes Material 2 Coordinate Geometry b The equation for a circle of radius v8, centre (7,2) is given by (x- 77 + (y-2y =8 Now put x = k when y = 0 into this equation, (k-7)? + (0-2)? =8 = (k-7¥+4=68 5 = (k-7)'=4 > (k-7)=4va=22 = =7H2 Sok=9ork=5, ‘We have seen that the equation of a circle of radius r and centre (a,b) can be written (x - a)? + (y= b)* = r*.Ican also be written in another form by multiplying out the bracketed terms and then collecting like terms. (xa) +(y-b)? =7 => t= 20x +0") + (y= 2by +B) = = x) ty? —2ax -2by + (a? +b? - 7) =0 Since a, b.and r are all constants this can be ‘simplified’ to the general form of the equation for a circle (by writing a® +b? —r* = c). a> -» Notico that in this form: @ The centre (a,b) can be identified from the coefficients of the x and y terms. @ The radius is not as straightforward to identify as it was in the other form, Example 3 a Find the equation of a circle of radius /7 and centre (3, —2) in its general form. hb The equation of a circle is x? + y? — 2x + 4y — 4 = 0. Find the centre and radius of the circle, Solution a Ifthe centre is (3,—2) and the radius v7 then the equation of the circle is, (3)? + (y +27 = (v7? Baicrial Urheberrechtlich gesch (Complete Advanced Level Mathematics: Pure Mathematics Expanding the brackets, (x*— 6x +9) + (y? +4v +4) =7 = x+y? —ox+4y + (944-7) > xt 4y" -6x+4y+6=0 b Compare x* + y* ~ 2x + 4y ~ 4 = 0 lo the general form of the equation fora circle, x* + y* — 2ax — 2by + ¢ = 0. Notice that. and b can bo found by equating coefficients of x and y. Equating coefficients ofx, — —2 2a, ‘Equating coefficients of y, 4=—2b, sob = -2. So the centre of the circle, (a,b), is (1,—2). Now equating the constant terms in each equation, ¢ = Reeall that ¢ = a? + b* — * So P+ a4 = 0) +(-2?-P ==4 > 1$4-Ps—4 = Psltaee > Poo > r=3 So the centre of the circle is (1, -2) and the radius is 3. Tangents and normals A tangent to a circle is a straight line that touches the circle at one distinct point. A normal is the straight line perpendicular to the tangent that ‘passes through the point of contact between the tangent and the circle. Notice that the normal is an extension of a diameter. Normal ‘Tangent Example 4 Find the equation of the tangent to the circle x* + y? — 3x -—y -2=0at the point (3, 2). Then find the equation of the normal. 2 Coordinate Geometry Solution Compare x? +" ~ 3x — yy — 2 =0 tothe general form Both the tangent and the x? 4 y? — 2ax— 2hy += 0. The values of a and b can be established by normal pass through equating coefficients. (3, 2); we know one point on each line. The Equating coefficients of x, soa=}. ‘normal passes through Equating coefficients of y, sob=}, the centre of the circle, so start by finding the So the contre of the circle, (a,b) = (J). t ‘The gradient of the normal can now be found by using the points (3, 2) and (.)). gradient of normal = il " wna Since the tangent is perpendicular to the normal its gradient must be —1. We now know the gradient of the tangent and the coordinates of a point ‘on the tangent (3, 2), so the equation of the tangent is given by ¥—¥, = m(x—™) > > We knew another point ‘on the normal (the centre of the circle), so ‘we could also have used the technique for finding the equation of a straight line knowing > yoxt1=0 two points on that line. Similarly, we know the gradient of the normal and the coordinates of a point on the normal, so the equation of the normal is given by oy = mx —™) + y-2=1(K-3) > y-2=x-3 sBMrorial Urheberrectttich geschi Complete Advanced Level Mathematics: Pure Mathematics 2.5 The Equation of a Circle Exercise Technique [11] Find the contre and radius of each of the following circles: a (x5) +(y—a)h=7? be @w+o)F+(y+ay=25 © +(y+7)F 121 do (x+1)?+(y-2)'=11 [2] ‘Write the equations of the circles with the given centres and radii in the form x? + y? —2ax — 2by +e= 0: a centre (—1,3), radius 2 b centre (2,~1), radius 3 © centre (—2, 0), radius V5 d contre (3, ~3), radius 3v2, 3 | Find the centre and radius of the following circles. a xt4y?—6x—-2v+6=0 bo x+y? t2x-4ye1=0 co xttyt+oxs+ay+e=0 do @yytay+t=0 Contextual [1] the point (3.x) lies on the circle x? +y? — 6x —4y —51 = 0. Find the values of K [2] The point (k.0) lies on the circle with centre (4, 1) and radius V0. Find two possible values of k. [3B] The point A ties on the circle with contre (1,3) and radius V3. Given that A lies on the y-axis, find the possible coordinates of A. [4] Find the equation of the tangent to the circlo.x* + 4x +y* = 21 at the point (1,4). Find also the equation of the normal at this point. [5] Asstraight tino touchos tho circle x* — ax +y* ~ 10 — 71 = 0 at tho point (8, 3). Find the equation of the line and the equation of any line perpendicular to it passing through the point (4,3). [16] the points A (—7,7) and B (1,1) form the diameter of a circle. Find the equation of the circle, Find the length of the tangent from the point (9, 8) to the circle x+y? -2x-4y=31. 2 Coordinate Geometry Consolidation Exercise A [1] 4.8.C are the points with coordinates (4,7), (~1,2) and (6,1) respectively. a Prove that the triangle ABC is isosceles. State the coordinates of the i mid-point, M, of AC and find the area of the triangle. ‘ Find the equation of the line BM. Find the equation of the line through A, perpendicular to BC. d_ Find the coordinate of the point H where these two lines meet, and deduce that CH is perpendicular to AB. (OCSEB) 2 | The coordinates of the points A and B are (2, 3) and (4, —3) respectively. Find the length of AB and the coordinates of the mid-point of AB. (UCLES) 3 | P,Q, Rare the points whose coordinates are (2,4), (@, —4) and (14, 8) respectively. a Find the equations of the perpendicular bisoctors of the lines PQ and PR. b the two bisectors meet at C, calculate the coordinates of C and show that GP* = 50. © —Deduce the equation of the circle through P, Q. and R in the form iy? 4 prtqvir=0. (OCSEB) [4] Aline through the origin with gradient m cuts the fixed circle in Fig. 1 in two points provided that 2(2m +1)" > 9(m? + 1), Show that this inequality is equivalent to m* — 8m + 7 <0 and find the solution set for m. o Fig. (OCSEB) Urheberrechtlich geschitztes MBSrial Complete Advanced Level Mathematics: Pure Mathematics 5 | Find the length of the tangent from the point (6, 2) to the circle xtiyt4ox =o. G | Solve the inequality (x — 3)(x —6) > x +9. (AEB) Exercise B 1 | tho coordinates of the points A and B are (3, 2) and (4, —5) respectively. Find the coordinates of the mid-point of AB, and the gradient of AB. Hence find the equation of the perpendicular bisector of AB, giving your answer in the form ax + by + c= 0, where a, b and c are integers. (UCLES) 2 | Tho straight lino P passos through the point (10, 1) and is perpendicular to the line R with equation 2x + y = 1, Find the equation of P. Find also the coordinates of the point of intersection of P and R and deduce the perpendicular distance from the point (10, 1) to the line R. (UCLES) 3 | Find the length of the tangent from the point (7, 6) to the circle sty? an = 18, 4 | Find the equation of the straight line that is parallel to y + 20x = 90 and passes through the point (4, —10). (NEAB) 5 Pind the sot of values for which —* > 2, (ULEAC) G | Acircle, centre P, passes through A (1, 1), B (—2, 2) and G (~7, ~3). a Find the equation of the perpendicular bisector of AB. b Find the equation of the perpendicular bisector of BC. © Using your answers to a and b, solve the equations simultaneously to find centre P. d_ Find the distance AP. & Hence write down the equation of the circle. 84 rheberrechtlic chit Material 2 Coordinate Geometry Applications and Activities Constructing a circle through three known points Mark three points anywhere an a piece of paper. Now try to construct a circle that passes through all three. Gan it always be done? Mark the three points on a piece of graph paper so that their coordinates can be read, Repeat the problem but this time find: a the coordinates of the contre of the circle b the radius of the circle and ¢ the equation of the circle, Summary The formula for the distance between points A (x;,1) and B (x2, ¥2) is The gradient of a line is a measure of its steepness. change in y-coordinate change in x-coordinate Parallel straight lines have the same gradient. gradicnl ‘The product of the gradients of perpendicular straight lines is —1. The equation of a straight line is generally written in the forms = mx +¢(y =ayx+ a9) and ax + by +6 =0, ‘The equation of a straight line with gradient m passing through (x1,¥41) is, FERPA -~) ‘The equation of a straight line passing through (x;,/1) and (2,92) is x=x 2% ‘The mid-point M of a line joining the points A (x),41) and B (xz,¥2) has coordinates a) The symbols >, <, <, > mean ‘greater than’, ‘less than or equal to’, ‘less than’ and ‘greater than or equal {o', respectively, Complete Advanced Lave! Mathematics: Pure Mathematics @ Linear equalities can be represented on a number line using o~ and «as appropriate, © Quadratic inequalities can be solved by rearranging them as factorised quadratic oxpressions with zoro on one side. Solutions can be checked by drawing graphs. @ The equation of a circle has general forms sy -- and (E+ <-> @ = Acircle, (x — a)’ + (y — b)? =/°, has centre (a,b) and radius r. @ = The normal toa cirele at a given point is perpendicular to the tangent at that point, and passes through the centre of the circle. 86 Urheberechtlich geschiitztes Material 3 Trigonometry | What you need to know How to use Pythagoras’ theorem. Factorisation methods, including factorisation of quadratic equations. How to write down the sine, cosine and tangent ratios for acute angh How to find the area of a triangle. ‘That the term solving a triangle means finding the lengths of the unknown sides and the sizes of the unknown angles. How to calculate bearings. Review Use Pythagoras’ theorem to find the length of side AB in the following © o triangles: Pythagoras of Samos (e. 560-c. 480nc) a B b 26 A Pythagoras provided the c 6 24 A 3 ¢ a cA B aoc 2 Lt 2 B 6a Cc A a abt b @+3c+2 a c h+h-12 f 3p'+14p-5=0 87 Complete Advanced Level Mathematics: Pure Mathematics [3] write down the values of sinx, cosx and tan x forthe following triangles: i e 3 b 60 c 61 a é [4] catculate, to thee signifieant figures, the areas of the following triangles: a b c o Ly ; \. % = 7m 5 | Solve the following triangles: aA b G es oe * a c 4 B © Go dq 22cm B A 5m) 61cm elem © B c G | For cach of the following diagrams write down the bearing of B from A: c x ooaN x N A 30° BN ose B A 88 Urheberrechtli Trigonometry | 3.1 Trigonometric Functions he origins of the ‘Trigonometry is the study of angle measurement, and in particular the devsopaat mi study of triangle measurement and calculation. In order to distinguish Sigeromety ae between angles and lengths of sides the convention of capital letters for scone was largely. a vertices and lower case letters for the corresponding opposite side is mur of algebra and adopted. geometry, used ‘Whenever lengths and B angles needed to be calculated. hypotenuse ie Me side ‘opposite & Cr a b c side adjacent to @ Right-angled triangles are used to define the three basic trigonometric functions for some acute angle 4; sine, cosine and tangent. Recerstirwmsncera ‘abbreviations sin side opposite 0 coed _b adjacent to ¢ and cos were “hypotenuse “et hypotenuse introduced by Wiliam ‘Qughtred in his book: side opposite 0 published in 1642, He abo introduced the > sign for ruiply, side adjacent to 0 This principle can be used to define the sine, cosine and tangent of any angle 0. Draw perpendicular axes Ox and Gy, and a circle centred on the origin, with radius 1 unit. Then é will fix some point P on the circle. The first quadrant The coordinates of P (x,y") are then (cos 0, sin). Now adopt the convention that @ is measured anti-clockwise from the positive x-axis. The quadrant between the positive x-axis and the positive y-axis is called the first quadrant, In this quadrant 0 is always acute. ‘The second quadrant is between the positive y-axis and the negative x-axis. In this quadrant 0 is always obtuse. When @ is obtuse (greater than ‘Complete Advanced Level Mathematics: Pure Mathematics 90°) sin@ and cos ¢ are equal in magnitude to the sine and cosine ratioof ‘The magnitude is the the acute angle (180° — 6). So in the second quadrant the coordinates of P numerical value, or s are still (cos 0, sin 0), but note that in this quadrant sin is positive, and _of the trigonometric cos@ is negative, -< sin (180° — ) =sin 0 ratio, ignoring the sig 0s (180° — 0) = -cos 0 {positive or negative), ‘The second quadrant Example 1 Find cos 147° asa trigonometric ratio of an acute angle. Solution We know that 147° lies in the second quadrant and, for obtuse 0, cos = — cos(180° ~ 0). So.cos 147° = —cos(180° — 147°) = — cos.33°. By making 0 a reflex angle, we can extend these results into the third and fourth quadrants. In the third quadrant both sin ? and cos@ are negative. In the fourth quadrant cos 0 is positive and sin 0 is negative. (Think carefully about the coordinates of the point P). 0° 0° ( 0° “| e* 20 20" The thind ‘The fourth quadrant quadrant Remembering the definition of cos@ and sin @ as the coordinates of P, the gradient of the line OP gives us tan 0. This allows us to establish the quadrants in which each of the three trigonometric ratios are positive. One way of remembering which theberrechtlich geschiitztes Material 3 Trigonometry | trigonometric ratios are positive and which negative in each quadrant is to remember only the positive ones. Think about the coordinates of P(cos@, sind) and remember that tan 0 is positive when sin 0 and cos0 have the same sign. All the trigonometric ratios are positive in the first quadrant, sine is positive in the second, tangent is positive in the third and cosine is positive in the fourth quadrant. There are several good mnemonics (aids to memory), such as All Silly Tom Cats: All Squirrels Take Chestnuts; All Silver Tea Cups. Example 2 Write tan 227° as a trigonometric ratio of an acute angle. Solution We know that 227° lies in the third quadrant, and that tan 0 is positive in the third quadrant, So tan 227° = tan(227° — 180°) = tan 47° Special angles. Some acute angles are special because they occur so frequently. Two triangles in particular are very useful for finding the trigonometric ratios of these angles. These triangles have the advantage of giving exact results and not decimal approximations. @ An isosceles right-angled triangle with sides 1 unit, @ | Halfof an equilateral triangle of side 2 units. {| Complete Advanced Level Mathematics: Pure Mathematics Using these triangles we have the following special results, Copy and memorise _ ie 30 i ¥ oe # & # : 60 2 1 va This now allows the ratios of related angles in the second, third and fourth quadrants to be evaluated exactly. Example 3 a Find sin 150°. b Find cos 330°, c Find sin 410° as a trigonometric ratio of an acute angle, Solution a We know that 150° lies in the second quadrant, where sine is positive. So sin 150° = sin(180° — 150°) = sin30° = 4. b We know that 330” is in the fourth quadrant, and cosine is positive in the fourth quadrant, So cos330° = cos(360° — 330°) = cos 30° = 2. © Angles outside the range 0’-360" always va lie in one of the four quadrants, We find that 410° — 360° = 50°, so 410° is in the first quadrant, where sine is positive. So sin 410° = sin(410" — 360°) = sin 50" Graphs of the trigonometric functions Now draw a new pair of axes and plot the angle @ along the x-axis and the y-coordinate of P(sin0) along the y-axis. This gives a continuous curve. This curve, or wave, repeats itself every 360’, so the sine curve is said to have a period of 360°. The curve has a maximum value of 1 (when = 90°) and a minimum of —1 (when = 270°), These correspond to P being at the top and bottom of the circle. he systematic st trigonometry is altributed to Hippar an Alexandrian astroy who lived around 15¢ He made a table of sis and used tt find th Aitance between the Moon and the Earth, sing, 3 Trigonometry | ‘You can draw a similar graph plotting the angle 0 along the x-axis and the a-coordinate of P (cas 0) along the y-axis. The graph of cos 0 is the same shape as that of sin@, but it has been shifted by 90°. The 90° is sometimes referred to as the phase difference between the two graphs. cos @, ‘The similarities can be described using the following equations, IO) cos OHNE) = a BOE) sav GHEY sino Example 4 Show that sin 120° = cos 30°. Solution sin 120° = sin(1a0° — 120°) = sin 60° = 2 = cos 30° An alternative method would be to read off sin 120° and cos 30° from the graphs for sine and cosine. In both cases there is an answer of 0.8660... (=), although you are unlikely to be able to read a graph to this level of accuracy, Urheberrechtlich ges (Compicte Advanced Level Mathematics: Pure Mathematics. ‘The graph of tan@ dosn't look like cither of the other two graphs. This is because tan @ is defined as ‘The denominator, cos 0, has the value 0 when 0 — 90°. This means that the graph of tan 0 will not be continuous. To show this on the graph we use dotted lines called asymmptotes. Like sin @ and cos 0, tan 0 is periodic, but this time the period is 180” instead of 360°. You should memorise the main features of the sine, cosine and tangent graphs. The main features are the general shape, maximum and minimum. Points, intersections with axes and positions of asymptotes. Trigonometric equations A trigonometric equation is one containing a trigonometric function such as sine, cosine or tangent. Solving these equations means finding values of the angle that satisfy the equation. Usually the range of angles that are acceptable as solutions will be restricted. The restriction could be due to the nature of the problem, or imposed by the person setting the question. To find angles within these ranges the trigonometric graphs can be used. In addition to finding the sine, cosine and tangent of a known angle, a Alternative notation | calculator can be used to find an angle with a particular sine, cosine or inverse trigonometric tangent. The inverse trigonometric functions, written sin“!, cos~* and angles that you may tan-', are used. On a calculator they are usually located above the ‘sin’, ‘come across is aresin ‘cos’ and ‘tan’ function keys. Sometimes an ‘inverse’, shift’, “2nd function’ —_arccos and arctan (ai or ‘are’ key needs to be pressed first. sometimes arsin, are and artan). Ifa calculator is used to find an angle it will give an answer called the principal value. Other solutions can then sometimes be found by adding, multiples of the period for that function. Alternatively, once the principal value is known, the symmetry of the trigonometric graphs can be used to find solutions. ‘3 Trigonometry | Example 5 a Find values of 0 for which 4 sin 0 = 3 such that 0° < 6 < 360°. b Solve the equation tan @ = —2 for 0° < 0 < 360°. Solution a 4sin0=3 = sind=} 4 Recall that + means ‘implies’. O=sin (0.75) = 48.6" (3 5.6.) ‘The solution in the first quadrant is 48.6". ‘The solution in the second quadrant is 180° — 48.6° = 131.4" b tan0=—2 = 0 =1an '(-2) 0 = 63.4" (3 5.6.) Add multiples of 180°, the period of tan, to find solutions that lie within the acceptable range. 63.4° + 180° = 116.6" 63.4" + 360° = 296.6" 63.4° + 540° = 476.6", which is outside the acceptable range. ‘There are two solutions; 6 = 116.6° and 296.6°. (Notice that they are in the second and fourth quadrants, whora tan 0 is negative.) Complete Advanced Level Mathematics: Pure Mathematics The reciprocal ratios ‘There are three other trigonometric ratios, which are known as the ‘These ratios are used reciprocal ratios, These are cosecant (cosec), secant (sec) and less frequently but ar cotangent (cot). applied in astronomy navigation and eq mecha ics, particula eae in projectile motion, Example 6 Find: a cosec30° bd cotao" © secaz” Solution a cosec0? = wduy = 1.3457 (4d-p.) ~ 07431 Example 6 demonstrates the need to wark to an appropriate degree of accuracy when using trigonometric functions on a calculator. Example 7 Find one solution to cosec = 3. Solution 5 cosecl=3 = ane > 1=3sin0 > sind=4 O=sin*(1) = 19.5" (3 s.£) Notice that this solut is the principal value From these examples we see that a useful strategy for solving trigonometric equations is as follows. Step © Rearrange the equation to make sine, cosine or tangont the subject. Step @ Uso a calculator (if necessary) to find tho principal value. Step @ Using a graph or by adding multiples of the period find solutions in the acceptable range of angles. 9 Trigonometry I 3.1 Trigonometric Functions Exercise Technique Write each of the following as trigonometric ratios of positive acute angles: a sin 120° e tan4oo" bcos 165° f cos(—137") © tan 220° 8 sin(-29°) d- cos 305" h sin(-697°) Write down the exact value of the following, leaving answers in terms of surds if appropriate: @ cos 150° e cos 120° b sin 225" f tan420° © tan 300° 8 cos(—300°) sin 330° hh sin(—420°) Solve the following trigonometric equations for 0 < 0 < 360°. Give your answers correct to one decimal place: a sind=0.314 di atand = v2 bcos? = 0.52 e sin? =cos0 c tané=2.561 . f 2sind =3cos0 Find, correct to four significant figures, the value of: a cosee39° e cot200° bb secd1” f cosec307° © cot93° 8 cot420" d= sec129° hh cosec(-15") Find the principal value solutions to the following: a 5sin@=~3 a b tané+3 © secd=4 £ Construct a table giving values of sin, cos @ and tan for appropriate values of in the range —90° < # < 450°. On separato pieces of paper, draw the graphs of sin 0, cos and tan for this range of angles. Use the graphs to solve the equations: a sin@=-05 b cos@=0.8 © tanf=3 97, es MatErial ‘Complete Advanced Level Mathamatics: Pure Mathematics 7 Draw the graph of y = sin@ for values of @ in the range —90° < @ < 90°. ‘Use your graph to find: a sin™'(-0.5) b sin™1(0.71) Contextual ft] ‘The depth of water in a harbour, y metres, can be modelled by the equation y = 5 sin(30t)’ + 12, where t is the time in hours from midnight ‘on a particular day. Draw the graph of this function over a period of 24 hours. b From the graph, find the times of high and low tides. ¢ Use the graph to find the length of time for which the depth of water in the harbour is greater than 15 metres. [2] ‘The height of a tide can be modelled by a function of the form +h =a cosbt° +c, where h is the height in metres of the water and (is the time in hours after midnight. Find the values of a, b and ¢ for the following tide table. High 00:00 12 Low 06:00 2 High 12:00 12 Low 18:00 | 2 [3] The hours of daylight over a period of time can be modelled using a trigonometric equation. If n is the number of hours of daylight and x is the number of days from 1 January, then, n = 12 — 6.cos(x + 10)". a Calculate the length of the daylight on 1 April, which is day 90. b Use the equation to find the dates of the longest and shortest days. ¢ Comment on the reliability of the model. d= Suggest an amendment to the model, | Trigonometry | 3.2 Equations and Identities ‘Trigonometric equations can increase in complexity and solving them: requires combining algebraic techniques and knowledge of the trigonometric functions, Example 1 Solve 4sin@ — 3.cosé = 0 for 0° < 0 < 360°. Solution 4sin@—3cos8=0 + 4sin@=3cos0 = sind 3 cos? 4 = tan0=0.75 0 = tan“! (0.75) = 36.9° ‘Tangent is positive in the first and third quadrants. The solution in the first quadrant is 36.9°, and the solution in the third quadrant is 216.9". 4180 +369 = 216.9. ¥, Example 2 Solve cos (sin 0 = 2sin0, for 0° < 0 < 360° Solution Wh - cosdsind =2sin@ = — cos@sind—2sind=0 remember that you can’t = sin 0(cos0-2)=0 < Factorise. divide throughout by sin 0, because sin 0 may + sin6 =0 or (cos—2)=0 aloe When siné ), 0 = 0°, 180°, 360°. ‘When cos @ = 2 there are no solutions. So the solutions are 0 = 0°, 180°, 360°. Complete Advanced Level Mathematics: Pure Mathematics 100 Example 3 Solve the equation 2sin* 0 + sin@ = 1 =0, for 0” < 0< 360°. ‘The notation sin* @ Sofution means (sin0)*. The equation 2 sin? @ + sin 0 — 1 = 0 is a quadratic in sin 0. It can be solved directly, or by making a substitution for sin (which can make the process look less complex). @ Direct solution 2sin*0+sind-1=0 + (2sin@ = 1)(sin@ +1) =0 = 2sin@-1=0or sind+1=0 Sosin@ = forsin@ © Substitution Let y = sind. 2sin?0+sin0—-1=0 = ayt+y-1=0 = (yv-1v+1)=0 > ty-1=00ry+1=0 > y=h}ory=-1 But y = sin@, so sind = $ or sind = Both methods give the same result. What else do you notice? You should notice that these are the trigonometric ratios for ‘special angles’. When sin@ = 5, @ = 30°, 150° for 0° < @ < 360°, and when sin@ = —1,0 = 270°. So the solutions are @ = 30°, 150°, 270°. Anothor type of trigonometric equation is one involving brackets. Example 4 Solve the equation cos(0 + 30°) = } for ~180" < 0 < 180". Remember that this h Solution ‘many solutions. cos(0 + 30°) =4 = (0+ 30°) = cos”'(4) = 0430" =...,-60°,60°,... = 6 = -90°, 30° in the given range So the solutions are @ = —90°, 30°. Give solutions in the Check that these are solutions by substituting them back into the original required range only. equation, cos(—90" + 30°) = cos(—60") cos(30° +30") = cos60° =! 3 Trigonometry | Example 5 Solve tan 2? = 1, foro <0-< 360°. Solution Tf tan 20 = 1 then 20 must be a special angle. Note also the range of values for 0. The equation is in 20, so we must solve it for 20 in the range 0° <20< 720°, tan20=1 = 20=tan-'(1) 20 = 45°,225° 405°. 585" 0 = 22.5°, 112.5", 202.5", 292.5" Pythagorean identities ‘When equations involve more than one trigonometric function, and factorisation does not appear to be an obvious method, other techniques can be used. Often il is possible to substitute an equivalent expression for one already in the equation. Three results in particular are useful for this. ‘They are known as Pythagorean identities. An identity is an equation thal is true forall values of the variable. It is sometimes distinguished by the symbol = instead of =. ‘The Pythagorean identities are: Beato = Identity 1 [RDS 070 Identity 2 ¢ sec") —tan?@=1 FRGREesec*0 identity 3 cosec* cot? Identity 1 can be demonstrated by using Pythagoras’ theorem, B © Ne a sind =2 cos =8 A b G © 2 ast sin? o+cost0= (2) +(2) 2) *\e _ ato = a Urheberrechtlich geschotzil Are sin? @+cos?0=1 works in all four quadrants, Complete Advanced Level Mathematics: Pure Mathematics ‘The following examples illustrate how these results can be used to simplify trigonometric equations and establish further identities. Example 6 Solve the equations: a 2sin’ 0+ 5cos0+1= 0 for ~180" < 0 < 180° b 3cot” @+ Scosec @ +1 =0 for 0° <0 < 360°. Solution a 2sin?@+5cos@+1=0 = 2(1~cos?0)4+5cos0+1=0 = 2~2c0s'0+5cos0 + =o > 2cos" 0 —5cos0—-3 = 0 ‘This is a quadratic in cosd that can be factorised. (2cos 0 + 1)(cos0— 3) =0 So 2cas0+1=0 or cos—3=0 cos? =—} or cos0=3 cos ==1 > 0=~120°,120° cos = 3 has no solutions So the solutions are @ = £120", b Scot? + 5cosec0+1=0 + B(cosec*#—1)+5cosecd+1=0 = deosec?— 4+ Scosecd+1=0 = Scosec’ # +5 cosec# ~2 =0 > (Beasec0 — 1)(cosec +2) =0 = 3cosec —1=0 or cosecd +2 =0 So cosec @ = } or cosec @ = —2. So now sin 0 = 3 or sin0 = —3, Using the definition of cosec 0, sin = 3 has no solutions and sin0=—} + 0 =210',330° So the solutions are 0 = 210°, 330°. Example 7 Prove tho identity sin 0 tan 0 = sec 0 — cos 0. 102 theberrechtlich geschiitztes Material 3 Trigonometry | Solution sin@tan@ = sind=—— cos sin@ Using the definition of tan 0. sin’ @ 4 Using identity 1. = secl —cos 0 Thatis, sin@tan@ = sec@ — cosé. Example 7 illustrates a useful procedure to adopt when establishing identities. Begin with the left-hand side (LHS) and then perform logical manipulations line by Line until the form of the right-hand side (RHS) appears. At first this may appear very difficult, but keep in mind where you are trying to go. The range of equivalont forms mean that tasks like these are difficult for computers and symbol manipulators to do quickly. Example 8 Prove the identity tan @ + cot 0 = secOcosec0. Solution Boginning with the LHS, _ sind, cos tan 0 +cotd = wand sind a sin’ 0+ cos’ 0 ~ sin O cos 8 a! real = ala 4 Using identity 1. eax t ~ sind * cosd = sec x cosecO And so we arrive at the RHS. Thatis, tan6+coté = sec @ cosecé, Another useful technique that can help in proving identities is the ability. to multiply by 1, with 1 written in a convenient form that allows further algebraic manipulation. sin@_, 14cos0_| 1—tand For example, t= Ty e080 =! T= land = 1,and soon. Urheberrechttich ‘Complete Advanced Level Mathematics: Pure Mathematics 104 Example 9 cos Prove the identity 7— wing = 208 + Han 0. Solution cos0 Beginning with the LHS, pis already written in terms of sine and erry cosine, so we appear to be stuck, However, multiply by 1, and force the denominator to be the difference of two squares. cost __cos0_ 1+ sind 1—sind~ 1—sind 1+sin0 _ cos 61+ sin) ~~ aFsin?@ =e <4 Using identity 1. 1+sind cos 0 1, sind cos" cos = sec 0+ tan So we arrive at the RHS, as required. That is, cos i—sind = sec +tan 0 1 geschit s Material 3 Trigonometry | 3.2 Equations and Identities Exercise Technique =] Solve the following trigonometric equations for 0° < 0 < 360° (giving angles correct to one decimal place): a 3sin@—4cos0=0 d= 4sin0 = cos b sin? +2cos0=0 e cot? =tand © 3sin@+cos0 =0 £ 2sind =} cosect 2 | Solve these equations completely for -180° < 0 < 180°: a tan?#—Stan+2=0 d 4cos*0=3 b 3sin’0—4sind+1 e 8sin’d—Gsind+1=0 “© 2cos*0+cos0-1=0 £ Stanto=1 3 | Solve these equations completely in the range 0° < 0 < 380° (giving answers correct to one decimal place where necessary): a sinz0=} d- tan(3@— 40°) =6 b tan(0- 33°) = 0.4616 e sin(@— 90°) © ¢os(20— 20") = 0.212 £ tan20=1 [4] ‘Use Pythagorean identities to solve these equations in the range =180" < 0< 180°: a Geos’ + sind—5 =0 d= 2sec*=Stan@ basin? 0+ 5cosO+ e@ cosec"# = 3 cotd © sec’@=3-tand f tan?@+sec?@=17 5 | Prove the following identities: sec’ 0 + cosec* = sec* Ocosec’? a b secd+tand= 1+ #ind cos © cos'0+3sin’ 0 =3—2c0s"0 d cos* @—sin* 0 = cos’ 0 ~sin?0 e cot? + cos" = (cosecé — sin 0) (casec@ + sin 8) fp sindtand =1 oc T—cos0 =*** Urheberrechtlich ges Complete Advanced Level Mathernatics: Pure Mathematics Contextual 1 | The trajectory of a golf ball struck from a tee can be modelled by the equation 2 cect 8 ye=xtang -~ Se where 0 is the acute angle of projection, x is the horizontal distance of the ball from the tee in metres and y is the vertical height of the ball above the fairway, also measured in metres. If the ball just clears a tree 16.8 m high, whose base is 16 m from the tee, find, to three significant figures, the angle of projection. [2] The equation of the path of a projectile referred to horizontal and upward vertical axes Ox, Qy for a golf ball is x? sec? 8 tang ~~ Show how to reduce this equation to a quadratic equation in tan. Show that there aro two distinct values of for which the ball can pass through a given point (X,Y), where X > 0, provided 200Y < 2500 ~ 4X*. Interprot this result in terms the golfer would understand. 106 3 Trigonometry | 3.3 Compound and Double Angle Formulas In the trigonometry studied so far all the relationships contain trigonometric functions of a single variable, @. There are other useful relationships that involve trigonamotric functions of two variables and these are known as compound angle formulas. ‘They are also known as the addition theorems since they show how the trigonometric functions of a sum oF difference of two angles can be expressed in terms of the trigonometric functions of the individual angles, HUBBER Hessian antsy team hs formulas. Notice that they are wrilten as identities. This is because they are true for all values of A and B. Identity 1 can be proved for the case and Bacute by considering a diagram illustrating the geometry of the angle sum (A +B), R Q Let OPQ and OQR be right-angled triangles _, containing angles A and B respectively (as triangles OPQ and RSQ. a U p shown in the diagram), Look at triangle OQR. are similar, If OR = 1 then RQ = sin Band OQ = cosB. Now look at triangle RQT. RQ = sinB, so RS = sin BeosA (call this result [1 Now look at triangle OQR. QP = OQsinA, and OQ =cosB, 50 QP=cosBsinA (call this result [2]) (Complete Advanced Level Mathematics: Pure Mathematics ‘Now look at triangle ORU. OR =1, so RU = sin(A +B). But AU = RS + SU = RS + QP, so from results [1] and [2], sin(A+ B) = sin Bcos A + cos Bsin A ‘That is, sin(A+B) = sinAcosB+ cosA sinB. ‘This demonstrates a proof for A and B acute. The proof of cos(A +B) is very similar. The results for the differences, sin(A — B) and cos(A ~ B), can ‘be found by replacing B with —B in each of the proven results and using the results sin(—B) = — sin B and cos(—B) = cos(B). These last results can ‘be seen clearly from the graphs af sine and cosine, yesinx ‘They illustrate the results for negative angles. GEA esl) cosa "These compound angle formulas allow us to evaluate more sines and cosines exactly and demonstrate further identities. Example 1 a Find sin 15° in surd form. b Find the exact value of cos. 75°. Solution a sin15° = sin(60° — 45°) sin 60° cos 45° — cos60°sin45° <4 Using identity 2. -a)0%) ~ va} \2° v2, = W- =a 1) v2 =4a- itis usual to write th expressions with the angles in alphabetica order, p Ih 1 ‘We know we want ar answer in surd form, try to write the angle a sum of special angl Remember to rationa the denominator. 3 Trigonometry | ‘b cos75" = cos(45° + 30°) = ©0845" cos 90" — sin 45° sin 30° =e 7 “( 5°) V2 V2 “This is consistent 1 =aRs-0 v2 =F 03-0 “These compound angle formulas also allow the addition thearems for tangent to be established. tan B PY, ates tan meer =tan B ‘Aton B Identity 6 Example 2 Prove the identity tan(A +B) = poate Solution ‘Boginning with the LHS, _ sin(A +B) tan(A + B) = ‘wos +B) — SinAcosB+ecosAsinB Using identities 1 and 3. © cosAcos B— sinAsinB sinAcosB | cosAsinB cosAcosB " cosAcosB ~ cosAcosB sinAsinB cosAcosB cosAcosB tanA + tan B Eliminate terms; write ~ 7 —tanAtanB tangents where possible, So we arrive at the RHS. The compound angle formula for tan (4 — B) can be derived in the same way. Example 3 Solve the equation sin( + 45°) = 2.cos(0 + 45°) for 0° < 0 < 360%. chatztes #09) Urheberrechtlict (Complete Advanced Level Mathematics: Pure Mathematics: 110 Solution sin(0 +45") => tan(0+45") tan @ + tan 45° So 3tan@ = 1, and tan ‘The solutions are ( = 18. "198.4" (1d.p.). Double angles “The compound angle formulas provide some mare very usoful results in the special case where A ~ B, These are known as double angle formulas. HEBER es tera, BE. ‘To get these results simply put A = B = 0 in the formulas for sin (A +8), cos (A + B) and tan(A +B), We can also use the Pythagorean identity, sin? 0 + cos? 0 = 1, in the expression for cas 20 to provide two other versions. WEEE) 2 tne x O74") = tan 135" = tan(180° — 45°) = —tan45° =-1 Example 6 Prove the identity tan 0 +cot@ = Zcosec20. Solution Beginning with the LHS, sind, cos tand + col = a+ and ca 2 = HEO+ Sg adding fractions. cosOsin 0 1 ~ gin 06080 2 Multiply numerator ~ BsinDcosd and denominator sn {Remember the definition of cosec 0. Peeere = 2ensec 20 Complete Advanced Level Mathematics: Pure Mathematics 3.3 Compound and Double Angle Formulas Exercise Technique ihout using a calculator, find the exact value of the following: Hint: use compound a sin75° bcos 105° © tan 15° mgt cen ds cosi5° e sin 105° f tan 105° 2 | write down the exact value of the following exprossions: @ cos 40° cos 50" —sin40°sin50° d_— sin 40’ cos20° + cos 40° sin 20" b sin60' cos 15°—cos60°sin15° @ —cos85° cos25° + sin85° sin25° tan 47° = tan 17” g tan90" + tan 30° 1+ tan47 tan 17 1=1ang0 tan 30" 3 | Solve the following equations for 0° < 0 < 360°, giving angles correct to ‘one decimal place: a sin(0 +30" © cost ~ 60" =2e080 bh sin(0 +15") = 3cos(0~ 15°) jsind d_sin(0 +45") = —2c0s0 4 | Prove the following identities: a cos(45° — 0) ~cos(45° +0) = V2sine b tan(@ + 45°) + tan(0— 45° tan 20 © sin(A 4B) + sin(A—B) =2sinAcosB _ sin(A+ B) ds tana +tanB = AwsB 5 | sina = 4, where Ais acute, and cosB = %, find the exact value of the following: a sin(A+B) © tan(A+8) i tan2a b cos(A+B) f tan(A-8) j sin2B © sin(A~B) g osin2A k cos 2B d cos (A—B) bh cos2a 1 tan2e G6 | Prove the following identities: a secHcosecd = 2cosec 20 b 500204 ton2g = 2+ sind cos — sind sin2d =coto sin 20 and cos 20 1+ c0s20 112 3 Trigonometry | Contextual ramp for a wheelchair, as illustrated in the diagram, can be moved by a person lifting on a handle at B and wheeling the ramp ona castor attached at A. The dimensions of the ramp are AB = 3m and BC inclined at an angle 0 to the ground as shown, = 50 cm. The base of the ramp is Ifthe why sin(@ + p is to be moved through a doorway 2m high, explain 5°) < 0.6976. ii Solve this equation for @ iii Comment on the practicaliti b Asecond ramp can similarly be moved by a person lifting on a handle at B. The ramp can be wheeled on a this ramp are AB = 2,5 mand BC i Write down the new equation for @. of @ achieving ils maximum value, jor at A. The dimensions of 60cm ii Solve the equation. iii Compare the heights to which B has to be lifted, 113, Complete Advanced Level Mathematics: Pure Mathematics 114 3.4 The Cosine Rule and the Sine Rule “Solve a triangle’ means find all the angles and the lengths of all the sides of the triangle. Given three of the six quantities (angles or sides) can the remaining three be found? Pythagoras’ theorem applies to right-angled triangles. How do we deal with non-rightangled triangles? One technique is to use the cosine rule, The cosine rule For any triangle ABC, HEBPWE 2heona To demonstrate this result, drop a perpendicular from C to meet AB at D. Now look at triangle ACD. G c A € ss Using Pythagoras’ theorem, Baht +xt So ht =bi—xt it) Now look at triangle BCD. Using the same technique, a? =h? + (o-x)* ht =a" ~(e—x)? lel Eliminating h by equating [1] and [2]. bP =27 =a" -(c- xf So bY =x* =a" — (¢ ~ 2ex 4x) ‘Multiplying out the ae ee bracket. Baa —C +2cx ‘The x7 terms cancel Pur+?—ux each other out. 8 Trigonometry | But from triangle ACD, cosA =f, givingx = beos A. So now a? = b* +c — abe cos A. By using a symmetrical argument, the two other equivalent versions can be found (that is, by dropping the perpendicular from any angle, or simply by relabelling the triangle), BBE 20b cose So the cosine rule can be used to find the length of the third side, when the lengths of two sides in a triangle and the angle between them are known, Example 1 In triangle ABC, b = 10cm, S G=68cmandA = 50°. Finda, 10 Solution Using a? =b* +c? - 2bccosA, 2 = 10? + 8? 2x 10 x8 x cos 50° = 100 + 64 ~ 160c0s 50° 64 — 102.85 =6115 Soa = V61.15 = 7.82 om (3 sf.) Example 2 In triangle ABC, a = 12cm, ¢ = 17 em and B = 129°. Find b. CG 12 115 eberrechilich geschitztA SPaterial ‘Complete Advanced Level Mathematics: Pure Mathematics: Solution Although # is obtuse, the rule still applies. Using b’ =a’ +c" — 2accosB, bP = 12? +477 —2 «12 x 17 x cos 129° = 144 + 289 + 256.76 = 689.76 So b= V689.76 = 26,3em (3 sf.) ‘The throe cosine rule formulas can be rearranged to give expressions for cosA, cos B and cos Cin terms of a, band c. ‘This means that if the lengths of each of the three sides are known then each of the three angles can be found. Example 3 In triangle ABC ifa = 12cm, b = 20em and c= 14cm, find A, B and C. B 4 2 A 20 c Solution 2 2 Usingeasa = = i= 20% 4147 — 127 SO = 20 x 1d 400+ 196-144 452 = ag 8072 So A= cos*(0.8071) A= 36.2" (1 dp.) 116 Utheberrechilich geschiitztes Material 3 Trigonometry 1 P+ ob sing cosB =", Ualng Dae - coop = 12+ 14-207 2x12« 14 144 + 196 — 400 60 gas 6 = 1788 So B= cos '{—-0.1786) B=100.3° (1. dp.) We know that the angles in a triangle add up to 180°, So now C= 180° = (A+B) = 180° — (36.2° + 100.3") = 180° - 136.5" =43.5° (1 dp) The sine rule ‘The sine rule is another useful result to help solve triangles. For any triangle ABC, Eee a This particular version of the sine rule is useful for finding sides. To find angles the more convenient form to use is ‘To demonstrate this result, drop a perpendicular from C to meet AB at D, c c a @ BOA D B Looking at triangle ACD, } = sina, Soh=bsinA Urheberrechttic! Complete Advanced Level Mathematics: Pure Mathematics 118 Looking at triangle BCD, * = sin B. Soh=asinB Now eliminate A by equating these two results. Thon asin B = bsina. Use the geometry of the figure or simply relabel to find the equivalent eet ratio ain ‘The sine rule is useful in the following situations: @ two (and henee three) angles and one side are known; @ two sides and one angle (but not the angle between them) are known; © one angle and its opposite side, and one other piece of information are known. Example 4 In triangle ABC, A = 35°, a = 17 cmand B = 50°. Find b. Cc s 17cm A B _ 17 xsin50° ~ sin 35" _ 17. «0,760 ~ 0.5736 b=22.7cem(1 dp) 3 Trigonometry 1 Example 5 In triangle ABC, A = 60°,a = 6cm. c and b = Sem. Find B. So B= sin '(0.7217) = 46.2° Exercise some caution when using the sine rule to find angles. Sometimes an ambiguity may arise, where two possible angles will solve the trigonometric equation involving sine. The calculator will always give the acute angle but on occasions the obtuse angle gives an equally valid, if different, solution. Example 6 Salve the triangle for which A = 30°, c Oem B Solution Using the sine rule to find G, sinC _ sina So C= 38.7°,141.3° (1 dip.) Notice that the abtuse angle (141.3°) is possible here. (Complete Advanced Level Mathematics: Pure Mathematics ‘Angle B can now be found by using the angle sum of a triangle, When € = 38.7", B= 180° — (30° + 38.7") = 111.9, When G = 141.3°, B= 180° ~ (30° + 141.3") = 87°. ‘Now use the cosine rule (or the sine rule again) to find b for each of the possibilities for B. ‘Using the cosine rule, when B= 111.3", bY =a* +c —2accosB =8* +10? 2% 8 10 ¥ cos 111.3" = 1644 58.12 = 222.12 So b=14.9cm(1 dy ‘Using the cosine rule, when B= 8.7", bt =a? +? —2accosB = 8+ 107-2 x 8x 10 x cos 6.7" =164— 158.16 = 5.84 So b=24em(1d.p.) ‘This means the triangle ABC has two distinct solutions, A=30',a=8em A= 30°, a =8cm B=1113'.b=149cem B=8.7", b= 24cm C= 38.7", ¢= 100m C= 1413", ¢= 10cm One sketch can be used to illustrate the geometrical interpretation. 120 Urheberrechilich geschitztes Material 3 Trigonometry | 3.4 The Cosine Rule and the Sine Rule Exercise Technique [11] Use the cosine rule to find the length of bin the following triangles: a B=70',a=#cm,c= 12cm b B=120°,a=11cm,c= 15cm c B=48',a=14.2cm,c=85em 2 | Use the cosine rule to find the angles in the fallowing triangles: a a=Gem,b=4em,¢=5cm b a= 13cm, b= 12cm,c=19em © @=7cm,b=6em,c= 14cm. 3 | Calculate the length of the lettered side in the following triangles: 4 | Solve the following triangles: a oc b c 5.5.cm 85cm) em 8 A 45em A 12300 B e ¢ d c ates 100m 25m ms x B 23 em ‘Complete Advanced Level Mathematics: Pure Mathematics. Contextual 11 | A ship sails 4 nautical miles from A to B on a bearing of 075°. It then changes to a bearing of 150° and sails 6 nautical miles to C. Calculate the distance AC and the bearing of C from A. [2] ship « is 12 mites from lighthouse B on a bearing of 050°. A second ship C is 15 miles from the same lighthouse on a bearing of 330°, Find the distance between A and Cand the bearing of G from A. [3] 4.8 and C are three triangulation points used on an Ordnance Survey map. B is 5 km due south of A. G is 4 km from B on a bearing of 300°. Find the distance from A to C and the bearing of C from A. [4] From a point 4 on the same level as the base of a radio mast, the angle of elevation of the top of the mast is 25°. From a point B, 20 metres closer to the mast, and on the game lave, the angle of elevation ie 92°, Find the helght af the radio mast. 122 Urheberrechtlich geschiitztes Material ‘3 Trigonometry | 3.5 The Area of a Triangle ‘You should already be familiar with the rule for the area of a triangle that reads: area =} x base x perpendicular height ‘This formula is only useful if the length of the ‘base’ and the ‘perpendicular height’ of the triangle are known, When this is not the ease other techniques are required. ‘A more useful result is one that incorporates lengths of sides and sizes of angles, In general the area of a triangle ABG is given by: HEREIEENISAC=foesn? oor ime eas Remember the symmetry of this expression, ‘To demonstrate these results drop a G perpendicular from A, B or Gto meet a, bor c respectively. Suppose a ‘ a perpendicular that meets AB at Dis drawn, as shown, A D B Looking at triangle ACD, f! = sind. area of triangle ACB = } x base x perpendicular height adxexh =}xexbxsinA = Yoesina ‘To show one of the other symmetrical versions, simply drop perpendicular lines onto BC or AC. Example 1 Find the areas of the following triangles: a c b c. osem 7m A 112em a A 12m B 123, Complete Advanced Level Mathematics: Pure Mathematics 124 Solution a Using area = }acsin B, area =} x 9.4 x 11.2 x sin 56° =43.6cm’ (3 sf) b Using area = ibesina, area = } <7 x 12 sin 110° = 39.5m" (3 sf) Hero’s formula ‘These formulas for area are not really useful if only the lengths of the sidos of the triangle are known. The cosine rule could be used to find an angle but often it is much more efficient to use another method called Hero's formula. This is named after Heron of Alexandria (who lived about 758c). Hero's formula has many practical applications. For example surveyors who know lengths of sides of a three-sided lot can easily compute the area. Hero's formula states that, for any triangle ABC with sides of length a, b and c: : =tthte 4 Learn this result. Example 2 Which of the following triangles has the larger area? 1 2 Cc Cc 25m 25m 25m 25m A 40m B A 30m —B Sofution Looking al triangle 4, 8 =}(25 425 +40) = 45 ‘mathematicians were forced to consider the | ef eaeulstion by prob neat rg -of astronomy an techanics This i an example of where the need to solve prablems forced the development ‘ew techniques in mathematics. 3 Trigonometry 1 So the area of triangle 1 = //45(45 — 25)(45 — 25)(45 — 40) = V45 «20 x 20x 5 = ¥90000 = 300 m* Looking at triangle 2, s=} (25425430) =40 So tho area of triangle 2 = //40(40 — 25)(40 — 25)(40 — 30) = Vi0% 15 x15 x 10 = ¥90000 = 300m" Since the area of both triangles 1 and 2 is 300 m’, neither has a larger area. Urheberrechtich geschatztel Merial (Complete Advanced Level Mathematics: Pure Mathematics 3.5 The Area of a Triangle Exercise Technique [11] Fora sight-angled triangle with sides 2m, 4m and 5m find the area using: area =} (base » perpendicular height) b area = tab sinc Hero's formula. [2] Find the areas of the following triangles: Pe wan cae! Nwen rae tim em B Contextual 1 | The area of an acute-angled triangle ABC is 12.cm?. Given that a=b=5em, find c. [2] A surveyor wishes to ostimate the area ofa triangular patch af woodland. Askotch is made showing the following measurements. Estimate the area of the woodland. [3] Pouta, the mathematician, wants to share hor square birthday cake equally amongst the seven people at her party. She wants everyone to have the same amount of cake and icing. Her friend, Kevin, suggests dividing the perimeter by 7 and cutting from the centre to the appropriate points on the perimeter. Ifthe cake measures 7 inches down each side and is 3 inches high will Kevin's suggestion work? 126 Urheberrechilich geschitztes Material 3 Trigonometry | 3.6 Radian Measure All methods of measuring angles are based on division of the circle, We have previously measured angles in degrees, where 360° is a full circle. So a degree is simply =i; of a circle. Another way of measuring angles is to compare the length of an arc formed by the angle with the radius of the circle, The unit used in this method is called the radian. In the diagram the radius of the circle is r ¢ and the length of the arc PQ is also r. ‘The angled is said to be 1 radian. Lf are length 1 radius r ‘The circumference of the circle is 2xr. This means that a complete circle, defined in degrees as 360°, is 2x in radians, So 360° = 2 radians (rad). ‘Then 180° = x rad, so Similarly, a radian has an equivatence in degrees. We have 2 rad = 360°, so Using a calculator to evaluate this statement, Arad = 57.20578...° = 57.9 (3 sf) Since this isn't a rational value (because of the involvement of x, which is irrational) it is more usual to express radians as fractions of x, Example 1 Express the special angles 30°, 45°, 60° and 90° as radians, Urheberrechilich ges chttzepgipaterial Complete Advanced Level Mathematics: Pure Mathematics: Solution en Since 1° = +55 rad, Pe Se) yseox =" 30° = 30% 7 =F rad 60" = 60 x7 =F rad Learn these results, ses - o% Yoox 2 45° = 45 x Fg gtd 90" 90% 55 giad Example 2 Convert 3 rad to degrees, Solution 360° br 5x 360° Ind =] = Grd = SxS = 150° Radian measure has several advantages in mathematics and will be used. later in curve sketching and integration. This measure allows both axes to ‘be scaled in real numbers and is particularly advantageous when the requirement is to solve a problem by reflecting in the line y = x, or finding the intersection of graphs. Another application of radians is in finding Q arc lengths and sector areas. If we take a sector of a circle its are length and area will be a fraction of the whole circle. ‘The formulas for these have a simpler form when the sector P angle is measured in radians. When 0 is in radians, RRA 70 Learn those results. Example 3 Find tho are length and area of a soctor of radius 4cm and angle 1.5 rad. Solution When 0 is measured in radians, arc length 5 =r = 41.5 =6cm oem sector area A= 470 =1x 4? x 1.5 = 12cm? 3 Trigonometry | Example 4 A piece of wire 40.cm long is bent into the shape of a sector. If the area of this sector is 10cm? find: a anexpression for the sector angle 0 in terms of radius r b the value ofr c the value of @ Solution a Think carefully about the given information, The three lengths (both radii and the arc \e/ Tength) add up to 40 cm. So arc length $= 40-21. Also, s = r0. = = ‘b Now use the information about the area, A=}e0 Substituting for @ from a, ton =} x (2=2) r 100 3% (40-20) 100 = 5 x 2(20— 1) 100 = 207 —r* So P-20r+100=0 + (r-10)(r—10)=0 (r-10/F =0 ‘So r= 10. That is, the radius is 10cm. 10 — (2 x 10) 10 So the sector angle is 2 rad. Utheberrechtlich geschitzt 2 areria (Complete Advanced Level Mathematics: Pura Mathematics 3.6 Radian Measure Exercise Technique [1] Without using a calculator convert the following angles into radians, leaving the answer in terms of x: a 15° eos e 225° b 150° d 180° f 315° [2] ‘Convert the following to radians, writing your answers correct to three significant figures: a 20° «© 129° e 269° bo 78° d= 222" f asi" [33] without using a calculatar convert the following angles to degrees: aon © Sn et dF qi [F [4] Find the arciengths and sector areas of the following sectors, correct ta ‘one decimal place: a 0=11adr=5cm bo 0=1.5rad,r= 66m © 0=2radr= 110m d 0=2.6rad,r=9cm [5] A sector OPQ has arc length 20 cm and sector area 100 cm® (as shown). Write down the two equations involving rand 6 (the radius and sector angle}. Solve these equations to find the value of rand 0. O<@) awa=100cm? | 200m | Given that the area of sector OPQ is 50.cm® and that 0 = 1.2 rad, find the radius of the circle. T | Now go back to Exercise 3.2 on page 105 and do questions 1, 2, and 4 Bo using radians. 130 chitztes ‘3 Trigonometry | Contextual A wadge of cheese, 0.5 em thick, has a cross-section that is a sector of a circle. If the radius of the circle is Sem and the arc length is 4 cm, find the volume of the cheese. An oil drum of diameter 80 cm is floating, as shown. water surface If the are length PQ is 90 cm, find 0 (in radians). Find the area of the minor sector OPQ. Use the cosine rule to work out the length of the chord PQ. Find the area of the triangle OPQ. If the drum is 1.2m long, find the volume af the drum Lying below the water surface level, correct to three significant figures, eaore A circular cone with base radius rand slant height fis unrolled into a circular sector. Find the angle of this sector in radians. Use your answer to explain why the curved surface area of the cone is rl, A windscreen wiper clears a region ABCD of a car windscreen (assumed to be flat), where AB and DG are circular ares centred at O. Given that OD = 20cm, DA = 50cm and /AOB = 2.3rad, calculate the area of region ABCD. (NEAB) Urheberrechtli Complete Advanced Level Mathematics: Pura Mathematics. Consolidation Exercise A 1] Find the four solutions of the equation sin 20 = cos’ @ in the interval ‘0° < @< 360°. Give each of these solutions correct to the nearest degree. (NEAB) [2] the diagram shows a triangle ABC in which angle C = 30°, BC = xem and AC = (x +2) em, Given that the area of triangle ABC is 12.cm’, calculate the value of x, B A (e+ 2) em c (UCLES) Figure 1 shows the points A and B which two ships have reached after leaving a harbour H. The points A and B are at distance of 8km and 11 km. from H respectively. The bearings of A and B from H are 048° and 120° -rospectively. Caleulate: a the distance between A and B, giving your answer to 0.1 km; b the bearing of B and A, giving your answor to the nearest degree. N 8 km i Figure 1 {ULEAC) [4] Solve the equation 4 cot? @ + 12. cosec@ + 1 = 0, giving all values of 0 to the nearest degree in the interval 0° <0 < 360°, (AEB) [5] Find, in degrees, the solution of the equation tan(2x + 30°) = — v3, for 0 45°. b If sin4y is negative and cos 4y is positive, what value must y exceed? ¢ Ifthe conditions ina and b remain true, state, with reasons, whether the following are positive or negati i tanx+y) ii tangy. oD a Tao tin Forrmila tame nd = 28MO— 1A 8, toy that tainted) ia 1=3tan*@ root of the cubic equation x’ — 3x” — 3x +1=0. b Solve the equation sin 2y = 2— 2cos2y (0 Juventus Turin —* Real Madrid “Sporting Lisbon Gities » European foatball clubs Two other types of mapping exist: @ one-to-one mappings @ many-to-many mappings. 140 theberrect hit ‘al 4 Functions: Amsterdam - Germany Liston << F taly Madrid European competitions: ‘The relationship between the seven foatball clubs and the major European. club competitions is an example of a many-to-many mapping. Several of these clubs have won more than one of these competitions. Functions ‘Any mathematical mapping that takes any object in the domain and maps it to one, and only one, element in the co-domain is called a function, Since there can only be one possible image for each object in the domain, only many-to-one and one-to-one mappings are functions. Example 1 Which of the following mappings are functions? € i ¢ () & Solution a This one-to-one mapping is a function. Each object in the domain maps to one, and only one, image. echilich geschatztel Asie Complete Advanced Level Mathematics: Pure Mathematics 142 b This many-to-one mapping is also a function. Note that it is not necessary for every element in the co-domain to be an image of an object in the domain. ¢ This many-to-many mapping is nol a function because the element ‘a’ in the domain maps to more than one element in the co-damain. ‘The domain of a function is the set of objects or input values for which the function is defined. In Example 1 the set {a,b,c} is the domain in functions a and b. The ca-domain is a set containing the possible output values of a function. In Example 1 the set {x,y,z} is the co-domain in functions a and b. ‘The range of a function is the sot of images, or output values, to which the objects in the domain map. This can be the entire co-domain, as in the function in Example 1a, which has the range (x,y,z), or it can bea subset, of the co-domain, as in the function in Example 1b, which has the range {x2}. The words onto and into are used to highlight the difference. The domain of the function in Example 1a maps onto its co-domain, because it uses every element in the co-domain. The domain of the function in Example 1b maps into its co-domain, because only some of the elements are used, A number of different but equivalent notations are used to define a function, For example, the function f, which maps values of x in the domain to values of 2x in the co-domain, may be written fix 2x <4 f maps.x to 2x. or f(x) = 2x or y=2x In this last form of notation, objects x in the domain map to images y in the co-domain, These (x,y) pairs can be used to represent the function graphically, Example 2 Functions f and g are defined for domain {~-2, ~1,0,1,2) by fox = 2x + 1and g: x — x7, Draw a mapping diagram for each function. Stato the range of each function and explain which type of mapping is shown, fmapsxto 2x41 g maps x tox 4 Functions fixate +1 gece ‘The range of function f, for the given domain, is {—3,—1,1,3,5}. of: — 2x +1is a one-to-one mapping. ‘The range of function g, for the given domain, is {0,1, 4}. g:x =x" is a many-to-one mapping. has been possible to draw mapping diagrams for each function in Example 2 because the domain is discrete. This means the domain. consists of separate values of x, and does not contain any values between ‘those stated. If the domain isn’t discrete then it can contain any value within specified limits. In this case the domain is said to be continuous. Functions with continuous domains are usually represented graphically. Example 3 ‘The functions f and gin Example 2 are defined for the continuous domain {x €R: ~2 ax +b, where a and bare constants. If a and bare both unknown, then they can be calculated using the value of the function at two different values of x. Example 4 ‘The linear function f:x ax +6 is such that f(1) = 8 and f(3) = 14. Find tho values of a and b. Solution F(1) and (a) are found by substituting x = 1 and x = 3 into the function. fa)=6 + atb=s fal f(a)=14 > 304¢b=14 fal Eliminating b by subtracting equation [1] from equation [2], 2a =6 + a=3 Substituting @ = 3 into equation [1], 3+b=68 > b=s Soa =3,b =5 and the function is fx — 3x +5, All quadratic functions can be written in the general form. fix — ax? + bx +c, where a # 0. If the coefficients a, b and care all unknown, they can be found ina similar way to that used for linear functions (Example 4). This time use the value of the function f at three different values of x. This creates three linear simultaneous equations involving a, b and , These can then be solved using either the elimination or substitution methods. 144 rheberrechtlich geschiitztes Material 4 Funetions 4.1 Mappings and Functions Exercise Technique 1 | The functions below are defined for the given discrete domains. In each case: i draw a mapping diagram fi state the range of the function fii state the type of mapping, fix + 1 = 2x for {x: — 10,-5, 0,5, 10} f:x— 2x7 for (x: —2,-1,0,1,2} fix +R for (x: 0,1,4,9, 16) fix b for (x: 1,2,3,4.5} Roge 2 | The functions below are defined for the given continuous domains. In each case: i sketch the graph of the function ii state the range of the function : state the type of mapping. a fexsixt3 for {veR: 24 ht for x >2 Explain why g is a function, but is not a function. [BJ che functions f and g are such that f(x) = 13x~ 4 and g(x) = 4x? —1. a Find the value of a for which f(a) = 35. bh Find tho values of b for which g(b) ~ 15. © Find the values of c for which f(c) — g(c). [6] ‘The linear function f is defined by f: x — ax +b, where a and b are constants. Given that f(2) = 4 and f(-3) = 13, find a and b, Hence, calculate the value of c for which f(c) = 0. [77] sthe quadratic function g is defined by g:2¢—+ ax? +be +6, wheraa, band are constants. Given that g(0) = —3, g(1) = 4 and g(2) = 15, finda, band ce. Hence calculate the values of d for which g(d) = 0. 146 Utheberrectitlich geschtitztes Material 4Functions 4.2 Inverse Functions For many mathematical mappings, there is a corresponding mapping that has the opposite or reversing effect. For example, the mapping “subtract 6', or x — x— 6 is the opposite of the mapping ‘add 6’, or x + x +6. For the domain {0,1,2,3}, this last mapping has the range {6, 7,8, 9}. 7 Ge XOx+6 xox=6 ‘Taking {6,7.8,9) as the domain for the reverse mapping, x — x — 6, this maps back onto the original domain, So for this particular domain, the mapping x — x + 6 and its inverse x —+ x — Gare both one-to-one relationships. They are therefore both functions. To write this more formally, if the function f is defined such that f:x—+ x46, then its inverse function f-" is defined as f-": x + x ~ 6. Now consider the function f(x) =x*, defined for the domain {x € Z: —2 oxy-ysBx41 = y(v—1)=3x41 Beth vet di 3x+1 1y) = So g (x)= ‘Try putting x = 1 into the inverse function, What happens? Division by zero causes a problem, creating an answer that is undefined. So this inverse function is dofined for all roal values of x except x = Example 5 ‘The quadratic function f(x) = {xeRix 24}. ‘8x + 14 is defined for the domain a By ‘completing the square’, express f in the form (x + a)* +b, where a and b are integers. bb Hence, or otherwise, find an expression for the inverse function f-. & Sketch the graph of y = f~"(x), and state the domain and range of this inverse function. d_ Solve the equation f(x) =f~'tx). Mal Complete Advanced Level Mathematics: Pure Mathematics ‘Completing the squat and spotting that 16 = 4%, bb Writing the function in the form y = (x — 4)? —2, and interchanging xandy, 4) -2 Rearranging to make y the subject, (y-4fax+2 “+ yrda vere > yaatvrt2 ‘Therefore, the inverse function is f-'(x) = 4+ Vx+ 2. Reflecting the graph of y = f(x) y ovate) T™* in the line y = x gives the graph y= Mb) afy =f (x). ‘The domain of f~! is {x € R: x > —2}. ‘The range of f-' is {y € B: y > 4}. d= Equating f(x) =x? — 8x +14 with f-'{x) =4 + VN¥2 gives x Bx4 14544 Ve FD Although progress can be made with this equation, it will involve cubic (x*) and quartic (x) terms, Look at the graphs, Notice that the point at which f(x) = -"(x) is also the point of intersection of the line y = x and the curve y = f(x), so solving the simultaneous equations y= x and y = f(x) may be an easier way to find the solution. jh x Bx p14 =x'-ex+14=0 4 Collecting like terms, enero = (x-2)(x-—7)=0 4 Factorising. x=2orx=7 Since function f is only defined for x > 4, ignore the solution x = 2. ‘Therefore, the only solution to the equation f(x) = f~'(x) is x= 7. Sometimes it may be easier ta solve the equation f(x) = f-'(x) by finding the intersection of y = x and y = f~'(x). This is the same as solving. xa fi") 154 rechtlich geschitztes Material 4 Funetions Inverse trigonometric functions ‘The sine, cosine and tangent functions are periodic. For the domain of real numbers they are many-to-one mappings, as illustrated. ve restricted domain where sin x48 onesto- sin.x = 0.5 forx = -330°, ~210°, 30°, 150°, and so on. va restricted domain where 60s is one-to-one 0.5 for x = ~240", ~120°, 120°, 240°, and so on. tanx = 1 forx = ~315", -135°, 45°, 225°, and soon, Urheberrechtlich geschatzted SB ‘Complete Advanced Level Mathematics: Pure Mathematics 156 ‘The inverse trigonometric functions sin", cos’ and tan”* can each be Another notation is defined for a restricted domain. These domains are the values of x for arcsin, arccos and which the sine, cosine and tangent mappings are one-to-one. The restrictions are different for each of the three trigonometric functions. For f(x) = sinx and f(x) = tanx to be one-to-one mappings, =90" 0 [3] the rational functions f, g and h are defined on the set of real numbers such that: = _ _2x+3 : fo= FG a=, toy SE I a State the values of x for which each of these functions are undefined. b Find an expression for each of the inverse functions, and state their domains, Urheberrechilich geschitztes @Siviial Complete Advanced Level Mathematics: Pure Mathematics, [4] the functions f and g are defined such that ax+2 By-1 Jo) =F. gtad= where a and b are constants. a Find expressions for f-"(x) and g”*(x). b State the values of a and b for which functions f and g are solf- inverses. [Bi] 2 express x? — 2x — 4 in the form (x +0)* + b, where a and b ara intogers. b Function f(x) =x? — 2x — 4 is defined for the domain x € R:x > q. Find the least value of q for which function f is a one-to-one mapping. © State the range of f. Find an expression for the inverse function f~!. Solve the equation f(x) = fx). [6] ‘the function f is defined for the domain 0 < x < {by f(x) ~ cos 3x. a Find anexpression for f-', State the domain of f-', ‘b Calculate the value of f~ (-8) in terms of r. ‘The diagram shows the graph of y = f(x), where f(x) = sin 2x. 4 State the restricted domain for which function f isa one-to-one mapping. ‘b For this restricted domain; i find an expression for f* ii state the domain of f-" iii sketch the graphs of y = f(x) and y =~"(x) on one set of axes, using the same scale on both axes, ¢ State the domain of the function g(x) = sinax, where a is a positive constant, if the inverse function g~" exists. 4 Functions. 4.3 Composite Functions The output values of two or more functions can be combined using addition, subtraction, multiplication and division. If f(x) = 3x and g(x) = sinx, possible combinations include: f(x) + g(x) = 3x +sinx A(x) — g(x) = 3x —sinx fx) x g(x) = 4x sinx ax F(x) + a(x) = Foe (provided sinx # 0) Itis also possible to take the output values from one function and use them as the input values for another function. Using f(x) = 3x and g(x) = sinx, first evaluate function g for different values of x, and then input the values of g(x) into function f. What happens? x2. g(x) = sine 1. fig(x)) = 3 sinx The resulting composite function (or “function of a function’) is f(g(x)) = 3sinx, and is commonly referred to as fg(x). Note that the g is written closest to the x because function g is the first stage of this composite function, Now evaluate function f first and use the corresponding values of f(x) as inputs into function g, What happens? x fie) = ax 4 a( fe)) = sin 3x Notice that the composite function isn't the same. This time the output is f(x) = sin3x, Example 1 The functions f, g and hare defined by f:x — cosx, g:x — x*, h: x — 2x + 1. Find expressions for the following composite functions: a gf(x) b hg(x) ce gh(x) do Afx) © haf(x) E fg(x) Solution a af(x) = g(f(x)) Input cosx into 4 function g. = (cosx ( a Recall the conventional = cos? way of writing (cos x)*. Complete Advanced Level Mathematics: Pure Mathematics b hg(x) = Age) =h(x?) x* replaces x in func = 2x7 )41 A =o +1 © gh(x) = g(h(x)) = g(2x +1) = (2x+1)? dbf) =A( f(x) = h(cosx) = 2(eosx) +1 =2eosx+1 © haf(x) = hig f(x))) = h(g(eosx)) = h(cos*x) = 2(cos* x) +1 = 2c0s*x+1 £ fhg(x) =f(hgex)) =S(2x? +1) = cos(2x* + 1) Example 2 Each of the following functions are composite functions of the form gf(x). Ineach case find the camponent functions f and g. a pa) = ste b q(x) =8x° © x) =5-x Solution Use flow diagrams to decompose each of these functions. ce a foot 1 a plej:x x2 ‘Therefore p(x) = gf(x), where f(x) =x +2 and g(x) =}. Bb g(x): x Set a? ‘Therefore q(x) = gf(x), where f(x) =x" and g(x) = 8x. ‘There is an alternative way of expressing function q as the composite of two functions. glx): x 22 2 SM, ax? ‘Then q(x) = gf(x), where f(x) = 2x and g(x) =x°. 160 theberrechtlich geschiitztes Material 4 Functions. co r(xjx 2), x 4, 45 25x ‘Therefore 1(x) = gf(x), where f(x) = —x and g(x) =x +5. Again, there is an alternative, xi), rx)ix 2 p52, (x5) =5-x ‘Then r(x) = gf(w), where f(x) = x — 5 and g(x) x. Instead of feeding the output from one particular function into a different function, we could input back into the original function itself, Using f(x) = 3x again, the composite function Sle) =F (®)) = f(3x) = 9x Note that {f(r) is often referred to as f?(x), indicating that the function is to be carried out twice (and not that the output from the function is to be squared). Similarly, for this particular function, P(x) = AFG) = SEU) = (38x) = 3(9x) = 27% Example 3 ‘The functions f, gand h are defined by fx)=2-x, gx) = (x#-1), and h(x) = 2x-1 3 x41 a Show that f2(x) =x. b Find an expression for g*(x), and state for which two values of x itis undefined. c Solve the equation A(x) = Solution a fw) =f(fls)) (2=x) =2-(2-x)= bgt) = gle) 3 ==) Ei x41 se 3(x+1) of the fraction in the +a" eta denominator. aa Sine g(x) is undefined for x = —1, it is not possible to evaluate g* (x) when x = —1. You can see that g*(x) is also undefined for x = —4. Urheberrectich geschatztel VAreria Complete Advanced Level Mathematics: Pure Mathematics © h(x) = (h(x) (2x — 1) = 2(av-1)-1 =4x-3 Now h¥(x) = h(A?(x)) = h(ax — 3) =2(4x-3)-1 =8x-7 So when h(x) =x, ax-7=x > x=7 = ees Domain of a composite function ‘The relationships botween the domains and ranges of functions f and g, and the range of composite function gf are illustrated here. Hf 8 his go range off = domain of g ee. oe How is the domain of a composite function found? First, establish which values of x in the domain of function f produce values of f(x) that lie in the domain of function g. These can then be fed into g(x). However, depending upon the nature of functions f and g, and the values of x in the domains for which they are defined, the composite function gf may have a restricted domain. Example 4 ‘The functions f and g are defined by f(x) = 1 + 2x for {x € Rix > 0} g(x) = +5 for (x Rx > 4} State the range of f and g for the given domains. Find an expression for gf(x), and determine its domain. © Find an expression for f(x), and determine its domain. r 162 errechtlich geschiitztes Material 4 Functions: Solution a For the given domain, the least value of f(x) is 1. This is when x =0. ‘Therefore the range of f(x) is {y €R:y > 1}. What about g(x)? As x increases, x — 1 also increases. This means that as x increases 1; decreases. As x becomes very large, g(x) will tend towards zero. Conversely, for values of x close to 1, the denominator is very small and g(x) becomes very large. This means that the range of g(x) is {y €R:y > O}. b af(x) =g(1+ 2x) ae (1+ 2x) ‘The domain of the composite function gf is the set of values of x in the domain of function f that give values of f(x) that are in the domain of function g. What about the domain of g? The domain of function g is {x ¢ Rix > 1}, 80 we require the values of x in the domain of f for which f(x) > 1. Are there values in the domain of f that do not satisfy this? The only value of x in the domain of f that does not satisfy this condition is x = 0. This means that the domain of the composite function gf(x) is {< € Rx > 0}. © fate) (4) =1+2(4) x=1 x x-1+2 x-1 xt “x-4 ‘The domain of the composite function fg is the set of values of x in the domain of function g which give values of g(x) that are in the domain of function f. ‘What about the domain of f? The domain of function f is {x € R:x > 0). This means that we require g(x) > 0. Are there any values in the domain of g that do not satisfy this? All values of x in the damain of g satisfy the condition g(x) > 0. So the domain of the composite function fg(x) is {x €R: x > 1}; that is, the entire domain of g The inverse of a composite function ‘The flow diagram shows how the composite function fg is obtained by putting values of x into g, and then putting values of g(x) into f. x4 glx) falx) “ Complete Advanced Level Mathematics: Pure Mathematics 164 The inverse of this composite function is (/g)". This is obtained by reversing each of the individual component functions and the order in which they are applied, gyi ox Read from right to So values of x are put into f-', and then values of f-*(x) are put into g*. ‘This gives: wer Example 5 ‘The functions f and g defined for x € R by f: x =» x° and g: x =» 2x +1. Find: a ftandg? b fgand (fg) © gfand (gf)! nig) 24 g: = b Jalx) = f(2x +1) = (2x41) (BY) =s'F ee) =e VR) _Ve-1 =“ c Of(X) = gi) =a +1 @w=f a") (!) ator 2 substituting values f Note that having found the composite function, the inverse can also be found by writing it in the form y = fg(x) (or y = gf(x)), interchanging the x and y, and then making y the subject of the equation. Applying this technique to Example 5b, writing fg as y = (2x + 1)° and then interchanging x and y gives x = (2y + 1)®. Making y the subject of this equation, y = 3(5/@ — 1). So (fg)"(x) = 3 (V® —1). Applying this technique to Example Sc, writing gf as y = 2x? + 1 and then interchanging x and y gives x = 2y? +1. Making y the subject of this equation, y = 4/2 (x ~ 1). So (gf)"(x) = §/3 (e — 1), as before. 5 4 Functions 4.3 Composite Functions Exercise Technique ‘Tho functions f, g and h aro dofined by f(x) =x*, g(x) = $and h(x) = 2 — x, Find an expression for each of the following composite functions in terms of x. a fe bg c fh dif e gh f hg 2 # ho i oghf jo hep For each of the following, express function p as a composite of the functions fx —+ 5x, g: x «x 4+ 3and hx — sinx: @ px—5x43 do px Ssinx+15 b p:x—sin(x+3) © pxoxt6 G pxosinx+3 f pix — sin(Sx+15), Given that f(x) =x —3, g(x) = 10x and h(x) =} (x #0): a find an expression for fgh(x) b salve the equation fgh(x) = x x-5 Functions f and g are defined by f(x) = ax +5 and g(x) = a Find expressions for f*(x) and g*(x). b Show that f(x) = x and gf(x) = x. € Comment on the significance of your results in b. ‘The functions f and g are defined by f:x — 4 — x? for {x € R} and gix— YR for {x ¢R,x > 0}, a State the ranges of f and g for the given domains. b Find an expression for fg, and determine its domain. © Find an expression for gf, and determine its domain, ‘The functions f and g are defined by fx — 1” for (x €R) and se gx <2 for {xe 8.x 4 1).Find expressions forthe following. in each case stating any values of x for which the composite functions are undefined: a fax) Bb af(x) efx) ds g(x) Given that f(x) =x", g(x) = 1 = 3x and h(x) =? (x £0), find expressions for: a fig tandh b fe, (fa) band gf © hg, (hg) ‘and gt do fh, (fh) and hop! Utheberrechilich geschiitztes MiG5ial ‘Complete Advanced Level Mathematics: Pure Mathematics 4.4 Transformations of Graphs and Functions Translations Graphs of the formy = f(x) +a Using a graphical calculator draw \ 4 tyes: the graphs of y =x", vy =x° +5 and y * _ 3 on the same axes. How are the graphs of y =x? +5 and y = x* ~3 related to the graph of y = x7? Notice that the shape of the curve is the same. The curve has been translated, or moved. ‘The three curves all have the same shape, Although they appear to be getting closer together, their vertical separation at every value of x is constant; the graph of y =x" +5 is 5 units above the graph y = x*, while the graph of y =x? = 3 is 3 units below y =x", In fact the curve y =x? 45 is obtained by simply translating the curve y = x* by +5 units parallel to the y-axis and the curve y = x* — 3 is obtained by translating the curve y =x" by =3 unils parallel to the y-axis, This can be generalised as follows. Graphs of the form y = f(x +a) Usinga graphical calculator, draw the graphs of y = x? y = (x —4)* on the same axes. How are the graphs of y y= (x —4)* related to the graph of y = x7? x +3) and x+3)? and yest yy stx-4y eH, yates i | 166 i 4 Functions ‘The graphs of y = s*, y = (x +3)! and y = (x 4)? all have the same shape. The curve y = (x +3) is obtained by translating each point on the curve y = x* by —3 units parallel to the x-axis, The graphs of y =x’ andy = (x +3)? have the same value of y when the x-coordinate on the curve y= (x +3)’ is 3 units smaller than that on the curve y =X". The curve y = (x — 4)? is obtained by translating the curve y =x by +4 units parallel to tho x-axis. These results can be generalised, as follows. Example 1 ‘The diagram shows the graph of y = f(x) for f(x) = x* —3x* — 9x +11, Functions g(x) and A(x) are related to f(x) with g(x) = f(x — 1} and h(x) = f(x+2)—3, a Explain how the graphs of y = g(x) and y = A(x) cam be obtained from the graph of y = fix). b Sketch both of these graphs, indicating the coordinates of their turning points, © Show that g(x) = x7 — 6x? + 16 and h(x) =x? + 3x7 -9x- 14. yas) (3,=16) Solution a The graph of y = g(x) = f(x — 1) is obtained by translating the y= f(x) graph by +1 unit parallel to the x-axis. ‘The graph of y = h(x) = f(x +2) —3 is obtained by translating the y =J(s) graph by —2 units parallel to the x-axis, and ~3 units parallel to the y-axis. Urheberrechtlich geschitztesygyerial Complete Advanced Level Mathematics: Pure Mathematics b Notice how the coordinates of the turning points have been translated. © g(x) =fix-1) = (x1)? = 3(x - 1)? - 9(x-1) +11 = (x? — 3x? + 3x — 1) ~ (3x? ~ Gx + 3) ~ (9X -9) +11 =x! = 6x" +16, as required h(x) =f(x+2)—3 = (x +2)? a(x +2) -9(e +2) +11-3 = (x? + 6x? + 12x + B) = (3x* + 12x + 12) = (OX-+ 18) +8 =x° + 3x" = 9x = 14, as required Example 2 ‘The diagram shows the graph of y = x". The graphs of y = f(x) and ¥ = g(x) have been obtained by translating the curve y =x". Describe the transformation by which the graph of y = x* is mapped to the graphs of y = f(x) and y = g(x). b Find expressions for f(x) and g(x). 168 Urheberrechtlich geschiitztes Material 4 Functions. Solution a The graph of y = f(x) is obtained by translating the curve y = x" by +1 unit parallel to the x-axis, and +3 units parallel to the y-axis. ‘The graph of y = g(x) is obtained by translating the curve y = x* by —2 units parallel to the x-axis, and —4 units parallel to the y-axis, b f(x) =(e-1" +3 ‘Expanding the brackets and collecting like terms, f(x) =x - 3x? + 3x42 ax) = +2)'-4 Expanding the brackets and collecting like terms, a(x) =x" + 6x7 + 12044 Stretches Graphs of the form y = af(x) Using a graphical calculator, draw the graphs of y = si sinx on the same axes. How are the graphs of y = 3sinx and Look at the graphs of y = sinx and y = 3sinx. Both curves repeat themselves with a period of 360°. This is a characteristic of the sine function. However, the curve y = 3 sin x has an amplitude of 3. This is Amplitude is the bocauso its maximum and minimum values are +3 and —3 respectively. In greatest height reached fact, tho curve - = 3 sinx can bo drawn by simply strotching the curve by the wave from the y =sinx parallel to the y-axis by a factor of 3 (scale Factor 3). weanis. ‘The graph y = |simx also has a period of 360°. Notice that its amplitude is 0.5. It can be obtained by ‘stretching’ the curve y = sinx by a scale factor Complete Advanced Level Mathematics: Pure Mathematics 170 of } parallel to the y-axis. Notice also that, a stretch factor between 0 and 1 actually has the effect of compressing the curve (that is making it closer to the x-axis), ‘This can be generalised as follows: ph of y = f(x) the y-axis. Graphs of the form y = f(ax) jails Using a graphical calculator, draw the graphs of y = cosx,y=cos 2x and | v= cos($) on the same axes. How are the graphs of y = cos 2x and | iraph y= cos(§) related to the graph of y = cos x? Explain the transformation required to obtain the graph of y = f(ax) from the graph of y = f(x). ‘The graph of y = cos x repeats itself every 360°. The curve y = cos 2x has a period of only 180°. It completes two full cycles every 360°. In fact, the curve y = cos 2x has been obtained by ‘stretching’ the graph of y = cos x parallel to the x-axis by a scale factor of }. This has the effect of compressing each cycle into half the space. Notice that this is a one-way stretch and that the amplitude of the cosine curve is not altered. The graph of y = cos($) completes half of a full cycle within 360°. It has a period of 720°. It is obtained from the graph of y = cosx by a one-way stretch of scale factor 2 parallel to the x-axis. Stretches like this can be summarised as: Example 3 The diagram shows the graph of y = f(x), where f(x) = 3 — 2x —x*. This graph is mapped to "= g(x) by a stroteh of factor 2 parallel to the y-axis. 4 Functions ‘The graph of y = g(x) is then itself mapped toy = h(x) by a stretch of factor 3 parallel to the x-axis. a Sketch the graphs of y = g(x) and y = h(x). Indicate clearly the coordinates of the images of points A, B, C and D. b Find expressions for g(x) and h(x) in terms of x. Solution a ve D'-1,8) D(-3, 8) 1b The graph of y = g(x) is obtained by stretching the graph of y = f(x) by a factor of 2 parallel to the y-axis. So B(x) = f(x) = 2(3- 2x—2*) =6-4x— 2x" Urheberrechtlich geschiitzteg yigtarial Complete Advanced Level Mathematics: Pure Mathematics ‘The graph of y = h(x) is obtained by stretching the graph of y = g(x) by a factor of 3 parallel to tho x-axis. So ix) = g(x) ae gfx\? =0-2-2(6) 2 eae 3a. 9 Reflections Graphs of the form y = —f(x) Using a graphical calculator, draw the graphs of y = 2x + 1 and y = =(2x +1), How are these two graphs related to each other? Now draw the graphs of y =x? + 1 and y = —(x? +1), Are these two graphs related in the same way? yoxte1 ay ye ext) y=-(x8 +1) ‘The graphs of y = 2x +1 and y = =(2x + 1) are shown. The latter has boon obtained by reflecting the line y = 2x + 1 in the x-axis, Positive values of y = 2x +1 become negative values of y = ~(2x + 1). ‘The curve y = ~(x! +1) is similarly a reflection of the curve y = x? +1 in the x-axis. This can be generalised as follows: ofy =f (x) Graphs of the form y = f(—x) Use a graphical calculator to draw the graphs of y = 2x +1 and y = 2(—x) +1 om the same axes. Now draw the graphs of y = x? +1 and ¥ = (-x)' + 1. How are these curves related to one another? 172 4 Functions % % gees yest 1 J ‘a mx ° Pe: atap+ 1 yetaei ‘The graphs of y = f(x) and y = f(~x) for f(x) = 2x +1 and f(x) = +1 respectively are shown. In both examples, the graph of y = f(—x) is obtained by reflecting the graph of y = f(x) in the y-axis. This transformation can be summarised as: the graph of y = g(x) by a reflection in one axis, and a translation, Describe the transformations that have occurred. Clearly state the order in which they were carried out. Find an expression for g(x). f(x) = 2° Solution Notice that there are two way's in which the graph of y mapped toy = gtx): 2° can be @ reflection in the x-axis, followed by a translation of +2 units parallel to the y-axis; @ = atranslation of —2 units parallel to the y-axis, followed by a reflection in the x-axis. Urheberrechttich Complete Advanced Level Mathematics: Pure Mathematics: Using the first of these, an yer expression for g(x) can be found. | rfc in ands Therefore g(x) = -2" + 2y “2 | acsbation of + units parallel to yan 242 ‘The second combination of yao transformations gives the | twunelation of ~2 units pact to yon same expression. yor 2 | resection tae ~(2 -2)=-2°42 The general quadratic curve All quadratic expressions can be written in the form c|(x +a)* +6], where a, band care constants. Why is this rearrangement useful? The graphs of y= x" and y = o[(x +a)* +6] can be compared. The values ofa, b and ¢ then give an indication of the transformations used. Example 5 Express the function g(x) = 2x” + 4x +6 in the form e[(x + a)? +b], where a, b and ¢ are constants. Describe the transformations by which the curve y =x? is mapped to the graph of this function. Sketch the graph of y = g(x). Solution g(x) = 2x? 48x46 = 2x7 44x43) = (x7 44x 44-1) = 2[(x +2) - 1] The graph of y =x? is mapped to the graph of y = g(x) by translations of ~2 units parallel to the x-axis and —1 unit parallel to the y-axis, followed by a stretch of factor 2 parallel to the y-axis. applied before the stretch. 174 M: 4 Functions 4.4 Transformations of Graphs and Functions Exercise Technique The diagram shows the graph of y = f(x), where f(x) = a7 — 2x. A ae) For each of the following transformations of this function: i describe the transformation(s) geometrically; ii write down an expression for the new function: iff. sketch the graph of the transformed function, indicating the coordinates of its intersection with the axes and the turning point: a y=fixy42 b y=fix=3) ce y=f(2x) a p= -3f(x) e y=fi-}) £ opaseraea ‘The function f is defined by f(x) = x* + 6x +8. a Express f in the form f(x) = (x + a)* +b, where a and b are integers. b Describe the transformation by which the graph of y =x? is mapped to the graph of y = f(x). The function g is defined by g(x) = 2x? — 12x + 19. a Express g in the form g(x) = 2(x + a)" +b, where a and b are integers. b Describe the transformations by which the graph of y = x? is mapped to the graph of y = g(x). Clearly state the order in which they must be applied. Each of the following quadratic curves is a transformation of the curve y =x". In each case, state the transformation(s) that have occurred. State also the order in which they must be applied. e yaheda f y=x'-6x+10 g yoxt+ax hoo y=2x? 4x43 S475)" (Complete Advanced Level Mathematics: Pure Mathematics [By] the diagram shows the graph of y = x? —ax?, a Describe the transformations that 4 map this graph to each of the following graphs. State the order in which they must be applied. Use these transformations to write down the equation of each graph. a b ea) » 4 faa . * ho oa « [77] the function fx — 3, x 4 0 is transformed to give the function g. For each 4 a % \ so —h x Flas (4) Describe the transformations by which the graph of y = sin x is mapped to each of the curves below. Make a sketch of the graph of each of the transformed functions for -860° 0 and f(x) =-x, fore <0 Since its graph is symmetrical about 4 the y-axis, the modulus function Fx) = |x| is even. Ha) = |x ; x ‘To obtain the graph of y = | f(x)|, first sketch the graph of y = f(x). The parts of this graph for which y"is positive are kept: the parts for which y is negative are reflected in the x-axis. Example 3 Sketch the graphs of: a y= ?-a| b y=|sinx| Solution a First sketch the graph of y= x? — 4, Now reflect in the x-axis those parts of this graph that Ite below the x-axis. 179 Complete Advanced Level Mathematics: Pure Mathematics 180 b Sketch the graph of y = sinx and then reflect in the x-axis those parts that lie below the x-axis, yefain| ‘The graphs of functions related to the modulus function can be sketched by applying the appropriate transformations to the graph of y = |x|. Example 4 Sketch the graphs oft a y=|x+3] bo y= |ax+4). Solution a Thegraph of y = |x + 3] is obtained by translating the graph of y = |x| by —3 units parallel to the x-axis. y= |e +3] b Thegraph of y = [2x + ais obtained by stretching the graph of y = |x| by a factor of parallel to the x-axis, and a translation of —4 parallel to the x-axis. ‘To obtain the graph of y = fi(jx|), first sketch the graph of y = f(x) for positive values of x. For negative values of x, the value of y = f({x}) is found by substituting the equivalent positive value of x into the function. This simply reflects the graph of y = f(x) for positive values of x in the y-axis. Example 5 Sketch the graphs of: a ye |s|-2 boy =sin|x}. Urheberrechtlich geschitztes Material SS ee 4 Functions Solution a Thegraph ofy =[x| ~ 2 is obtained by first drawing the graph of y= x—2 for positive values of x, and then reflecting this in the paris. b Thegraph of y ~sia|s| y is obtained by first 7 drawing the graph of fete y =sinx for positive iviiuee dteeand thon reflecting this in the ai am axis. Periodic functions Periodic functions have graphs that regularly repeat themselves. Examples af periodic functions include the sine, cosine and tangent functions. The graphs of y = sinx and y =cosx repeal themselves every 360° or 25 radians, This value is referred to as the period of the function, The graph of y = tanx has a period of only 180° or z radians. Urheberrechtlich geschatztes 4B Bria Complete Advanced Level Mathematics: Pure Mathematics 182 In general, periodic functions f(x) are such that for some constant k, Kis the smal Stk (x), for all values of x, where k is the period of the function, distance aver which function repeats itsel Writing the trigonometric functions in this form sin(x £360") =sinx — cos(x +360") =cosx — tan(x +180") Once the behaviour over one period is known, the graph of a periodic function can be drawn. Example 6 ‘The function f(x) is periodic with a period of 4 units. It is defined by Sixy= hx", O 0} and g:x— VF, for {x ©R:x > O}. a State the range of f. b Sketch the graph of f and hence explain why the inverse function f-* exists. Find f-', stating its domain, ¢ Thecomposite function f og is defined for x > 0. i Find f og(x). ii Determine the exact surd solution to the equation f(x) =fo.g(x). (AEB) f(x) | The function f has as its domain the set of all non-zero real numbers, and is given by f(x) =} forall x in this set. On a single diagram, sketch the following graphs, and indicate the geometrical relationships between them: a y=f(x) b y=fa+1) © y=f(x+1)+2 Deduce, explaining your reasoning, the coordinates of the point about which the graph of y = mats in symamatiical, (UCLES) 184 4 Functions 5 | Function f is defined on the domain 0 q} is one-to-one. Find the least value of q and sketch the graph of this function. (ULEAC) 8 | The diagram shows the curve y = f(x +a). where a is a positive constant. ‘The maximum and minimum points on the curve are (~a, 3a) and (a, 0) respectively. (a, 3a) y=fix+a) Sketch the following curves, on separate diagrams, in each case stating the coordinates of the maximum and minimum points: a y=fix) b y=-2(x+a). (UCLES) Q | ‘The function f is defined on the domain x > O by f(x) =1+2. a i Find an expression for f-'(x). ii State the domain of f-. The composite function g is defined by ¢ = ff. bi Find an expression for g(x). ! fi State the range of g. (WJEC) Urheberrechilich geschitztes MB&Sial ‘Complete Advanced Level Mathomatics: Pure Mathematics Function f is defined on the domain ~1 O} by fix —+x4, g:x 4 186 and hi: x — v&. Express in terms of x: a ghts) b fehie) ‘State which two of the three functions f, g and h are inverses of each other. (UCLES) 4 Functions. [4] Functions f and g are defined with their respective domains by fix poy for {xe Rx ¢ dp and g: x + x* +1 for {x eR}. a Find the values of x for which f(x) =. b Find the range of g. ¢ The domain of the composite function fo g is R. Find fo g(x) and Recall the alternative state the range af fo g. notation for the (AEB) composite function 5 | Function f is defined on the domain 0 < x < 2 by f(x) =x?. Function g is defined by translating the graph of y= f(x), with this domain, 3 units in the positive x direction and 5 units in the positive y direction to give the graph of y = g(x). Skotch the graphs of y = f(x) and y = g(x). State the domain and range of function g. Find an expression for g(x). Find an expression for g(x). State the domain and range of the function g~', seaoe (NEAB) 6 | Express x? + 4x im the form (x +a)? +5, stating the numerical values of a and b. Functions f and g are defined as f:x — x? + 4x for {x €R:x > -2) and g: x +x + 6 for (x € R}. Show that the equation gf x) = 0 has no real roots. State the domain of f-'. Find an expression in terms of x for f~*(x). Sketch, on a single diagram, the graphs of y = f(x) andy = f(x). (UCLES} aoore 7 | The diagram shows the graph of the function f defined for x > 0 by fixalt ye a Copy the sketch, and show on the same diagram the graph of f-', making clear the relationship between the two graphs. b Give an expression in terms of x for f(x), and state the domain of f-", x ¢ There is one value of x for which f(x) = f~!(x). By considering your diagram, explain why this value of x satisfies the equation 14+ Veex. By treating the equation 1 + YX =. as a quadratic equation for YX, or otherwise, show that the value of x satisfying f(x) = f-'(x) is x=1(3+ V5). (UCLES) Complete Advanced Level Mathematics: Pure Mathematics 188 Applications and Activities ‘There are four possible functions that map elements in the set {a,b} across to elements in the set {p,q}, as illustrated by the mapping diagrams below. (a\-> {(*) ae fe) (a). we Wy EY ee In the first two mappings, {a,b} is the domain and {p,q} is the range, containing all the output values. However, in the second two mappings, {p.q} is the co-domain of the functions, containing the possible output values. Now consider the functions that could possibly map elements in {a,b,c} to elements in {p,q}. a How many have {p,q} as their range, and how many have {p,q} as their co-domain? What is the total number of possible functions? © How many different functions exist in the more general case of m elements in the domain and n elements in the co-domain? Summary A function is a many-to-one or one-to-one mapping for which each input value in the domain gives only one output value. ‘The range of a function is the set of output values to which values in its domain map: the range is a subset of the co-domain. Only functions which, fora given domain, are one-to-one mappings, have inverse functions. ‘The graph of the inverse function y = f~"(x) is ebtained by reflecting the graph of y = f(x) in the line y = x for values of x where f is one-to-one, An algebraic expression for f-"(x) can be found by interchanging x and y in the equation y = f(x) and then rearranging it to make y the subject of the equation again. Functions for which f~!(x) = f(x) are self-inverses, ‘The inverse trigonometric functions y = sin“'x, y = cos”'x and y = tan”! x only exist for the restricted domains in which the sine, cosine and tangent functions are one-to-one. 4 Funetions @ The composite function fe(x) (alternative notation fo g(x) is obtained by pulling values of g(x) into function f. ‘The composite function f(x), or f*(x), is obtained by putting values of f(x) ‘back into the function f again. “The graph of y = f(x) can, by a combination of translations, stretches, and reflections, be transformed into the graph of a related function, as shown ‘in this table. feta) translation of +a units parallel to the y-axis translation of —a units parallel to the x-axis af{x) one-way stretch, of factor a, parallel to the y-axis Flax) one-way stretch, of factor , parallel to the-x-axis, = fle) reflection in the x-axis fx) Fetinclion falta panis Even functions are those that satisfy the condition f(—x) = f(x), and are therefore symmetrical about the y-axis. ‘Odd functions are those that satisfy the condition fx) therefore have rotational symmetry about the origin, Periodic functions are those for which f(x +k) = f(x) for some constant k: such functions repeat themselves every k units along the x-axis, and k is the period of the function. f(x), and. Urheberrechtlich geschitztes MEBMal (Complete Advanced Level Mathomatics: Pure Mathematics 5 Differentiation | What you need to know @ How to find the gradient of the straight line joining two points. @ = How to find the value of a function. @ How to find the equation of the straight line that passes through a given point, when its gradient is known, @ How to solve simple trigonometric equations. @ How to use negative and fractional indices. Review [71] Find the gradient ofthe straight line joining the following pairs of points: a (2,1) and (6,9) b (0,7) and (3,1) © (~2,11) and (1,—7) d= (~1,~2) and (-4,-11) [2] Find the value of the following functions at the given value of x: 2? + 5x46, when 3x +h. when x =} sin 2x ~ 2 cos* x, when x = J [3B] Fina the equation of tho straight line that passes through the given point ‘with the gradient indicated, giving your answer in the form y = mx-+c: a (1,4), gradient 6 hb (3,—11), gradient —3 © (~5,2), gradient —} d= (2,3), gradient 2 [4] Solve the following trigonometric equations for 0 <0 < 2m: b sind +cos?=0 d cos20+ cos? =0 [5] ‘Write each of the following in index notation: 2 10 a ak bo ove e (yx* 1 x5 190 5 Differentiation | 5.1 Finding the Gradient of a Curve ‘The gradiont at each point on the graph of a linear function, such as y= 2x-+ 1,is constant, Howover, many mathematical functions are not linear, and have curved graphs whose gradionts are continuously changing. Look at the graph of y =x”, Notice that the gradient changes from being negative to positive as the graph crosses the y-axis. It also becomes steeper for larger positive and negative values of x. Finding the gradient of the curve is not as straightforward as it is for the linear function. The gradient of the curved graph at any particular point can be found by calculating the gradient of the tangent to the curve at this point. Look at the graph of y = x* again in more detail. vi 16: Draw tangents to this graph at the points x = —3, —2, -1, 0, 1, 2, 3. Notice that for x > 0, the gradient is positive and increases as the tangents become steeper. When x < 0, the gradient is negative, andl Complete Advanced Level Mathematics: Pure Mathematics 192 becomes more negative as x decreases. By constructing right-angled triangles and using change in y _ Ay gradient = pange in x Ax we can estimate the gradient of the curve at these different points, f'(x) =2x-7 Substituting x = 3, f"(3) = 6 —7 =-1 b u(0) = (0? —1)(041) =P+4+0 0-1 => (0) =308 + 20-1 Substituting@ =3,u'(}) =3x}+2%4-1 =3+}-1-0 Example 7 Find the coordinates of the point(s) on the following curves at which the gradient has the value indicated: Solution a yar +4axt1 Vomes ay = SE = 8, then 2x44 = 8 ‘This gives 2x=4 3 x=2 = ya(2))+(4x2)41=13 ‘So, the gradient of the curve y = x? +4x +1 is 6 at the point (2, 13). 5 Differentiation I Whenx=}y=7)-2x()=2-1=1 oe ® When x=}, y= 35-2 (-D=-241=-1 i So the curve y =} — 2x has a gradient of —6 at the points (3,1) and (-},-). Differentiating with respect toy It.can be shown that the rate of change of x with respect to y, $s, is the reciprocal of ft. * 4 Learn this result. For example, ify = ax* 42x, then #¢ = 6x + 2, and gt = 1. = — 1 @ wre ‘This particular method is quicker than having to rearrange y = 3x7 + 2x to ‘express x in terms of y before differentiating with respect to y. Higher derivatives Sometimes it is useful to know the gradient of the gradient function at a particular point on the curve y = f(x). This is called the second derivative. It is found by differentiating the first derivative, $, with respect to x to give (#2), written £2, or f"(x). ‘The expression $4 is read “d-two-y-by-d-x-squared’, and shows that the differentiation process has happened twice. It is not the same as squaring i. Differentiating again with respect to x would give the third derivative, 4, or f"(x). Higher derivatives, of the form “ or f(x), can be obtained by differentiating y = f(x) n times with respect to x. Example 8 Find the first, second and third derivatives of the following functions: a yan! + 5x? - 9x7 + 2x-7 b fax)=x-} 201 Complete Advanced Level Mathematics: Pure Mathematics Solution a yaxt+5x- 9x7 +2x-7 = Wo ay 4 5x! 10x42 dy od %) dx? ~ dx \de. d = g(t + 15x? — 18K +2) = lex? +30x— 18 dy_d (dy de "de ) a = Gyll2” + 30x ~ 18) = 24x + 30 b F(x) = 3x? +x? a =at45 f' (x) = 6x =2K* 2 =Ox-5 f"(x) = 6 + 6x4 6 =045 Urheberrechilich geschtitztes Material 5 Ditforentiation | 5.1 Finding the Gradient of a Curve Exercise Technique 1 | Differentiate each of the following from first principles to find a ysx’ b ysax'-ox+5 oc ya! [2] Differentiate each of the following with respect to x: a yess? e ye-3 bo y=7+x—-ax'-1x% £ ye © y= (x- 8) +1) Bay dye h yao Find the first derivative of each of the following functions: a fix) =x? = by ad hlp) = b sO « fis)= [4] Find the gradient of each of the following curves at the point indicated: a y =x" +6x—3 at (2,13) b oy=2x?=7x= Salt (=1,0) © y=3+8 at (2,3) ds y= (x? —2)(x +1) at (-3, -14) +1)(2-1) e at)=er! —7s' — 2s Find the coordinates of the paints on each of the following curves at ‘which the gradient has the value indicated: a y=x"— Gx? 47x, where $= —2 bo y=3-5x+x", where 2 ys2xt1—4, where $=1 ds y=t, where #=-4 6 | Find the gradient of each of the following curves at their points of intersection with the x- and y-axes: a yoxt+2x-3 b y=(2x+3)(x-1) [7] Find the fist, second and third derivatives of each of the following: a yaSxt+od-mt-ox+2 bfx) =3-4 Urheberrechtlich geschatztes #3 ial ‘Complete Advanced Level Mathematics: Pure Mathematics 5.2 Stationary Points Look al the graph below. What do points A, B, Cand D have in common? y=f Oo Colin Maelauri (1698-1746) Maclaurin was the fi ___ present the corre a for cistinguis Notice that the gradient of the curve is zero at these four points. This means that $¢ = 0 at A,B,C and D. Points like A, B, Cand D are known as stationary points. They correspond to values of x for which the gradient of the curve is zero. This means the function f(x) is momentarily ‘stationary’, that is neither increasing or docreasing. Points A and C are known as turning points on the graph. Notice that at a turning point f(x) changes from being an increasing function of x to a decreasing function of x, or vice versa. Point A is called a local maximum. point and point G isa local minimum point. What about points B and Dt f(x) is a decreasing function on either side of point B, It is an increasing function on either side of point D, but the gradient is zero at both points, Such points are called stationary points of inflexion. Distinguishing between stationary points \ local dy. ‘minimum a <° point F(s) decreasing J (s) increasing W co J (x) decreasing f(s) increasing point 5 Differentiation | Asa graph passes through a local minimum point, its gradient function, changes from being negative to positive. It is zero at the minimum point itself. On either side of a local maximum point, there is a corresponding change in the sign of {f from positive to negative. £ >o J (3) increasing Boo wo f(s) increasing stationary points a of inflexion <0 =i F(x) decroasing oy 1 When x =1, y = (1)'—2(1) —15 = —16. So, the stationary point is at (1,—16). Now look at £ on either side of x = 1, Whonx=0, a —2. thatis <0. When x =2, # That is, x >o. Since the gradient changes from being negative to positive as the graph passes through (1,16) this is a local minimum point. The graph of the quadratic can now be sketched by drawing a parabola through the four calculated points, 1 yo xt-2e-15 Uthebertectlich geschitiztes Material 5 Differentiation 1 Example 2 For each of the following cubic functions, find the coordinates of any_ stationary points, and determine their nature. Use a graphical calculator to plot the graphs of these functions. Use the TRACE facility to check your. results, a y=x'=3x*=9x +10 bo yax'=ax? 43x41 Solution a The gradient function for the graph of y = x’ — 3x* — 9x + 10 is given by # = 3x7 = 6x = 9. To find the location of any stationary points on this graph, solve 4 = 0. 3x*- 6x-9=0 == a(x? -2x-a)=0 = Fat (x-3 = x=-lorx=3 When x = —-1, y = (-1)" —3(-1)* -9(-1) +10 = -1-349+410=15 Whenx=3, y =(3)~3(3)* - 9(3) +10 =27-27-27+10=-17 So, the graph of y = x" — ax? — 9x +10 has stationary points at. (+1, 15) and (3, ~17). Now look at the sign of $! on both sides of x = —1. When x = -2, B -5(-2)? -6(-2)-9=15. That is, Zoo. ma eta a when x=0, 2 — 3(0)?—6(0)-9=~9. Thats, Weo dx Since the gradient changes from positive to negative as the graph passes through (~1,15), this is a local maximum point. Now look at the sign of #¢ on both sides of x = 3. ao Wa r dy: When x= 2, 5 >= 3(2)'—6(2)-9=—9. Thatis, 72 <0. When x = 4, a =3(4)"-—6(4)-9 = 15. That is, Yo dx 207 ‘Complete Advanced Level Mathematics: Pure Mathematics Since the gradient changes from negative to positive, (3, -17) isa Jocal minimum point on the curve, yaxt-art- ox +10 The y-intercept is for by substituting x = 0 into the equation of (3,-17), b Any stationary points on the graph of y = x? — 3x? + 3x + 1 are found by solving # = 0. since Y oa?—ar43 dx then ao = of 6x49=0 => 3(x? — 2x41) =0 > a(x—1)' =0 > x=1 When x 1 y= (1) -3(1 +31) +1 =2, So the only stationary point on this curve is located at (1,2). ‘To determine its nature, look at the sign of {! on either side of x = 1, Whon x = 0, as, That is, so. mg a Whonx=2, Go=9. That is, 5 >0. So the gradient is positive on both sides of (1,2), which must be a stationary point of inflexion, yaad ant + 341 5 Ditferentiation | Increasing and decreasing functions We are sometimes more interested in whether a function is increasing or decreasing at various points along its curve. At points on the graph of y= f(x) whore the gradient is not zero, y must be either an increasing or decreasing function of x. @ If: > 0 (a positive gradient), then y is an increasing function of x. @ | 1f3¢ < 0 (a negative graclient), then y is a decreasing function of x. For continuous functions, sections of increasing and decreasing behaviour are always separated by a stationary point. In order to find out for which values of x a function is increasing or decreasing, its stationary point(s) must first be located. Example 3 Find the values of x for which the function y = x* — 9x7 + 15x +13 is increasing. Solution yaxt— ox? 415x413 dy _ ay? = Gp rast = tox +15 At stationary values of function y, 2° = 0. => 3x7 = 18x +15 =0 => B(x? -6x45)=0 = B(e—1)(r-5)=0 = w=1orx=5 ‘To dotermino the nature of these two stationary values, look at the ‘ f gradient of #f on both sides. when e=o, %— 15. Thatis, &> 0. dx dy seg ig A When x=3, G= 12. Thatis, 5 <0. When =6, Maas. thatis, > 0 ‘Complete Advanced Level Mathematics: Pure Mathematics ‘This means that y has a maximum value at.x = 1 and a minimum value at x = 5. It follows that y will be a decreasing function in the interval 1 5. x = 9x? + 15x 19 Increasing | decreasing! increasing function | function | function forxe 1! for | forx>s exes] Optimisation ‘Many practical problems involve finding the maximum or minimum values of a function as it changes with respect to a particular variable. If the problem can be modelled in terms of a mathematical function, differentiation can be used to locate and distinguish between any maximum and minimum values. Example 4 An open-topped gift box is formed by cutting squares of side length x from each corner of a 60 cm 60 cm square of cardboard, and folding the remaining flaps to make the vertical sides. ae ‘i Ay 60cm Y \ a oe a Find an expression in terms of x for the volume V (in cm) of the gift box. b Use differentiation to find the value of x that corresponds to the maximum possible volume of the gift box. ¢ Calculate this maximum volume. 210 5 Differentiation | Solution a The volume of the folded box is given by V = length of box x width of box x height of box and the dimensions of the box are length = width = 60~2x and height = x So V = (60 — 2x) x (60- 2x) xx. = x(60— 2x)* V = 4x! —240x + 3600x b Differentiating with respect to x, the gradient of this volume function is given by av z= 12x" — 480x + 3600 At the maximum and minimum volumes, = 0 => 12x" — 480x +3600 = 0 = 12(x? — 40x + 300) =0 = 12(x—10)(x-30)=0 > x= 100rx=30 Look at the sign of 4 on both sides of x = 10. When x= 5, wv = 12(3)? ~ 480(5) + 3600 = 1600, That is, >0. When x= 15, x = 12(15)* — 480(15) +3600 = —900. ‘That is, = <0. ‘This means that x = 10 corresponds to a maximum value of V. Look at the sign of ¢ on both sides of x = 30, ‘When x = 25, * = 12(25)? — 480(25) + 3600 = —900, ‘That is, “ <0. When x = 35, aa = 12(35)* — 480(35) + 3600 = 1500. ‘That is. ¥ +0. Urheberrechtlich Complete Advanced Level Mathematics: Pure Mathematics 212 ‘This means that x = 30 corresponds to a minimum value of x. In fact, the volume of the folded box is zero when x = 30 because cutting squares of this size from each corner of the original sheet would leave no cardboard at all to fold into a box, c Fromb, the maximum volume of the folded box occurs when x= 10cm, Then V = 4(10)° — 240(10)* + 3600(10) = 16 000em* In many optimisation problems, the function to be maximised or minimised is dependent on twe variables. However, usually one of these variables can be eliminated using additional information given about the situation, Example 5 A manufacturer wishes to make cylindrical stee! cans with a capacity of 500m! using the smallest ee quantity of metal possible, (Remember that h 1 millilitre = 1.cm’.) “ a Find an expression for the total surface area S cm? of a cylindrical can, in terms of its radius ¢ em only. b Find, correct to one decimal place, the values of rand A, the can's height, that would give the smallest surface area. Solution a The lotal surface area of the can, S, is given by S=2nr? + 2erh i) At the moment Sis expressed as an equation with two variables, r and h. We are required to express it in terms of r only, so we must eliminate h. ‘The volume of the cylinder is given by V = rh, where h is the height of the ean, Far this particular can the volume is 500.cm*, so arth = 500 (2) 500 Rearranging equation [2], 5 Differentiation | ‘Substituting this expression for h into equation [1], = 2nr! + ann Oe Eliminating xr in the second term, = 2a? 4 1008 T ds 7 = 4ar— 100077 =4er— as At the maximum and minimum values of surface area, 43 - p= 1000 _ 250 ioe > ve =4.3.cm (1d.p) Substituting for r in the rearranged form of equation [2] to find the corresponding height, Re 4-3 500 ee (VF) = 86cm (1 dp) ‘To decide whether these values for the radius and height correspond toa maximum or minimum value for the surface area, look at the value of $8 on cither side of r = 4.3. When r= 4, Baton Ow 9.2 ‘That is, Sen Whon r= 5, $8 20;— 1000 22, That is, $50. aay bss tacrse in minimising wasted ‘This moans that the surface area of a $00 ml cylindrical can is metal when cutting the minimised when r = 4.3 cmand h = 8.6 cm. pieces, and this may sive different values of r and h. Urheberrechtlich geschitzt®"Baterial Complete Advanced Level Mathematics: Pure Mathematics 5.2 Stationary Points Exercise Technique 1 | For cach of the following quadratic functions: i find the coordinates of the points where the graph of y = f(x) crosses the axes ii find the coordinates of the stationary point on the graph, and determine its nature sketch the graph of = f(x). Check your graph using a graphical calculator. f(x) =x* - 26-8 b f(x) =35 +2¢—x7 ce f(x)=4x*~16x-9 d fix) =x? + 8x +28 2 | For each of the following cubic functions, find the coordinates of any stationary points, and determine their nature: a pax’ +6x? 4 12x +7 bo y= 2x) — 12x74 186-5 G vox +7x7 + 19x +2 do ysax’-axt +a 3] a Find the coordinates of the points where the quartic curve + — 12x" crosses the axes. b Find 8, Hence find the location, and determine the nature, of the slationary points on the curve. © Sketchy = x* — 12x*. Check your graph using a graphical calculator. 4 | For cach of the following functions, find an expression for f'(x), and hence locate and determine the nature of any stationary points on the graph of y= SQ) a fiy= x4 b Js) - 52 « faa See aes 5 | Given that s = 3? — 8f + 3, find the minimum value of s and the value of t for which this occurs. 6 | Given that v = 26 + 11r— 2, find the maximum value of v and the value of r for which this occurs. 7 | Find the values of x for which f(x) = 20 + 8x — x? is a decreasing function of x, 8 | Find the values of t for which g(t) = + 31 — 9f +6 is an increasing function of f. 214 5 Differentiation | Contextual [1] An open box is formed from a 120 cm x 70 cm rectangular sheet of cardboard by cutting squares of side length xcm from each corner. The remaining flaps are then folded up to make the vertical sides of the box. 70cm 120cm a Find an expression in terms of x for the volume V (in cm’) of the box. b Find 4 and then solve the equation {¢ = 0. Find the maximum possible volume of the box. Justify your answer. 2 | A garden centre wishes to use fencing to enclose three equally sized rectangular plots next to each other, as shown in the diagram. The total area A of the three plots is to be 288 m*. x x x a Find expressions for the total area A (in m*), and the total length L (in m) of fencing required in terms of x and y. b Express £ in terms of x only. © Hence, find the dimensions of each rectangular plot if the total length of fencing is to be kept as low as possible. Justify your answer. d_ What is the minimum length of fencing required? [3] one particutar day, the #inanctal Times 100 Share Index (FTSE) opens in London at 4000. During the rest of the day, its value ¢ hours after the start of trading at 9 a.m. is given by F = 4000 — 161? + 81° —3t*. A broker is instructed to sell her client's shares only if the value of the FTSE is, falling. a What is the value of the FTSE at noon? b Calculate the highest value of the index during the day. To the nearest minute, at what time does this occur? i chilztespqgrial Complete Advanced Level Mathematics: Pure Mathematics ¢ Iftrading finishes at 4.30 p.m., by haw much has the index risen or fallen during the day? d= During which times of the day could the broker have sold her clients shares? {4] A cuboidal water tank, of height hcm, width x cm and length 2xcm, is 1litre = 1000cm?. designed to hold 700 litres when full. Show that h = 22200, Find an expression for the total surface area, S m*, of the six faces of the tank in terms of x only. ¢ Find the dimensions of the tank that correspond to the least surface area, Justify your answer. (Give your answers to the nearest cm.) ce [5] A school decides to organise a monthly raffle in order to raise funds. Tt estimates that 2000 tickets would be bought if the price of each one was 50 pence, and that only 1000 tickets would be bought if they cost £1 each. ‘The cost cof organising the raffle each month is £150 for prizes and 2 pence per ticket for printing, a The number of tickets sold is modelled by n= is the selling price, and a and b are constants. b Show that ¢ = (21000— 40s) pence. Show that, in terms of the selling price s, the monthly profit generated by the raffle is given by p = 3040s — 21 000 — 20s", and find the selling price that would maximise the profit. Calculate the meximum profit and the number of tickets sold to achieve it. + bs, where s pence id aand b. 216 5 Ditferontiation | 5.3 Further Applications of Differentiation Tangents and normals ‘The gradient of a tangent drawn to a curve at any particular point is the same as the gradient of the curve al this point. This means that the gradient of a tangent can be found by differentiation, For example, consider the tangent drawn to the curve y = x7 at the point (1,1). va ) ioe P angen ft) *Y ‘The gradient function of this curve is $ — 2x. So the gradient of the curve, and therefore the gradiont of the tangent, at the point (1,1) is 2. Recall from Chapter 2 that the oquation of a line with gradient m, that passos through a point with coordinates (x1,), is given ‘ve ) So the equation of this particular tangent is given by y — 1 = 2(x — 1) or yo2x-1. The normal toa curve at any point is the straight line that passes through the curve al right-angles to the tangent at that point. Because the tangent and normal are perpendicular to each other, their gradients, m, and ms respeatively, satisfy the condition mym; = =1. So, 4 : ianpent +Learn this result. So the gradient of the normal to the curve y = x* at (1,1) is —}. Now use ¥—¥1 =1m(x —x;,) to find the equation of this normal, ¥-1=-}(x-4) + y-t--}r4} > ya=-prt] Complete Advanced Level Mathematics: Pure Mathematics Example 1 Find the gradient of the curve y = (x + 2)(4 —x) at the point where x = 3. Hence, find the equations of the tangent and the normal to the curve at this point, Solution y= (e+ 2)4—x) =B8+2n—x7 so Yo2-2 When x = 3, y = Sand So the gradient of the tangent to the curve at (3, 5) is —4. ‘This means that the gradient of the normal is |. Now use y — y, = m(x —x,) to find the equations of both the tangent and the normal to the curve at (3,5). ‘The equation of the langent is y — 8 = —4(x ~ 3) + yo = av 412 * ys -4x417 ‘The equation of the normal is y ~ 5 = | (x ~ 3) > yoteet as x-4y417=0 Recall that straight line equations, involving fractions, can be rearranged into the form ax + by +¢=0. Use a graphical calculator or graph plotting software to draw the quadratic curve y = (x + 2)(4 —x). On the same graph plot the lines y = —4x + 17 and y = Lx +2. Verify that they are the tangent and normal to the curve at the point x = 3. Example 2 Find the equation of the normal to the curve y = x* + 4x — 2 at the point where x = ~3. Find the coordinates of the other point where this normal intersects y = x* + 4x — 2. Solution your +4ax-2 gs Ba 218 5 Differentiation | 2. Since the gradient of the tangent ta the curve at (—3,—5) is 2, the gradient of the normal will be 3. Use y —y, = m(x — x,) to find the equation of this normal. At the points of intersection of this normal and the curve y =x* + 4x ~2, x4 dx-2=1x-] = axttax-4ax-7 = wwittxet3=0 => (2x +1)(x+3)=0 = x=-lorx=-3 ‘This means the x-coordinate of the other point of intersection is ~ }. The corresponding y-coordinate can be found by substituting x = — } into the equation of the normal. Check that this gives y = — "5, So the normal to the curve at (~3,-5) crosses the curve again at (~4,—48). Check this result using a graphical calculator and the TRACE facility. tangent 7 Displacement, velocity and acceleration The branch of mathematics concerned with the study of moving objects, and in particular their displacement, velocity and acceleration, is called kinematics. ‘The displacement s of an object moving in a straight line is the distance it has travelled from a fixed point on the line in a specified direction. In many real-life situations, displacement can be a function of time t. Utheberrechtlich geschitzia4 Satori ‘Complete Advanced Level Mathematics: Pure Mathematics ‘The velocity v of an object is the speed with which it is travelling in the specified direction. By definition, velocity is the rate at which the object's displacement is changing with respect to time. So v = nent is measured in metres and time is measured in then the units of velocity are metres per second (written m/s ) @ If =0 then the object is stationary. Ifdisplac second: or ms” @ = ifv >0 then the object is moving along the line in the specified direction. @ Ifv <0, then the object is moving in the opposite direction. ‘The acceleration a of a moving object is the rate at which its speed in the specified direction is changing. So acceleration is the rate of change of velocity with respect to time; Notice that acceleration is the first derivative of velocity, and the second derivative of displacement. If velocity is measured in ms”! then the units of acceleration are metres per second per second, or ‘metres per second squared’ (written m/s* or ms~*). @ = Ifa =0 the object is maving with constant velocity (that is at constant speed in a straight line). @ Ifa > 0 the object is accelerating: that is its speed in the specified direction is increasing. @ Ifa <0, then the object is decelerating: that is its speed in the specified direction is decreasing. Example 3 The height ft metres at time f seconds of a ball thrown vertically upwards, from a fixed point O, with initial velocity 12 ms”, is given by h= 12-50, Find the greatest height reached by the ball. Find the acceleration of the ball, Find the height of the ball after 0.3 and 1,7 seconds. Find the distance travelled by the ball between these two times. Find the average speed of the ball during this time interval. saece 5 Ditferentiation | Solution Height h takes the place of displacement s in this problem. The velocity of the ball at any time is given by dh a 2=10t (ms!) At its maximum height, the velocity of the ball is zero, Then 12-10! =0 = [= 1.2 seconds When f = 1.2, A = 12(1.2) —5(1.2)? =7.2 So, the greatest height reached is 7.2m. 4 (12 — 101) = —10 (ms"*) © Whent=0.3,h = 12(0.3)—5(0.3)' = 3.15m b Acceleration, a = 4 ‘When t = 1.7, h = 12(1.7) — 5(1.7)° = 5.95 m Consider the graph of h against t. Height h (metres) 0 03 iz a7 Time t (seconds) ‘The ball reaches its greatest height and changes direction between £=0.3 seconds and t = 1.7 seconds. The distance travelled during this time interval is the sum of the distance the ball rises and the distance the ball then falls. Between t = 0.3 seconds and f = 1.2 seconds, the ball rises and. travels (7.2 ~ 3.15) = 4.05 metres upwards. Between t = 1.2 seconds and f = 1.7 seconds, the ball falls and travels (7.2 — 5.95) = 1.25 motros downwards. So it travels a total distance of (4.05 + 1.25) = 3m. stance travelled @ average speed = So 5.3(m) 14(s) =3.8ms" (1dp.) rheberrechtlich geschitztes Mgipel Complete Advanced Level Mathematics: Pure Mathematics Small changes Ify = f(x), then recall that by definition the gradient of its graph at any particular point is given by Remember that dy is the small change in the value of the y-coordinate at this point corresponding to a small change, 6x, in the x-coordinate. Provided 5x is very small, This means that the ratio gives a good approximation to. the gradient function at that point. Remember that 4 is the gradient of the curve y= /(8) at the point from which this small change is being made. This approximation allows the change in the value of y resulting from a small change in the value of x (or vice versa) to be calculated quickly. Example 4 ‘The side length of a 100.cm x 100cm x 100.cm x cube is increased by 2 cm. Find the approximate increase in the cube’s surface area. Solution If the side length of the cube is x, then the surface area $= 6x, Because the change in the side length, 5x, is small relative to the original length, 65 _ ds ace (de this approximation is where 58 is the corresponding small change in surface area, and #is the Pul.* = 102cm in thy value of the gradient function when x = 100cm, Therefore equation for surface area, $ = 6x", The ne ssa cae surface area is a 62.424em’. The actu = 12x x dx increase in surface ar = 12(100) « 2 = 2400 cm? is therefore 2424 cm’ the approximation is ‘The approximate increase in the cube’s surface area is 2400cm*, good, 5 Differentiation 1 ‘The change in a measurement or quantity is often expressed as a percentage of the original value. Ifv = f(x), then + x 100 is the percentage increase (+) or decrease (—) in the value of x, and £4 = 100 is the corresponding percentage increase (+) or decrease (—) in the value of y. Example 5 ‘The time period T of a simple swinging pendulum is a function of the length Z of the pendulum, such that T = 2ny/L/g, where g isa constant. Find the percentage change in the period if the pendulum is shortened by 6%. ‘The percentage change in the value of L is 6%, the minus sign indicating a decrease. Therefore: oL = TOG 48% = 7 = 0.06 Because this is a small change “That is, = 3 xBatx -0.06 = ~0.03, ‘The minus sign ‘This means, the percentage change in the period of the pendulum, as the indicates a decrease in length is shortened, is a decrease of 3%, the value of 7, Urheberrechtlich geschotztiaggigaterial Complete Advanced Level Mathematics: Pure Mathematics 5.3 Further Applications of Differentiation Exercise Technique {1] Find the equations of the tangent and normal to each of the following Give your answers in curves at the points indicated: the form * — 5x +1 at (6,7) (2x +1)(x — 5) at (1,12) *— 6x? +4x+ 1 at (3-17) y=x+at (2,33) v¥ at (9,3) x — x? al (-1,—F y mene ee 2 | Find the equations of the tangents to the curve y = x? = 4x +3 at the points where it crosses the x-axis, Show that these tangents intersect at the point (2, -2). 3 | show that the equation of the tangent to the curve y = x? — 6x? + 3x +10 at the point where it crosses the y-axis is y = 3x + 10. Find the coordinates of the other point on the curve whose tangent is parallel to ¥ =3x+ 10. Find the equation of this second tangent. 4| Find the equation of the normal to the curve y = x — ® at the point where: x = 3, Find the coordinates of the other point where this normal intersects yox-$ Contextual 1 | The height, h metres, of an object thrown vertically upwards at time seconds after its release is given by h = 40t — 54°, a Calculate how long it takes for the object to return te its point of projection. b Find the value of t at which the object is momentarily stationary, and ‘hence calculate the maximum height reached by the abject. 2 | The displacement, s metres, of an object from some fixed point O at time t seconds is given by s = — 3° +4f4 5. a Find expressions for the velocity v and acceleration a at time . b Show that the object is never stationary. Calculate the average speed of the abject during the first three seconds. ‘5 Differentiation | [3] Actosed cylinder has a base radius of8cmanda = <——> height of 20cm. Calculate the small changes a inits volume and b init surface area that result from a small change of 0.1 cm in its radius. Leave your answers in terms of z and cm assume height remains constant. [4] A cuboidal box has height x, length 3x, and width 2x. Calculate the porcontage increase in the value of x ifthe volume is to increase by 4.5%. Urheberrechtlich geschatztes MSS Complete Advanced Level Mathematics: Pure Mathematics 5.4 The Chain Rule and Related Rates of Change Functions such as y = (x +1)° and y = (2x — 3)* are examples of composite functions. They are also known as ‘functions of a function’. Notice that y = (x + 4)' can be written as y =u" where u=x+1. One method of finding derivatives of composite functions is to expand the brackels and then differentiate term-by-term. For example, (x +1)? = (x+ 1) + 1)0e4 1) y = (x? 42x +1)(x +1) =x 43x? banda = 3(x? + 2x +1) = 3x41)? Try finding # (2x — 3)° in the same way. What happens when you differentiate term-by-term? Notice that the algebra gets quite complicated, Factorisation of such expressions, which we would need to do to locate stationary points, can be very tricky. This is a major disadvantage of this ‘expansion and term-by-term differentiation’ method. We also need an alternative method of differentiating composite functions such as y = (4 ~ x)? and y = Y3x +1, which cannot be expanded into a finite number of terms involving powers. Differentiating composite functions — the chain rule Suppose y = f(x) is a composite function of x. This means that y = f(u), where u(x) is some intermediate function of x that can be identified in the construction of function f. Any small change in the value of x, dx, gives rise to a small change in the value of u, du. This then gives rise to a small change in the value of y, dy. Differentiating from first principles, ‘ z) nt (Se asso LOX bbut since it is possible to write ae ir _ oy ox” du “ox ‘5 Differentiation | it follows that dy by Su tnt (SS «) However, du — 0 as dx — 0, and therefore dv, (%) ® == limit ( = } x limit de jo ou Ox ox ‘This gives du 4 Learn this important result. dx ‘This method of differentiating composite functions by introducing an intermediate variable uw is the chain rule. Example 1 Differentiate the following with respect to x, using the chain rule. a y=(x+1) b y=(x-3) Solution a Fory =(x+1)", lety =u", whereu =x41. Therefore Wy gue and. se 1, du a Using the chain rule eeu ux 1= 30" Substituting u = x + Basen! b Fory = (2x~3)°, let y =u where u = 2x -3, ut and a Using the chain rule, Oe WAU a Bui 2 = 10 Substituting u = 2x — 3, = = 10(2« — 3)* Example 2 ‘Use the chain rule to find the gradient function for each of the following: 1 a yep bo y=Vax-T Urheberrechilich geschiitzteggrpierial ‘Complete Advanced Level Mathematics: Pure Mathematics Solution Before using the chain rule, y = raised toa power. In this case, y = i (4x-5y. Now let y = u~!, where u = 4x —5. Then Y= Using the chain rule, dy_dy du 1, 4 de> du“ de uF <4 ae Substituting w= 4x—5,%=--—_4_, "dx (ax —5)? b Similarly, write y = V3x— Tas y = (3x — 1)". Now let y = u'/?, where u = ax oy Then Sh = Using the chain rule, dy_dy du Substituting u= 3x — 1, 3 "i “Te de du 2a 3 oat must be written as a bracket ‘When using the chain rule, the choice of the intermediate function u(x) is ‘very important. It is essential that u(x) can be easily differentiated. In general, any composite function of the form y = [{(x)]", involving some function f(x) raised to a rational power n, can be differentiated using the chain rule, ‘Let y =u", where u = f(x). gy yet Then Se = nu ‘Using the chain rule, dy ay du Urheberrechtiict Note that n[ f(x))"" the derivative of the bracket ignoring its contents, and f(x) is dorivative of the contents of the brack geschiitztes Material 5 Differentiation | Example 3 3 ‘Tho tangent to the curve y =~? atx = 2 erossas tho y-axis at Aand the xeaxis al B. ue a Find the equation of the tangent. b Find the coordinates of points A and B. © Show that the area of triangle OAB is !£ square units. Solution a Inordor ta find the equation of the tangent to the curve at x = 2, first calculate the gradient of the curve at this point, Use the chain rule (o differentiate y = ao suc', where u=1+2*. dy _ gu? and f4_ ‘Then SE = —Su"%, and 54 = 2%. Using the chain rule, _ 10x. Ge = dy _ Substituting a =1 +x, 52 = Now, when x = 2, dy_ 0x2 20 dx (97 2F 35 4 5 ‘The gradient of the curve, and therefore the gradient of the tangent drawn to the curve, at x = 2 is — 4. Use y —¥, = m(x — x) to find the equation of the tangent. So y-1=—4ix-2) 4 13 ya-$e 8 or Sy +4x-19=0 b Atpoint A, where the tangent crosses the y-axis,x = 0 => y = 2. ‘The coordinates of A are (0,12). Urheberrechtlich ‘Complete Advanced Level Mathematics: Pure Mathematics Al point B, where the tangent crosses the x-axis, yoos -f+Be0 > x-8 4 The coordinates of B are (4,0) Recall that the area of a triangle is given by area =! base x perpendicular height. So area of triangle OAB = $ x 13 x 1! = 1% square units. Related rates of change Consider how to calculate the rate at which the area af a circular oil slick, of uniform thickness, changes. Using the area of a circle, A= ar*, the rate of change of area with rospoct to the slick’s radius r, 4 can be found by differentiation. That is, f= 2xr. From aerial observation, or by more detailed mathematical modelling, it may be possible to determine the rate of change of the radius with respect, to time, #f. ‘The chain rule can now be used to link these two related rates of change together. The rate of change of the slick’s area with respect to time is given by Its important to remember that if a particular variable is increasing then its rate of change will be positive, and if it is decreasing, its rate of change will be negative. Unless specified otherwise, the phrase ‘rate of change’ refers to the rate at which a variable is changing with respect to time. Jrheberrechtlich geschiitztes Material '5 Differentiation | Example 4 A melallic cube, of side length vem, is being heated ina furnace. The side lengths are expanding at the rate of 0.1cms”", Find the rates at which the cube's Ic surface area and the cube’s volume are changing, when x = 10cm. Solution ‘The surface area of the cube, § = 6x7, Hence 4§ = 12x. ‘The volume of the cube, V = x7. Sod! = 3x*. 1 Since the side lengths are expanding at the rate of 0.1. cms", = 12x x 01=12xem?s! When x = tocm,¢8 t20m?s *, ‘Using the chain rule, the rate of change of volume, av _ dV dx ‘di de” dt =ax* x 0.1 = 0.3%? em's* When x = 10cm, SY = 30cm's*, Example 5 Air is being pumped into a spherical balloon at the rate of 300. cm’ s~". Find the rate at which the surface area of the balloon is increasing when the radius is 15¢m. Solution ‘The rate of change of volume, 4! = 300 cm! s Since the surface area of a sphere is $ = 4ar’, its rate of change with respect to radius ris 48 = 6xr. But it is not possible to link these two related rates of change together to form an expression for 4S, the rate at which the surface area is changing, with respect to lime. Instead, we must involve a third expression. Urheberrechilich geschitzs ‘Complete Advanced Level Mathematics: Pure Mathematics: Using the chain rule, aS aS iar gave at dr“ dv" de Since the volume of this spherical balloon is V = 424’, then wv. ane dr 1 and so 35 gag ‘The rate af change of surface area is then ds_ as dr av dt dr” dv" dt 1 Sar x oor 900 When r= 150m, 5 = soem? Example 6 ‘Water is emptying out of a 85 cm x 85cm. square-based cuboidal tank at the rate of 900 millilitres per second. Find, correct to 2. decimal places, the rate at which the height of the water is falling in the tank. Calculate how long it takes for the height to fall by 10cm. them Sok sscm~e ~ Om ‘900 ml st Solution av 0, The function is increasing. eet) ain) BE Wie =o e-em (3) sin(F) = 2 that is, Bea 4 The function is decreasing. ‘This means (7, v2) is a maximum point on the curve. Urheberrechilich geschitzte: 243°" ‘Complete Advanced Level Mathematics: Pure Mathematics: Considering the point (%,-/2): When x = cos —sinz=—1~0 That is, <0. 4 The function is decreasing. a3t dy (38) an (38) vinons= 2, Yoo (28) ~sn (28) =0-(-0 ‘That is, 2 >0. 4 The function is increasing, ‘This means (3%, ~ v2) is a minimum point on the curve. ‘The graph crosses the x-axis when y = sinx+cosx =O = lanx=—-1 an andx = a {in the stated range). y = sinx + cosx Check this result using a graphical calculator. Example 5 A Ferris whee! at an amusement park, with a radius of 10 m, is centred 11m above the ground. It completes a revolution every 20s, The height of a particular chair on the wheel varies according to. m xt at banca (2) +0cm (2). Find, to the nearest second, the times during the first revolution that this chair is rising, and the times during which it is falling. Solution When the chair is rising, height A is an increasing function of time ¢. This means that we need to find the times when { > 0. When the chair is falling, h is a decreasing function of time t. This means that we need to find the times when ‘ff < 0. 5 Differentiation | In order to decide when the chair is rising and falling, first locate and determine the nature of the stationary points on the height-time curve. ‘Using the chain rule to differentiate the sine and cosine terms, dh _ at x at x GF= 00s (#) xo sin (FE) xz _ar m) ani (at =e a0) 5" a0 At stationary points on the height-time curve 0 fo) ta) 0 10) 5 10 ang (FL) — 3% ge (at : $c () -Han(#) 927, 4.069, 7.210,... 95, 12,95, 22.95, ... (2 dap.) So during the first 20-second revolution, there are stationary points after 2.95 and 12.95 seconds. To find the nature of these stationary points, look at values of tho gradient function on each side. jeberrechilich ges chit ‘Complete Advanced Level Mathematics: Pure Mathematics ‘Considering the stationary point at f = 2.958: ai~som(5) ea (§) a5 As) 5 AS When t = That is, ah >is: 0. 4 The function is increasing. Hence, there is a minimum point when f = 12.95 s. Given that the wheel is centred 11 metres above the ground, and has a radius of 10 metres, the maximum and minimum heights of the chair (at these stationary points) are 21m and 1m respectively. When t = 0, h = 11 +8sin(0) + 6cos(0) = 17m. Since the wheel completes a revolution every 20 seconds, the height of the chair will be 17 metres whan f = 20. Use this information to sketch the height-time graph. From the graph to the nearest second, the chair is rising between 0 and 3. seconds, and between 13 and 20 seconds during its first revolution. This means it is falling between t = 3 and t = 13 seconds. ‘h(rmetees) seconds) as a ho weg 246 Urheberrechich 5 Differentiation | 5.5 Differentiation of Trigonometric Functions Exercise Technique Differentiate each of the following with respect to x: cos*(3x) sin(x? +5) y = 400s" § y =asin(px) + cos(qx), where a, b, p and qare constants Find the gradient of each of the following curves at the point indicated: ay =3cosx, wherex =3 by =sin®x, where x =} © y =Goos§, wherex =3n dy =x" —cosx, where x = j Find the coordinates of the points on each of the following curves at which the gradient has the value indicat a Gsinx, for 0 2, Then P(x) = (x — a)Q(x) +R, where Q(x) is the quotient and Ais the remainder. This identity is true for all x, and so it is true forx =a. When x = a, this becomes P(a) = (a —a}Qla) + R. Since (a — a) =O, the remainder & = P(a). Example 4 Use the remainder theorem to find the remainders when: a 3x" + 2x —5 is divided by x = 4 bx — 2x? — 3x41 is divided by 2x +1. Solution a Let P(x) = 3x" +2x-5. ‘The remainder when P(x) is divided by (x —4) is P(4). B= P(a) = 314) +214) -5 =4848-5=51 b Let P(x) =x" = 2x7 3x +1. ‘The remainder when P(x) is divided by (2x +1) is P(~ J). PH) = (- 9) - 2-38-49 Check these results using the long division -}-H}+1-8 algorithm (method). Unheberrechitich geschitzt@> Materia Complete Advanced Level Mathematics: Pure Mathematics Example 5 When 5x* + x — 8k is divided by (x — 1) the remainder is 2. Find k. Solution Let P(x) = 5x? +x — Bk. PQ) =5+1=8k =6 ~8k But P(1) = 8, where His the remainder when P(x) is divided by (x ~ 1). Since R = 2, it follows that 6 — 8k = 2 > Bk= 4 > k=} The factor theorem ‘The remainder theorem can also be used to provide a useful result for analysing higher order polynomials. If the remainder from dividing the polynomial P(x) by (x —a) is zero, then the linear term (x — a) must be a factor of the polynomial P(x). REG) ie tater of Px). Example 6 Factorise x7 + 2x" ~ 5x 6, Solution Let Pix) =x? + 2x? — 5x — 6, a cubic polynomial of order 3. To find the linear factors of P(x), if any exist, substitute values of x until P(x) = 0. Adopt a trial and error approach, substituting factors of the constant term, —6, in P(x). 4 2(1)* —5(1)-6 #0 + 2(2)* —5(2)- 6 =8+8-10-6=0 Since P(2) = 0, (x — 2) is a factor of P(x). P(3) =3° + 203)" - 5(3) -6 40 \* + 2(6)? —5(6)-6 40 P(=1) =(=1)' +2(=1)? = §(=1) - 6 = -14+2+5-6=0 Since P(—1) = 0, (x + 1) is a factor of P(x). P(-2) =(-2)' + 2(-2)? - 8(-2)-6 40 P(—3) =(—3)' + 2(-3) — 5(-8) -6 = -27+18+15-6=0 Since P(—3) = 0, (x +3) is alsoa factor of P(x). So P(x) = (x 2)(x4+1)(x+3) 4 Chock that (x ~ 2)(x +1)(x+3) =n + 2x7 - Sx-6. 6 Algebra It I may not be necessary to keep substituting values of x until all the factors have been found. Once one factor has been found the polynomial P(x) can be rewritten as P(x) = (x — a)Q(x), and the resulting quotient Q(x) may then be easier to factorise. Example 7 Factorise ax? + 2x? — 19x + 6. Solution Let P(x) = 3x" + 2x" — 19x 4 6. P(1) = 3(1)° +2(1)? - 19(1) +640 P(2) = 3(2)° + 2(2)* — 19(2) +6 = 244+ 8-38+6=0 Since P(2) = 0, (x — 2) isa factor of P(x). So P(x) = (x= 2)Q(x) ‘The quotient Q(x) could be found by using the long division algorithm or by using a technique known as algebraic juggling and then equating coefficients. The jugaling method uses both terms of the linear factor to find the quotient by comparing coefficients of the polynomials on each side of the equals sign as follows. Pix) = (x — 2)Q(x) => ax 42x? — 19¢+6 = (x — 2)(ax? +bx+e) = ax? + bx? + cx — 2ax* — 2bx - 2c = ax" + (b —2a)x* + (¢ — 2b)x — 2c Comparing coefficients of x°, a = 3. Comparing coefficients of x*, b — (2 x 3) Comparing constant terms, -2¢ = 6. Soc b-6=2.Sob=8. So 3x" + 2x? — 19y +6 = (x — 2)(3x* + 8x — 3) ‘The quotient can now be factorised by using either the factor theorem again or by another appropriate method. Since Q(x) is a quadratic it can be factorised using PAFF. Q(x) = 3x7 + Ox —3 IRS! z cue Pis the constant term P:3x (=3)=-9 AL+8 F:9,<1 multiplied by the Q(x) = 3x7 + By —3 = ax + OX —K-—F coefficient of x*, Ais the _ i number of ‘x’s and F = x(x + 3)— 166 +3) values add to A and = (x +3)(3x 1) multiply to P. PEE aria Urheberrechtlich geschatzi4 ‘Complete Advanced Level Mathematics: Pure Mathematics ‘Having factorised Q(x) the original polynomial P(x) can be written in a factorised form as P(x) = ax" + 2x" = 19x +6 = (x-2)Qx) (x ~ 2)(x + 3)(ax — 1) ‘Writing cubic polynomials in their factorised form is particularly useful if the graph needs to be sketched or related inequalities solved. Example 8& Sketch the graph of y — x? ~ 2x — 13x ~ 10. Hence, or otherwise, state tho rango of values of x for which y > 0. ‘Solution Use the factor theorem first to show that, y =x" — ty? — 18x — 10 = (x + 2)(x + 1)(x — 5). ‘So y = 0 when.x = —2,—1, 5; the graph cuts the x-axis at (-2,0), (-1,0) and (5,0). ‘The graph cuts the y-axis when x = Oat the point (0, —10). Given the general shape of the cubic graph and the four points through which it must pass a sketch canbe made, Check this result using a graphical calculator. ‘The values of x for which y > 0 can be read from the graph; y > 0 when ~25. v1 yest 19%-10 ‘The remainder and factor theorems can also be used to find the value of unknown coefficients in a polynomial, This often creates simullancous equations. 6 Algebra i Example 9 The polynomial 2x? + ax? + bx +8 has (x — 1) as a factor and gives a remainder of 50 when divided by (x — 3). Find a and b. Solution Let P(x) = 2x" + ax? + bx +8 Since (x — 1) is.a factor, P(1) = 0. P(l)=2+a+b+8=0 a+b=-10 fl When P(x) is divided by (x — 3) there is a remainder of 50. This means P(3) = 50. (P(3) = 2.3° 40.37 +b3+8=50 54+90+36+8=50 9a + 3b = 12 (2) Equations (1] and [2] are simultaneous equations. Using the elimination technique we can find a. From equation [2], 9a + 3b = -12 3 x equation [1] 3a +3b = —30 Subtracting, 6a = 18 = =3 Puta =3 in equation [1] 3.4+b= —10 > b==13 So P(x) = 2x7 + 3x? — 13x + 8, witha = 3 and b = ~13. Urheberrechtlich geschttzte@e@erial Complete Advanced Level Mathematics: Pure Mathematics. 6.1 Polynomial Division 264 Exercise Technique Use the long division algorithm to find: x4 dx? 4 Sx 49 ata =x? -9x-6 rd x +27 -x-10 ‘Use the remainder theorem to find the remainder in the following: is 4+ 2x-5 x-2 co MtaK=3 2x +1 ? x-2 a 2x? -27 +4x4+15 +3 b 3x4 42x42 x41 q We teK-2 2-3 Factorise completely the following polynomials: a ax ay 44 b ox— 6x? +11x-6 ca x44 Solve the following inequalities: a x°— 12x74 39x — 28 >0 bo xt- 10x? + 11x +70 <0 © x—6x?-13x442<0 Contextual Let f(x) = a Factorise f(x) completely. (41) and (x — 2) are both factors of x’ + ax* + bx — 6. Find a, b and the third linear factor, dx) 45x — 8x —12 @ ax + 19x? + 22x48 fo ax +5x7+x-2 dx? -30x 30 x4xt— 12x <0 fax! —7x*— 46x -21>0 2x! — 12x" + 40x — 32, b When is f(x) > 0? Find the coordinates of the points whero y = x’ — 3x? — 16x — 12 crossos. the x-axis. When x* + ax* + bx ~ 1 is divided by (x — 2) and (x + 2) the remainders are 27 and 3 respectively. Find a and b. Solve the equation 6x? = 7x? = 9x = 2=0. f(x) <0. [G] sr /¢x) = 4x? — 1952 + 19% + 6, find the range of values of x for which Hint: First write the expression as the product of three linear factors. 6 Algebra tl 6.2 Algebraic Fractions An algebraic fraction is a fraction that contains an algebraic term. Some examples are x 2 3 6t+7 3’ oy’ ax¢1' P+3I-1 Algebraic fractions can be combined in the same way as numerical fractions, and obey the same rules for addition, subtraction, multiplication and division. Addition and subtraction Find } +4. Check that you get a result of }!. How did you arrive at this: result? When adding fractions the first step is to find a common denominator. This is usually the least (or lowest) common multiple (LCM) of the denominators of the fractions being added. Having found a common denominator each fraction is then written in an equivalent form using this new denominator. The numerators can then be added to give the solution tothe problem. aie iis dtaet ‘The same technique can be used to add and subtract algebraic fractions. Example 1 2 3 10 5 a Find Sy ters) b Find Ga)" Sa) Solution a The common denominator will be (x ~3)(x +5). Notice that this is the LCM of (x ~ 3) and (x +5). 204, 3 2(x+5) a(x —3) (3) (+5) &—3)K+5) | (X= 3)R+5) _ 2k +104 ax-9 ~ e848) _ Set ~ = 3)(K+5) b The common denominator will be (2x ~ 1)(x +3). 10 5 10(x +3) — 5(2x —1) eat) FI Bears) _ 10x+30 10x45 © (R= 13) _ 35 ~ Ded Complete Advanced Level Mathematics: Pure Mathematics Example 2 7 js 2 3 2x-1 Express asa single fraction, —— +} Ta ay Solution Again notice that both algebraic fractions are proper. Notice also that the quadratic expression in the denominator will not factorise. The same technique can be used, even though one of the denominators is quadratic and not linear. 3 wat _ 3(x? + ax + 1) + (2x — 1)(x +2) (e+2)) (44x41) (+ 2)(e? + 4x4 1) _ (ax? + 12 +3) + (2x? + 4x —x 2) - (e+ 2) + 4x +1) _ __Bxh 4 15x41 ~ (x F 2)? + ax + 1) Example 3 3 1 . Express 5 ~ Gy ayes # single fraction. Solution Notice that the term (x + 2) appears in both denominators and that the LCM of (x +2) and (x +2)? is (x + 2)*. This means the common denominator will also be (x +2)*. 3 1 _ 3(x +2) x42 (xt2) (esa) (w+2P _ 3x +2)-1 7 ray = 3K+6-1_ 3x45 “(e+ 2)F (+2) Multiplication and division Find 3 x §, Check that you ean get a result of $, How did you arrive at this result? When multiplying fractions the first step is to find if there is a common factor in the numerator and the denominator. If a common factor does exist, it can be ‘cancelled down’. Then multiply the numerators and denominators separately to give the numerator and denominator of the result. Gand 3 have a comn factor 3, so it can be ‘cancelled’. 6 Algebra Il Example 4 a Simplify 28), 2(x +3) 1 b Express as a single fraction, = c Simplify 7 Solution 7 arts) ay 2(x +3) 1 _ 2(x+3) 1 te 8 “a a)OrS) Tae = Berm 5 (x= 3) 3) 4 __2 * 50x — 3) Sx-12_Sx__ S(x— 4), _Sx 10" 2x-B 10 (x4) acai) Bk 216 th ‘The division of fractions can be tackled by reducing the problem to multiplication, Remember that to divide fractions, invert the second fraction and change the sign from division to multiplication. For example, }+7= =}. The same technique can be applied to algebraic fractions. Example 5 a ‘Simply Sie, bh Express as a single fraction Urheberrec! Complate Advanced Level Mathematics: Pure Mathematics 62 A 3x-6 , 5x’-5y-10_ 3(x~-2) = e=2), 6x “ax” B(x 2) 1) — 3a, 38e ~ 48h OBR) -_# ~ 20(x+1) pie eae, _ Me 2)e +1) si r a ax ant ic 4 Factorising, I 6 Algebra It 6.2 Algebraic Fractions Exercise Technique Find: ax 4 following as a single algebraic fraction: 2x41 2x43 ett gp aS 8 yx 12” Hye —2) x 3 D peyarca xe e © +74 f XH xi 42x41 Identify the LCM of these denominators and use it to write each of the Express each of the following as a single fraction: 3x-2° 12x 3 2 xt-0x-3 3 X4x-2 Ox-1 sate Bert 2) (x +2) _ 14“ @—2e-3) rechtlich ges chiitzte Complete Advanced Level Mathematics: Pure Mathematics 270 6.3 Partial Fractions ‘Two or more proper fractions can be combined to give a single fraction. For example 1 +2 = 11. Equally, a fraction can be expressed as a sum or difference of two or more proper fractions, known as partial fractions. We ean usually apply this technique to algebraic fractions, and it is sometimes quite advantageous to write an algebraic fraction as a sum or difference of simpler algebraic fractions (see Chapter 12), The key to this process lies in the factorisation of the denominator. We will consider three categories of denominators: © lincar factors in the denominator @ = quadratic factors in the denominator © repeated factors in the denominator, Linear factors in the denominator Bx Look at the fraction Fay ‘The denominator has been factorised into two linear factors, (x — 3) and (x +5). This suggests that this algebraic fraction could be rewritten as Sx+a A B (x= 3)(x+ 5) ~ (x3) © (x +5)" where A and Bare constants. Checking back to Section 6.2, Example 1a, you will find that A = 2 and B = 3 provide the solution. Bx+1 3 (&=3)e+5) (+5) Two techniques ean be used to find the constants that form the numerators of the partial fractions. The first uses the following steps: Step @ Identify the linear factors in the denominator. Step @ Write each linear factor as a new denominator with a constant term numerator. Step @ Add the algebraic fractions together and then equate the two numerators. Step @ Substitute values of x that make the coefficients of A and B zero, in turn, and solve the resulting equation, Example 1 Sx+7 Write ayaa) in partial fractions, 6 Algebra I Solution Notice that the denominator has been factorised into two linear factors, (v4 3) and (x — 1). This suggests that A B 4. @ Write each linear factor as (e+ -1) a new fraction, Bx+7 (x +3) 1) Now add together the algebraic fractions on the RHS of the identity, Recall that RHS means — Set? _Ale=1)+ BR +3) Add the fractions. (a+ 3)e— 1) (x + 3)(x—1) Since the denominators on both sides are equal it follows that the two numerators must be equivalent to each other. So Sx+7=A(x—1)+B(x+3) << @ Equate the numerators. ‘This statement is true for all values of x, so A can be found by substituting values of x that reduce the coefficient of 8 to zero, and vice versa. Whenx=1, 5+7=A(0)+(4) << @ Substitute values of x. > 12=4B > B=%: Whenx=-3, -15+7=A(-3—1)+B(0) > Solution ‘The denominator of this fraction is a quadratic. Since it has not been’ written as a product of linear factors the first step is to factorise the denominator. Check that x7 — x —2 = (x +1)(x—2). < @ Identify the linear factors. a a xe —x-2 > RF TR—2) Now the denominator contains two distinct linear factors. 3 | B <4 @ Use each linear factor to 7 1@e-2) +I) &%—2) —_—_write down a new fraction. Urheberrechtlich geschiitzte™ Merial Complete Advanced Level Mathematics: Pure Mathematics Adding together the algebraic partial fractions, 9 A= 2) + Be + 2) ‘i EDELH = Sawa tDAddthe fractions. Equating the numerators, BSAlx-2)+B(x+1) 4D Equate the numerators, Whenx=2, 3=A(0)+B(3) << @ Substitute values of x. 3 1B B= , 3 =A(-1—2) +800) =>3=-3A >A=- got ge wox-2> (x41) 2° (x=2) ‘Step @ provides the second method of finding partial fractions. It is known as the ‘cover-up’ rule. The cover-up rule uses the fact that some values of x create a zero in the denominator. From above, we have seen that these values are also the ones that allow us to calculate the required numerators in the partial fractions. By covering-up these factors the whole process can be speeded up, and can often be calculated mentally. Example 3 5x +7 Use the cover-up male to express (37, —3y in partial fractions. Solution Fs Sx +7 Look at the fraction wen@— Notice that (x +3) = 0 when x Cover up (x +3) and substitute x = ~3 in the parts of the fraction you can still see. Sit Jie ne ‘The solution 2 is fou Ge -1) 4-4 ‘when (248) is covered up. Also notice that (x 1) =0 when.x = 1. Cover up (x — 1) and substitute x = 1 in the parts of the fraction you can still see. The solution 3 is fou ee ee when (¢ — 1) is + 4 4 covered up. 272 6 Algebra! Sx +7 __2 + 3 (+3)e—-1) @&+3) 1) Notice how the number found when ‘covering-up' becomes the numerator above that ‘covered linear factor. This means the cover-up Example 4 Bxt+9x-14 | 2 " Express 5 ayy ay pay i Pata factions. Solution Notice that the denominator has three linear factors. Using the cover-up rule this can be put into partial fractions in three steps, Covering up (x - 2) and substituting x = 2, Sxi+3x-14 _20+6-14_12_ 4 GBBe-De+) xs 3 Covering up (x ~ 1) and substituting x = 1, Covering up (x + 1) and substituting x = —1, Sx? + 3x —14 5-93-14 -12 ®-50- eo =3x-2° 6 Sxt¢3x-14 4 spe 2 @=2@-De+4)~ W271) FD So Quadratic factors in the denominators Some algebraic fractions have denominators that contain a quadratic 8x" 4 5x +6 (FD 3) Notice that the denominator has a linear factor (x +1), and a quadratic factor (x* + 3), which doesn't factorise. What happens when fractions of this type are written in partial fractions? ‘The linear factor has a constant numerator, A, and the quadratic factor has a linear numeratar of the form (Bx +C). factor that doesn't factorise; for example, Example 5 Bx +5x+6 | i Express (-1)6 pa) Partial fractions. Complete Advanced Level Mathematics: Pure Mathematics 274 Solution This fraction can be written as the sum of two distinct fractions; one with (x +1) as its denominator and the other with (x7 + 3) as its denominator. ‘The latter fraction has a linear numerator. 3x7 4+ 5x +6 A Bx+C @&+ Det +3) 41) GF +3) Now add together the algebraic fractions on the RHS. Bt 45x46 _ Afx? +3) + (Bx + Cie +1) (xt lat +3) & +O +3) Since the denominators on both sides are equal the two numerators must be equivalent. ax? 4 Bx + = A(x? 43) + (Bx + €)(x +1) A, Band C can now be found from a combination of: © substituting suitable values of x © equating covfficients of powers of x. Letx =—1 ‘Then a(-1) +5(-1) +6 =Al(-1)? +3] + (-B+C)(0) = 3-5+6=4A+0 > 4=4A = A=1 Letx =0 ‘Then 3(0)? + 5(0) +6 = A(0* +3) +C(041) > G=3A+C Substituting A= 1,6 =3+C > c=3 Returning to the equivalence between the numerators, ax? + 5x + = A(x? +3) + (Br + C)(x +1) = Ax? +9A + Bx? + Bx + Cx + 0 = (A+ By? + (B+ Cx + (3A +0) Urheberrechilich geschiit es Material 6 Algebra Equating the coefficients of the different powers of x: x 3=A+B Lu] x: 5=B+C 2 constant terms: 6 = 3A+G [3] We know that A = 1 and C = 3, so the value of B can be established from equation [1] and checked in equation [2], or vice versa. From [1,8 =1+8 = In(2],5=B+G=2+3 Having found A = 1, B = 2 and C = 3, the original fraction can be rewritten in terms of partial fractions as: axt+8x4+6 1 2x43 e+ ie? +a) +a)’ +a) Example 6 Bx? + 2x43 : Express (yet pay it Pattial fractions. Solution at 42x43 A Bx+C ret +3) +a G43) _ Als? +3) + (Be + C(x +1) te Fiat +3) Equating the numerators, 3x* 4 2x +3 S A(R? +3) + (Bx + CV +1) Whenx=—1, 4 Substitute suitable values of x. 3(-1)? +2(-1) +3 =Al(-1)" +3] => 3-24+3=4A > 4=44 + A=1 ‘When x = 0, 30)? + 2(0) +3 = A(0? +3) + C(04 1) = 3=3A4C 4 From above working, A=1. chee RBBor9 Urheberrechtlich a ‘Complete Advanced Level Mathematics: Pure Mathematics Equating the coefficients of the powers of x; x: 3=A+B x: 2=B+0 constants: 3=3A+C fa] (2) (3) ‘We know A= 1 and C =0, so equation [4] or [2] can be used to find B. From (1), S=A+B a=14+B B=2 4 Check using equation [2]. ax? ¢ox43 50 eater +3) ax wry 4 Check this result. Repeated factors in the denominator Some algebraic fractions have denominators that contain repeated factors: ax+5 Sx +15 wrap pera” What happens when fractions of this type are written in partial fractions? for example, Example 7 3x+5 Express (i in partial fractions. Solution wx+5 A,B +2) 42) @et2e Natice that the partial fractions have denominators (x + 2) and (x + 2)’. ‘The two numerators are assumed to be constants. Adding the algebraic fractions gives SN+S _ Ale +2) +B (w+2yh (x +2) The two numerators can now be equated since the denominators of these two fractions are equivalent. 3x +5 SAN +2) +B Since this statement is true for all values of x substitutions can be made. constants: 5 = 2A + Chock that this gives same solution, geschiitztes Material 6 Algebra When x=—2, 3(-2)+5=B > -64+5=8 B=-1 Whenx=0, — 3(0)+5=A(0+2)+B = 5=2A+B 5=2A-1 2A=6 A=3 So a (et2)? #2) 0 (et 2" +2) (es 2) Example 8 S5x+15 Express eer in partial fractions. Solution Notice that the denominator has a linear factor (x — 1) and a repeated factor (x +4). This means the fraction will split into three partial fractions, with denominators (x — 1), (x + 4) and (x +4). Sx +15 A B c @-pa+a &-t) &s GO rayP ‘Adding togethor the algebraic fractions with « common denominator of (x-1)(e+4)), SR415 —_ A(t 4)? + B(x —1)(x +4) + C(x 1) (x-A)(x+4y (x-i)(x +4)? Equating the numerators, Sx +15 =A(w + 4)? + Bix — 1)(x +4) + C(x 1) When x= 1, (1) +15 =A(1 +4)? <4 Substitute suitable = 5+15=254 welee ote, 20 = A=a 4 - a=} Whenx=—4, 5(-4) +15 = C(-4~1) => -20+15=-5C > -5=-5C = c=1 Complete Advanced Level Mathematics: Pure Mathematics To determine B, either substitute another value of x or equate coefficients of some power of x. Equating coefficients is usually the quicker method. So, equating coefficients of x*, O=A+B We know A=$, 505 Sx 415 _ (x ~ 1)(x +4) Improper fractions These techniques can be extended to improper algebraic fractions. Remember that fractions are improper when the numerator is of a degree equal to, or higher than, the denominator. In these cases polynomial long division must be used first. Example 9 Write the improper fraction wea in partial fractions. Solution Notice that the numerator is a polynomial of degree 3 and the denominator is a quadratic in factorised form; the fraction is improper. (x-2)e+8) =a? +x-6 Now rewrite the problem in the usual long division format. The long division algorithm gives ax- 5 x px 6)5x + Ox + Ox +15 sxt45x*- 30x | = 5x7 + 30K +15, 5x +30 35x —15 Bx" +15 38x — 15 Sate EE epee OE &-20+3) 5+ yaar) ‘We can now write the proper algebraic fraction in terms of partial fractions, in the same way as before. 35x —15 A B (= 2)(8 +3) ~ &= 2)" &F3) 278 rheberrechtlict ‘Try equating cooffici ‘of x or constant term: check whether you g the same result. Chock this result | ing the ‘The quotient is (5x—¢ and the remainder is 35x—15, 6 Algebra I! Since the denominators are all linear we can use the cover-up rule. Covering up (x — 2) and putting x = 2, 35x-15 70-15 _ Covering up (x + 3) and putting x = —3, 35x—15 _ -105—15 =24 SoA = 11 and B= 24. Sxi+15 u 24 Dar OT e2) ED The partial fractions techniques can be summarised as follows: A useful rule to remember is that the number of canstants (A, B, C....) needed when first writing down the partial fractions is equal ta the order of the denominator of the original proper fraction. For example, (x —1)(x +4)? is of order 3, so three constants, A, B and C, are needed. Urheberrechtlich geschitt Complete Advanced Level Mathematics: Pure Mathematics 6.3 Partial Fractions Exercise Technique [1] Express the following in partial fractions (denominators with linear factors): 6 a 2(3x +5) (x 5)[x+ 1) (x=1)(«+3) 2x ee kk (x=1)(x+3) x =3x42 Sx +29 £ 5x -17 (&—4)(x +3) 7x4 12 [2] express the following in partial fractions (denominators with quadratic factors): ax? — 2x47 d ax? 45x49 &— 20 +1) wee tees b 5x? — 2x 15 3x? 4 20x43 xe +a) © 2) + ax Fa) 2(x? + 2x —4) t ax? + 5x +14 w= 3) +2) wre ax ea [3B] express the following in partial fractions (denominators with a repeated factor): a et? 2(ax +3) qr (+3) b x+1 a 4 (2)? ts Fi 18x-9 (+5) (x+2)(x— 1)? 4 | express these improper algebraic fractions as partial fractions: 2x? — x? — 18x - 22 a 1ox* +20 @—4)e +2) &—3)0 +1) xt +15, xia ax @2)a +3) * @=2e+ ax? +12 p bart +5x~5 (x = 1)(# + 2) (x +1)? Express the following in partial fractions: Bx? 4 2x —41 4 ant 4 Bx +6 &+ie—3R—4) @ Fie +3) ay? = ax +32 X4+1ixe4 = He = 20-7) ° @opwrD ax? + 16x ~ 60 t att —2 (FI) = 25) 2 ED) 6 Algebra! 6.4 Curve Sketching ‘Curve-sketching’ means producing an outline graph that shows the general behaviour ofa function or polynomial. It is not necessary to draw a table of values and plot points accurately. The sketch should show the main features, Step @ Find where the graph crosses or intercepts the axes. Step @ Identify any stationary points (Local maxima, minima or stationary points of inflexion). Step @ Find out what happens for large positive and negative values of the variable (the behaviour as x tends to +90 and —20). Step @ Identify any values for which the function is undefined (or discontinuous). Example 1 Sketch the curve y = 2x? + 8x +6, Solution The main features of this curve are identified separately and then a sketch is produced combining them all. @ = The graph crosses the y-axis when x= 0. <4 (@ Intercepts. x=0 => y=2(0)? +8(0)+6=6 So the graph passes through (0,6), ‘The graph crosses the x-axis when y= 0. ¥=0 > O=2x + 8x46 => a(x’ +4x+3)=0 => 2(x+3)(x+1)=0 => (x+3)=0 or (x+1)=0 = x=-3 or x=-1 So the graph passes through (~3, 0) and (~1,0). @ Stationary points occur when$¢=0, 4 @ Stationary points. yoax' +6x+6 > W acto Wao > 4x48=0 3 x=-2 When x = -2, y = 2(-2)° +8(-2) +6 = 281 Complete Advanced Level Mathematics: Pure Mathematics So there is a stationary point at (—2, —2). Using the techniques from Chapter 5, verify thal it isa minimum point. Remember to look at gradient of the curve either side of x = —2, © Asxgels large (x — +00),,» also gots large (y —- +50). € @ Lange x. Similarly as x + —o0, y > +20 because of the x term. © Tho function is dafined for all values of x, so the graph should be a continuous curve. << @ Continuity. Now combine all four findings into one graph. Verify that the graph of y= 2x% + 8x + 6 has this form using a graphical calculator. yaatearse ‘mlniuae polqt at (22,2) Example 2 Sketch the curve y = x? + 2x? —x — 2, Solution The graph crosses the y-axis when x = 0. @ «=0 + y= (0) +20)? -(0) -2 4D Intercepts. ‘The graph passes through (0,—2). ‘The graph crosses the x-axis when y = 0. y=0 3 420 -x-2=0 ‘This cubic equation can be solved using the factor theorem. Let P(x) =x + 2x* -x-2 Check that P(—2), P(—1) and P(1) are all zero, This means (x + 2), (x + 1) and (x — 1) are all factors of P(x). This means the graph crosses the x-axis al three distinct points: ‘Check that (=2,0), (=1,0) and (1,0). P(x) =(x+2)(x+1)(x Urheberrechilich geschitztes Material 6 Algebralll @ Stationary points can be found from$=0, 4 @ Stationary points. My =x! +2x* —x —2 then $¢ = 3x? + 4-1 SofE=0 = 3x'+4x-1=0 ‘This particular quadratic equation cannot be solved by factorising, so ‘use ‘completing the square” or the ‘quadratic formula’ instead. Verify that the solutions are x = 4(-2+ V7). ‘This means there are two turning points. The approximate coordinates are (~1.55,0.63) and (0,22, -2.11), Check that these. turning points are a maximum and a minimum point, respectively, by looking at the values of $f on either side of each point. @ Asx — ce, y — ov because of the behaviour of the x" term. Similarly, as x—+ co, y = =00. <4 D Large x. @ The graph will be defined for all values as this polynomial can be evaluated for all values of x. << @ Continuity, Combine all four findings into one graph. Graphs of discontinuous functions Discontinuous functions have graphs that get closer and closer to some straight line and yet never cross it. This straight line is called an Look at the graph asymptote. This type of graph has been met already. ofy=tandin Chapter 3. Recall that tan @ is undefined for a sequence of values of 0. Urheborrechlich geschitate® Ste Complete Advanced Level Mathematics: Pure Mathematics 7 t ' ' 1 1 t What is special about these values of 0? Recall that tang = 2207 cos For these values of 0, cos 0 = 0. The rational function that then defines and has a denominator of zero. This observation indicates a useful technique to adopt when sketching rational functions. Identify those values that produce a zero denominator. Vertical asymptotes will be located at those values. Example 3 __ a Sketch the graph of y= wa Solution @ xv=05¥ << D Intercepts. ‘The graph crosses the y-axis at (0,— 3). =050--3.. a “@=3) Since there is no value of x for which y = 0 the graph does not crass the x-axis, So the line y =0 (the x-axis} must be an asymptote. = 3 al 3 <2 Stationary © rg 7 ae ea points. Since {¢ ¥ 0 for all values of x there are no stationary points. @ Asx +0," 40" 4 @ Large x. i Asx = -s0.y +0 above (positive) and O° means x — 0 fron @ The function jl is undefined when.x ~2=0 -¢ @) Continuity. ee eas nw (negative), So the graph is discontinuous when x = 2. Since x =2 isan asymptote the behaviour of y either side of this value should be checked. 3 aigt 70h aap A882 (hom above) x2 — 0°, sy = 5 Asx— 2° (from below), x 2 + 0°,s0y — ~c0, 00. Urheberrechilich geschitztes M 6 Algebra I {All of these features can now be cambined to give the graph of f yomasre2t yh a cad yoeOrasx te = Ys0ns Efpeeane Example 4 Sketch the graph of y a Sojution Look closely at the algebraic fraction. Spot that it is improper and rewrite it in quotiont/remainder form. x+1 re 4 Vorify that y = == ean bo written as y = 1-5. @ Putting x = 0 gives y = }. So (0, !) is the intercept on the y-axis. Putting y = 0 gives x = —1. So the graph crosses the x-axis at (—1,0). oy 4. Since © Vorify that Gy = Gaye Since Fy # O forall values of x, there aro no stationary points. @ Asx — t00,y + 1,s0y =1 isa horizontal asymptote. @ = The graph is not defined for x = —5. When x = —5 the denominator is zero. Sox = —5 isa vertical asymptote. Those can now be combined to give the graph. Example 5 Sketch the graph of y = x42 a2. ‘Complete Advanced Level Mathematics: Pure Mathematics Solution Look closely at the algebraic fraction. Spot that it is improper. Writing the equation Check that y = +2 eanbo witon asy = + 1 + nerent oe ae The quotient is now (x +1) and contains the variable x. @ Tho graph crosses the y-axis when x = 0 at (0,=2). <€ @ Intercepts. The graph does not cross the x-axis. Why? Ify = 0 then the numerator x? 42 =0, and this equation has no real solutions, 3 42 Stationary tha points. = 3 @ My=xt+1+5 “Ea For stationary values, 32 = 0. - a. - -1= 1 ato ews > x-l=dv3 > x=1tv9 Check that the stationary point at x = 1+ v3 is aminimum point and that the stationary point at x = 1— /3 is a maximum point by looking at the value of the gradient of the curve on either side. © Asx becomes very large (both positive and negative), aa becomes very small, and tends towards zero. Therefore y — x + 1 as x —+ -too. Soy =x +1 is a skew, or slant asymptote. @ Tho graph is not defined for x = 1. Whenx = 1 the original denominator is zor0. So x = 1 is a vertical asymptote. 286 Urheberrechilich geschotztes Material 6 Algebralll Modulus graphs ‘The modulus of a function, | f(:s)|, was defined in Chapter 4. To sketch graphs of the modulus of a function, first sketch the graph of the function f(x), and then reflect in the x-axis all parts of the graph for which the y-coordinate is negative. Example 6 Sketch the graphs of: a pal? b ya feet eye 2 Oe Yo lees 7 lea Solution Compare these with the graphs of the functions found in Examples 3-5. All that is required now is to reflect the parts of the graph that are below the x-axis (where y-coordinales are zero) in the x-axis. Sketching a modulus graph is a very useful technique when we have to solve equations or inequalities that involve the modulus function. Urheberrechilich geschitzte@ Beer ‘Complete Advanced Level Mathematics: Pure Mathematics Example 7 Solve the equation |x| = |8x — 24]. Hence, or otherwise, solve |x|<|8x — 24). Solution To salve the equation first sketch the graphs of y = [x| and y = [8x ~ 24] on the same axes. Notice that we got two distinct V-shaped graphs. The graph of y= |8x ~ 24] is steoper and touches the x-axis atx = 3. ‘The points of intersection of the two graphs are the solutions to |x| = [ax — 24]. These can be found algebraically by solving two sets of equations: x= 6x—24 and x =~(8x—24) The first equation corresponds to the point where y = x intersects y = 8x ~ 24, The second equation gives the solution created by the reflection in the x-axis. This corresponds to the point where y =x intersects y = —(8x — 24). Solving these equations gives x= ax— 24 = Kaa xn —(8x-24) = 9x=24 = 4 satisfy the equ x= |x| = |ax — 24. Verify this result on ‘To solve the inequality |x| < |x — 24), use the graph. Notice that the graph graphical calculator. of y = |x| is below the graph of y = |ax — 24| when x < 28 orx > 24. So |x| < [ax — 24] when x < 4andx > 2 When we need to solve inequalities that involve the modulus function it is often useful to check the graphs. We can usually identify where the required condition is, geometrically, by finding where a graph is either above or below another graph or axis. 6 Algebra it 6.4 Curve Sketching Exercise Technique | 11| Sketch the graphs of the following continuous functions: a f(x) = 2x7 47x? — 5x —4 b h(x) = |x? - 6x — 7] ¢ g(x)=x'— 19x +30 ds h(x) = [2x" + 15x" + 31x +12) 2 | sketch the graphs of the following rational functions: z b f(x) =— a ae) 5 © au=z d no) = || 3 | Sketch the graphs of the following rational functions: xt 3 x-3 ce Agy=S a fx) [4 | solve the following: a [xj=|5e—40) bef =f12—ax] efx 2] = [6 — 2 d |y| < [8x — 40} e fx| > [1 Sa] £ [ax +4 > [5 — 3x] Contextual 1 | Given that s(x) = 3x9 = 7x? — 7x 43: ‘@ factorise f(x) and then sketch ils graph b onaseparate diagram sketch the graph of |/(x)|- 2 | Given that f(x) =2x +1 and g(x) == +2, find the range of values of x for which: 4) a fixj>o b gix)>0 © Fe 3] sketch the graph of y = 7°. Hence or otherwise find the range of values of x for which y <0 4 | On separate diagrams sketch the graphs of: 2x-6 x+8 aye b y= Ea Qo ‘Complete Advanced Level Mathematics: Pure Mathematics Consolidation Exercise A f] Use the factor theorem to find a linear factor of x" — x — 6, Hence express x? — x —6 as the product of a linear factor and a quadratic factor. (OCSEB) [2] Itis given that g(x) = (2x — 1)(x +2)(x—- 3). a Express g(x) in the form Ax? + Bx* + Cx + D, giving the values of the constants A, B, C and D. b Find the value of the constant a, given that (x + 3) isa factor of g(x) tax. c Express “Gay i arta rations d Solve the inequality g(x) > 3x(x + 2)(x — 3), (UGLES) [3] a When divided by (x — 3) the expression x* + ax* ~ (a* ~ 3a + 17)x gives a remainder of 3. Find the value of a. b Find the three linear factors of x — x* — 75x + 25. [4] The polynomial p(x) = x* + cx? + 7x +d has a factor of (x +2), and leaves a remainder of 3 when divided by (x ~ 1). a Determine the value of each of the constants ¢ and d. b Find the exact values of the three roots of the equation p(x) = 0, (AEB) [5] Given tnat &—" fs = A+ B4= £42 40, find the values of A, B,C and D. (OCSEB) ax+ B Cc [6] Given that y = Se wee express y in the form 4 eto eo (NEAB) ‘The cubic polynomial x? - 2x? —x — 6 is denoted by f(x). Show that (x= 8) isa factor of f(x). Factorise f(x). Hence find the number of real roots of the equation f(x) = 0, justifying your answer, Hence write down the number of points of intersection of the graphs with equations y = x* — 2x — 1 and y = & justifying your answer. (UGLES) [= a c Sketch the curve y = Given f(x) a b 6 Algebra I {8 | tfpte) = 2x? — 5x? — 20x + 15: factorise p(x) b solve p(x) =0 sketch the graph of p(x) 4 solve p(x) > 0. x x sketch the graph of f(x) ona separate diagram sketch the graph of |/(x)]. Exercise B a When the cubic expression x* + ax’ = (2° + 12)x + (7a + 10) is b A polynomial f(x) can be expressed in the form p(x + 2)* +q where p and q are constants, When f(x) is divided by (x — 1) the remainder is 40 divided by (x — 1) the remainder is 7. Find a. Find the three linear factors of x* + 2x" — 20x +24. and when f(x) is divided by (x +3) the remainder is zero, Find the values of p and q. Given f(x) = (3x —2)(x —1)( +1): a b © ‘The cubic polynomial f(x) = 5x? + px? express f(x) in the form Ax* + Bx? + Cx+D x+1, . ey partial fractions solve the inequality f(x) > 2x(x —1)(x +1). when divided by (x — 1) and also has an exact factor (x — 3). b Express the cubic polynomial as the product of three linear factors. Find the values of p and q. With the aid of a sketch, solve f(x) < 0. Sx+4 Accurve has equationy = Say ai “Eiggpans Ga in partial fractions. b 2 = (2x 1)* (x= 2) that the curve has a turning point when x = —3. Determine the value of x at the other stationary point of the curve. Show that 2 ‘11x + has a remainder of —12 —4__ and hence, or otherwise, show + : Now find the values of x for which y > 0. (WJEC) (NEAB) (AEB) Urheberrechtlich ges 291, Complete Advanced Level Mathematics: Pure Mathematics Applications and Activities Parametric equations Sometimes, instead of having an equation relating x and y directly, y =f(x),x andy are written in terms of a third variable, called a parameter, Then x and y are represented by two separate functions, x =g() and y = h(t), called parametric equations. Eliminating the parameter, or variable, reduces the two parametric equations into a Cartesian form with two variables. Parametric equations are a useful technique to uso with graphical calculators, when it is difficult to write y as a function of x. Use a graphical calculator in the parametric mode to help you with the following activities. Sketch the graphs of the curves given parametrically by the following and. describe the Cartesian form of these curves: x = cos and y = sind x =3}ecos@andy =4+sin0 -3 +cos@andy =4+sin@ ~3+cosOand y =-4+sin? 4-cos@ and y =—4 +sin 0 eaore nH [2] Predict, and then chock, the behaviour of the curve given parametrically by x = 2cos and y = 2sin 0. What do you expect the curve given paramotrically by x — 5 +2cos and y — 1+ 2sin0 will look [3B] investigate other parametric coordinates. 6 Algebra Il Summary ‘A polynomial can be divided by a linear factor and the result is checked using either multiplication or substitution. ‘The remainder theorem says that when a polynomial P(x) is divided by a linear factor (x ~ a) then the remainder is P(a). The factor theorem says that if P(a) = 0 then (x —a) is a factor of P(x), ‘The factor theorem can be used to factorise cubic and higher order polynomial expressions. Partial fractions can be classified into three categories: those with linear factors in their deriominators; those with quadratic factors in the denominator; and those with repeated factors in the denominator. The cover-up rule uses the fact that some values of x create a zero in the denominator to calculate the required numerators in the partial fractions. ‘To sketch the graphs of continuous and discontinuous functions, check: @_ where the graph crosses the axes @ the location and nature of any stationary points @ the behaviour as x — +50 @ where the function is undefined. The graph can be discontinuous at a point, This is shown by a line called an asymptote. — A vertical asymptote can be found when the denominator of a rational function is zero. = A horizontal asymptole ean be found when the function tends towards a fixed value for large positive and negative values of x. ~ Askew asymptote can be found by writing an improper algebraic fraction in ils quotient-remainder form. Produce a sketch of the modulus of a function, | /(x)] , by reflecting in the scaxis all parts of the graph for which the function is negative. Urheberrechtlict 294 7 Exponentials and Logarithms What you need to know How to write numbers in standard index form, The laws of indices. How to evaluate negative and fractional indices. How to use the x"! or x7 function key on your calculator. Review Write the following numbers in standard (index) form: a 93.000000 b 0,006.25 ¢ ten million d_ three hundredths Simplify the following: a a’xa® e (my! dsm, where m 40 f 5 Without using a calculator, work out: a 16! b o27 e 4! a @ e 64 f 1254 ‘Use your calculator to find the values of the following, correct to three. significant figures: a 10) b (0.03) © (12) d= (-200)4 ‘7 Exponentials and Logarithms 7.1 The Exponential Function Any function of the form a*, where a is a positive constant, is known as an exponential (or power) function. The variable of the function is x. Examples of exponential functions include 2°, 3, 5* and 10°. The graphs of these functions all have the same basic shape. Since a® = 1, all graphs yon 0 s yaar yas @ The graphs are defined for all values of x. ‘The graphs are always above the x-axis. The x-axis (the line y = 0) is an asymptote, and each graph tends to infinity quickly as x gets large. Exponential functions can be used to model growth and decay. To model decay we find that the exponent needs to be negative. Graphs of exponential functions with negative exponents are the same shape as those with positive exponents, but reflected in the line x = 0 (or the voaxis). Exponential functions are used to predict population growth, describe radioactive decay and determine compound interest. Population growth The world’s human population is growing by about 3% each year, Given that the population at the end of 1989 was estimated at 4.5 billion. (4.5 x 10°), we can predict population figures for future years using an exponential graph. Multiplying by 1.03 increases by 3%, be es ee eT Graphical calculator Complete a table of values or use a graphical calculator to draw the graph support of y = 4.5 x 1.03". Notice that the curve crosses the y-axis at 4.5 pack (representing 4.5 billion in 1989, our initial year), Using a trace and zoom function we can find when the population doubles (y= 2 x 4.5 = 9). Complete Advanced Level Mathematics: Pure Mathematics We find that y = 9.0 when x = 23.499772. Sox = 24. ‘This means we can predict that the world’s human population will have doubled in size from its 1989 estimate by the end of the year 2013 (1989 + 24 years). Radioactive decay Radioactive substances have ‘half-lives’ that are determined by the time it takes the radioactivity to halve. Radon-219, an isotope of the element radon, has a half-life of approximately four seconds. This means that if at some time ¢ = 0 we have 10000 Radon-219 nuclei, then four seconds later (t= 4) there will be only 5000. ‘The exponential model for radioactive decay will have a negative exponent (power). To find it, first tabulate some values. 10000 | 10000 x (}) | 10000 x (}* | 10 000 (3)* | 10000 x (5)* =5000 = 2500 =1250 = 625 Notice that every four seconds the power increases by 1. If N(J) is the number of atoms at time ¢, then N(t) = 10000 = (3)! = 10000 x (27!) = 10000 x 24 This model now allows us to calculate the number of nuclei of Radon-219 present in the sample at any time after timing commenced (1 = 0). Compound interest Banks and building societies add the interest to certain accounts at the end of a year. In the fallowing year interest is also calculated on this interest. This method of calculation is known as ‘compound interest’. Consider investing £500. What would give the best return, 109% per year or 196 every j, year? Compare the balance at the start of each year. ‘Trace and zoom The Trace’ facility 0 graphical calculator described on p.34. 1 ‘Zoom facility enabl you to zoom in ona region, or section, of plotted graph. Most Jeschitztes Material 7 Exponentials and Logarithms Clearly the second option gives a better return. But how can these figures be calculated quickly? In the first account £500 can be multiplied by (1.1)" where n is the number of interest paying years. [n the second account £500 can be multiplied by (1.01)" where n is the number of ; years for which interest is paid. Thus £500 x (1.01)" = £552.31, ‘The idea of reducing the interest rate but increasing the number of times it, is paid is nothing new. Today interest can be paid daily (so in a year, n would be 365) but the rate will be only a fraction of 1%. Now look at the following pattern: (1+0.1)" =2,593 742.46 (1 +0.01)" = 2.704813 829 (1 +0.001)"™ = 2,716 923 932 (1+ 0.0001)!" = 2.718 145927 ‘The left-hand column is generating the pattern (1 +4)" for powers of 10. ‘The figures on the RHS are converging toa limit that begins 2.718... This number is known as e' and is the limit ofthis sequence. ‘This can be written mathematically as ¢ = limit (43) or mare usually as e = lim (143). _— ‘The exponential function as a series The number ean also be defined ase= 1424+ 242 po. ‘The function e* is often referred to as the exponential function. It can be dofined, ina similar way to 0, as: If we differentiate this infinite power series term by term we gel a remarkable result, a xix RO ayy yt ae ‘That is, the gradient function equals the function itself for all real values. It is this result that makes the exponential function, e*, unique. schiter OF Complete Advanced Level Mathematics: Pure Mathematics. 7.1 The Exponential Function Exercise Technique 1 | Using a calculator find, correct to three significant figures, the values of: ae bet c ete? det 2 | Use a graphical calculator to sketch the graphs of the following. In each case describe the transformation that maps y= e* onto each of these functions: a yao) ob yaehtt ¢ yao! d yaot-2 3) Draw the graph ofy = 2°. From your graph find, as accurately as you can, the value of: a 2 b 29 2 oe doo | Using a calculator, sketch the graph of y = o*. Use the trace and zoom functions to find values of x correct to three significant figures for which: a =6 b eX =8.12 c e&=05 d @=n Contextual 1 | The mass of a colony of bacteria, measured in grams, doubles each day and is given by the formula M(t) = 7.0 x 2'. Find the initial mass of the bacteria and the number of days needed for the total mass of the bacteria to exceed 500g, kt 2 | The decay of a radioactive isotope can be modelled by M(f) = Mye™, where M(t) is the remaining mass after t days. If a given isotope has a half-life of 10 days and after this time 60g remain, find: tially (the value of Mg). ‘how much isotape was present i ‘the value of the decay constant k. ‘how much isotope has remained after 30 days. the number of days required for 80% of the isotope to have decayed. noes 3) Engine oil, at temperature T°G cools down according to the model T = 600+ 10, where is the time in minutes from the moment the engine was switched off. a What is the initial temperature of the oil when the engine is first switched off? b If the oil cools to 32°C after three minutes find the value of k. © How Long will it take for the oil to cool to a temperature of 15°C? 7 Exponentiais and Logarithms ‘The cost of living is increasing by 4% per year. By how much does the cost of living increase in: a amonth b six months c five years? How long will it take for the cost of living to double from its present value? [5] Acar bought for £12 000 depreciates at 20% per year. After ¢ years itis worth £x. a Sketch a graph of x against t. b Use your graph to calculate: i when the car is worth half its original value; ii when its value first falls below £1000. Urheberrechtlich geschiitzte ‘Complete Advanced Level Mathematics: Pure Mathematics 7.2 Logarithms When comparing numbers written in standard form, first look at the indox, (exponent or power) of the number ten, This gives an indication of the magnitude of the number. For example, if the index is 6 then the number will be in the millions because 10° = 1000000 = 1 million Now consider numbers between 10 and 100. Since 10 = 10" and 100 = 10* we would expect the index for these numbers to be between 1 and 2, What about 1.5? What number is equivalent to 10"? The answer is not 50, Using the ‘x”” function key on a calculator you should find that 10'S = 31.622777 (6 dp). ‘The number 1.5 is called the logarithm of 31.622 777 in base 10. If you naw try the ‘log’ function key on the calculator notice that oe Ithough invent Jog (31.622 777) = 1.5 Europe by John Napier in the sixteen century, there is plen |, evidence to show tha logarithms were knoy | Asian mathematicians the period around te { cighth century. The may have dev thre idea of lags with fusing them for any Practical purpose. Mathematicians generalise this relationship as follows, [Bia - me, and x is known as the logarithm of y in base a. Since 31.622 777 = 10"? then 1.5 = logy 31.622 777 Logarithms can be in bases other than 10. Example For each of the following, write down an expression for a logarithm in a suitable base: a ei=3t © 100= 10° Solution a 81=3",s04=log, 81 b 8 =2',s03 = log, 8 © 100= 10,802 = logy, 100 d 2°, so —5 = log.(h) The logarithm of a number is the power to which the base must be raised in order to equal that number. So logy 100 is 2 because the base, 10, must be raised to power 2 to equal 100. ‘The most common bases used for logarithms are 10 and e. In value, = 2.718 2818 and logarithms taken to this base are called natural logarithms, ‘To distinguish betwoen these types of logarithms the follawing notation is used. logo is written logx or Igx Joga is written Inx Calculators have two different function keys for logarithms, ‘LN’ and ‘LOG’, with inverse functions e* and 10* respectively. ‘The relationship between o* and Inx is very important. ‘The use of e as a base for logarithms is by no means accidental. The number ¢ has many applications in mathematics. Transcendental numbers A number that cannot satisfy an algebraic equation with integer coefficients is called a transcendental number. Many irrationals are not transcendental. For example, V/2 satisfies x* = 2 or x* — 2 = 0, which are equations with integer coefficients. Both e and 7 are transcendental. Urheberrechtlich geschitz 20M aerial Complete Advanced Level Mathematics: Pure Mathematics 7.2 Logarithms Exercise Technique 1} Without using a calculator, find the values of: a log1000 hb logo.01 cc log10000000 d= logo.0001 | 2 | Find the values of the following, to three significant figures: a logs0 blog © log(s) d log() [3] Without using a calculator find the exact values of x in: a x=log,64 b x =log.27 © x=log,() a x= log) [4] rind the vatue of the base. a, of the following logarithms: a 9=log, 12 b © —2= logs) d [5] Find the values of the following, to three significant figures: a In2 b Inso © In100 dns Evaluate the following without using a calculator: . a ine! b Inve 1 1 © in( ) a n(e | } [77] copy and complete the following table: § Material 7 Sxponentials and Logarithms Contextual ‘The Richter scale measures the magnitude of earthquakes using logarithms. The Richter valuo is only part of the number describing an earthquake's strength. If it has a Richter value of 6, the magnitude of the earthquake is 10°, Many people believe an earthquake with a Richter value of 8 is twice as strong as one with a value of 4. How do the two actually compare in intensity? [2] ‘The human ear responds to the ratio of the powers, measured in walls, when the power of sound increases. This ratio is measured in bels (B) but in practice the bel is too large and the decibel (dB) is used. This is calculated using the rule change in dB = 101og10 ae) old power a Explain why an increase in power output from 100 W to 200 W is only 3dB. b Approximately how many times will the power level emitted by a source of sound have increased if the sound level increases by 10 dB? ‘¢ Ifthe volume control on a personal stereo is turned down so that the power output from the loudspeaker changes from 500 mW (milliwatts) to 100. mW, what is the corresponding fall in sound level indB? [3] tm chemistry the acidity or alkalinity of a solution is measured by its pH factor, defined by pH = —logig [H"]. In this rule [H*] is a measure of the quantity of hydrogen ions present in the solution. A pH value of 7 indicates a neutral solution for which [H™] 1077 moles per litre. If the pH value is groater than 7 the solution ig alkaline. Ifthe pH value is eas ‘than 7 the solution is acidic, a Calculate the pH value for a hair shampoo of strength 2.5 x 10-° moles per litre. b Anacid X, has a pH of 5.0, and anacid, Y, a pH of 2.5. How many timas more concentrated than acid X is acid Y? Complete Advanced Level Mathematics: Pure Mathematics 7.3 Laws of Logarithms (iene tr, Remember that these two mathematical statements are identical and interchangeable, Now suppose that there are positive numbers c and d such that ¢ = m" and d = m' for some m > 0. Then p = logm ¢ and q = logy d and cx d= m? x m? = mint In logarithm form this can be written, I = =I d ra login (¢ x d) = p+ q = logg,¢+ log Pi Desa tapler (1550-1617) This result is true for any two positive numbers ¢ and d and for any ‘Napier introduced the suitable base m. Since it is true for any base, ERBBB ea + tos) Property 1 Learn this result. Example 1 Given log Z = 0.301 and log 6 = 0.778 find log 12. Solution log 12 = log(2 x 6) = log2 + log6 = 0.301 + 0.778 =1079 Now return to property 1 and put a = b. This gives the following result: loga? = loga + loga = 2 loga ‘This can be generalised to RRR ogc Property 2 < Learn this result. This is valid for all values of n, and not just integers. Example 2 Given log 6 = 0.778 find log 36. 7 Exponentials and Logarithms Solution log 36 = loge” = 2log6 = 2% 0.778 = 1.556 Check this using your Now return to¢ =m? and d = m7 with m > 0 and try division instead of multiplication, & d In logarithm form this can be written, am poe mm logs, @) =p-q=iog,c~iog,d ‘This can be generalised to Bee) Piece oxo Property 3 Learn this result. Example 3 Using the results from Examples 1 and 2, evaluate log3. Solution logs = ton (28) = log36 ~ log 12 = 1.556 - 1.070 = 0.477 ‘There are two other logarithm properties worth noting, based on the indices statements a = 1. and a' =a, TERRE 0 Property HERBS 1 Property ‘These five properties ean be used to simplify expressions containing logarithms and to solve logarithmic equations. 4 Learn these results. Example 4 Write the following as single logarithms: a log8 —log6 + logo b 2loga +3 logb — loge Uthebertectilich geschitztgggaterial ‘Complete Advanced Level Mathematics: Pure Mathematics Solution a log —logé + log 9 = log(}) + logo -m(te) = log 12 bh 2loga + 3logb — loge = loga’ +logb? — loge = log(a*b*) —loge a®b® ~6(F) Some logarithmic equations are a mixture of logarithms and ordinary numbers, When that is the case, try collecting together the logarithmic terms so that the logarithm properties can be used to combine them into a single logarithmic term. Example 5 Given that 1 + log,(7x — 2a) = 2 log, x + log, 3, find, in terms of a, the possible values of x. Solution 1.4 log,(7x — 2a) = 2log,x +log,3 + 1 = 2log, x + log, 3 — log,(7x — 2a) + = log, x? + log, 3 — log, (7x — 2a) > 1 = log,(3x7) — log, (7x — 2a) > i= lose 5 as But log, a = 1,50 ee Te— 2a > 3x? = a(7x — 2a) > ax? = Zax — 2a" => Gx? — 7ax +2a" = 0 Factorising, (3x ~a)(x— 2a) =0 F: —6cand —a. F: Replace —7ax wit =6ax = ax, and factorise. a Sox=" orx=2a 3 306 Urheberrechtiich geschit —_ 2 3 7 Exponentials and Logarithms 7.3 Laws of Logarithms Exercise Technique Write the following as single logarithms: a In74In2 d log 12 —logé b log2 + logs + log4 e In6+In10-In5 ec 2In3 f logi—log3 Given that log 2 = 0.301 and log3 = 0.477 find, without using a calculator: a log4 b log27 © logis d log36 e logi44 f logil Simplify: a logx +2 logy + logz do inx’-dInx b 3inx-2iny e Slogx+2logz cc loga ~2logb~ 3 loge f Inx—2Iny +Inz Express, in terms of Inx, Iny and Inz: a In(wz) b mn (“) is mnt a nig : i ff F int Solve the following logarithmic equations: a log =3 b logy16=—4 e Inx'-Inx+Inyx=% ds log, 24+ log, 9+ 3 = 3log, 4 Contextual Explain why xy = 100 and logiax + logia.y = 2 are equivalent, interchangeable statements. On the same axes sketch the graphs of y = Inx and y = In(4). Using the laws of logarithms suggest a possible transformation for these graphs. Describe an inverse function for y = In(2). a Show that log, 12 = log, 144 b Evaluate log, 12 — log, 9 without using a calculator. By taking natural logarithms, or otherwise, find g(x) such that x* = a'®*! for all values of x. Given that log, x + 2log,y"= 4, explain why xy = 16. Urheberrechtich Complete Advanced Level Mathematics: Pure Mathematics 7.4 Solving a* = b Consider the equation 3° = 20. This can be solved by trial and improvement using the power function key ‘x"” on any scientific calculator, although this method ean be quite time consuming. A more. concise way of solving exponential equations like these is to use the properties of logarithms, The method is based on the principle that if two sides of an equation are equal then the logarithm of one side must be equal to the logarithm of the other. Example 1 Salve 3° = 20. Solution Taking natural logarithms of both sides, In3* = In20 xIn3=1n20 4 Using property 2. In 20 ‘Ind 2.996 = Fogg = 2-73 G5) ‘The same principle can be used when the powers become more complicated. Example 2 Solve 4°") = 79, Solution ‘Taking logarithms of both sides, Logyy 4°) = logy 79 (3x +4) logig 4 = logy) 79 = 181079 ott _ 1 (logio 79 v= (tae bia, xo717s = 2 (28078 1) 0.717 (a sit) oae es Notice ~ 30.6021 " me is close to 79. Notice rounding error. This technique provides @ useful tool for solving more complicated exponential equations where one power is a multiple af another. 7 Exponentials and Logarithms Example 3 Solve the equation e+e — 6 = 0. Solution Notice that 4x = 2(2y), so this equation could be written (e®F +e%—6=0 ‘Now substitute v =o, This transforms the exponential equation into a quadratic equation. yity-6=0 > (y+3)0-2)=0 > yo-dory=2 Soo =-3 or o™ = 2. ‘These two statements can be analysed using logarithmic functions. e* = —3 has no solutions since e* > 0 for all values of x. Ife = 2 then In(e) = In2 2x Ine = In2 2x = In2 x= jing = 0.347 (3 sf.) So the equation e* + e** — 6 = 0 has only one solution; x = 0.347 (3 s.f.). Logarithms can also be used to solve problems involving exponential functions and inequalities. Example 4 a Find the smallest integer p such that 2.5” exceeds one million. b Find the largest integer x such that (0.7)* > 5. Solution a Taking logarithms of both sides, Logi 2-5” > logy 1000000 => plogio2.5 > 6 6 o P> Teg S = p> 15.077 Since p has to be an intoger, p = 16. Note that logyy is preferable here because 1.000000 is a power of 10. Cheek that 2.5 >1x 10° and 25" <1 10". Uthebertecttlich geschittz 80 aerial Complete Advanced Level Mathematics: Pure Mathematics by Taking logarithms of bath sides, logia(0.7") > logia5 => xlogy0.7 > logieS Logie 5. = Tog i007 > xX <-4.512 ‘Since x must be an integer, x = Check that the negative result is sensible. This can be dane graphically by considering the intersection of y = 0.7" and y = 5. Cheek this result ona graphical calculator. o * Sometimes x can appear as a power on both sides of an equation. To solve this logarithms can still be used. Example 5 Find a value of x such that $* = 7", Solution Start by taking logarithms of both sides: Ins* = In7**) = xln$ =xln7—2In7 sing property 2. = xIn5 ~xln7 =-2In7 Collecting like terms = x(In5 —In7) = —21n7 and factorising. yo Zin? 2in7 InS— In? in7—In5 x= 11.6 (3 sf) Compare 51°" and 7° ‘on your calculator. Graphical applications We have looked at techniques of solving problems of the form a = b. Those techniques also have a graphical application. If an exponential 310 theberrect schiitztes Material 7 Exponentials and Logarithms relationship exists between two sets of data, then logarithms can be used. to simplify their exponential graph into a straight line. This technique is called reduction to linear form. There are two basic cases to consider: @ the exponentis variable ‘Exponent’ ‘index’ and ‘power’ all mean the @ =the exponent is constant. The exponent is variable ‘This means that the relationship between x and y is such that y =a x B® = ab" where a and bare constants. By using the logarithm techniques: logy =logab™ logy = loga +logh™ 4 By property 1, logy =loga+xlogh 4 By property 2. Look carefully at this result. What do you notice? It has the form Y= mX + cwhore ¥ = logy, X = x, m = logb and ¢ = loga. So plotting logy vertically against x horizontally should give a straight line of gradient Jog b with a vertical intercept of log a. Example 6 ‘The relationship between the values of x and y in the following table is y =ab*, where a and bare positive integers. Find a and b. o}1|2]a]4 5 | 10 | 20 | 40 | 80 Solution Plotting the data on a graph gives an exponential style curve. o 1 2 a 4 = However, when logy is plotted against x a straight line is obtained. Urheberrechitich geschatztes Siria Complete Advanced Level Mathematics: Pure Mathematics 312 2 a 4 1.301 | 1,602 | 1.908 From the straight-line graph notice that the intercept with the ‘y-axis is approximately 0.7 and the gradient is about 0.301. We have already seen that an exponential equation in the form y = ab* can be written in the linear form logy = laga +x logb, where log is the gradient of the line logy =loga +xlogb, and loga is the intercept with the vertical axis. From the graph, loga ~ 0.7 and logb ~ 0.301. Soa=Sandb=2 So the equation that fits the data is y= 5 x 2°, The exponent is constant ‘This means that the relationship between x and y is such that y=axx" =ax!, where a and b are constants, This is really a power function and nol an exponential function. Using the logarithm properties, logy = log(ax”) logy =loga +logx’ By property 1. logy =loga +blogx By property 2. What do you notice about this result? This also has the linear form Y = mX £c if logy is plotted vertically as ¥ and log x is plotted horizontally as X. The resulting straight line will have a gradiont of b and a vertical intercept of log a. t somment. “tt Example 7 ant be dove by et ‘The relationship between x and y in the following table of values is ater went onto deve oar - ber of machines, y= ax? where a and b are positive integers. Use a graphical methad to lncluding the fst eal find aand b. hich he called his “amltical engine” 0] 02S smi o | 2] 16 | sa | 228 7 Exponentials and Logarithms Solution ‘The data plotted on a graph gives a curve. intercept iso.3 Or 02 08 04 05 O68 log From the graph notice that the vertical intercept is 0.3 and the gradient is 3, From the linear equation in logx and log y, log y — loga + blogx, this means loga ~ 0.3 and b = 3. Soa=2andb=3. ‘Thus the equation that fits this data is y = 2x", ‘When you got sets of data like these a decision has to be made. Which Check this model mode! should be used? If either of the graphs gives straight line then an _ against the original data. exponential or power form would be the expected equation. One way of saving time is to look at the original data and determine if it goes through the origin. If it does, then suspect that the exponent is constant. Urheberrechtich geschitzte® NBvoria Complete Advanced Level Mathematics: Pure Mathematics. 7.4 Solving a* = b Exercise Technique [11] solve the following equations: a 2 =4000 b e© 11% =151 da [2] solve the equations: a stags b c s*a51 d [3] Find the value of xin the followings a © b eto ao amelogiea [4] sotve the following exponential equations: ao = 505+ 6=0 b 208% -9e' +4 =0 eo ef 4e'-6=0 d 60-130" —5=0 [5] rind the smallest intoger p such that: a 2 > 1000 b 9? > 2000 «© (0.9)? < 0.0001 d= (0.5 <0.001 [6] Find equations for the following sets of data: * 0 a 2 a 2) ase as » 0 1 2 3 4 ee © RE SE Ears ee d 0 os 1.0 15 20 a 110 | 0.406 | 0139 | 0.0549 Solve the following equations: a 100"— 10" +16 =0 b 10% —10"*'421=0 10% 10.10" +24=0 d= 10*-100=0 314 eberrechilich geschttztes Material Contextual 7 Exponentials and Logarithms 4 5 1 2 3 121 | 184 | 28.0 42.5 646 Itis believed that P and ¢ are connected by an exponential relationship of the form P = ab‘, where a and b are constants. Verify this graphically and find the values of a and b. When is the population expected to exceed half amillion? exponential equation? 2 | show that the equation e* - 7e — 8 =O has one real solution. Why does the second solution of the related quadratic not produce a solution to the 3 | Before Newton's theory of gravitation, the best mathematical model to describe planetary motion was formulated by Kepler (1571-1630). Kepler stated three laws, the third of which gave a relationship between the distance of the planet from the sun, R (millions of kn), and the period of its orbit, T (years). Kepler had data for the following six planets. Using the model T = af, where a and b are constants, find Kepler's third law. Check this mode! on the following data for planets not known to Kepler: ‘Uranus 2870 83.943 Neptune 4497 104.061 Pluto. 5907 248.241 How accurate is the model? [4] Find the real value of k for which 10° = ¢ for all values of x. 1 | The population, ? thousands, of a new town is calculated every t years after 1979. The results are summarised in the table below. theberrechiich geschitztes WEAF Complete Advanced Level Mathematics: Pure Mathematics 316 Consolidation Exercise A log V3 —logav2 _5 1 | Without using a calculator show that logs loge 3" [2 | Solve the following inequalities: a In(x+2)<3 b (7/5 >3 3} The value of a car (in £) can be modelled by the equation V = 8500e-*, where ¢ is the age of the car in years and J is a constant. a State the value of the car when it was new. b After two years the value of the ear was £6560. Use this information to calculate the value of 4. Estimate the value of the car after three years. d= How long would it take for the value of the car to be half its original value? [4] IfAg = 100 and A,,., = 1.074, find a formula for A, in terms of n, ifn represents the number of years a sum of money is left in an account and Ay represents an initial investment of £100, what does 1,07 represent? ¢ How many years would it take for the investment to double? oe [5] Given that y = 10°, show that: y? = 100° b §=10""' ¢ Using the results from a and b write the equation 100° — 10.001(10"~') + 100 = 0 as an equation iny. d__ By first solving the equation in y. find the values of x which satisfy the given equation in x. (ULEAG) [6] The population of Portugal ¢ years after 1990 can ‘be modelled by the equation P = Poe’. a In 1990 the population was estimated to be 10.5 million. Write down the value of Pp. b The growth rate was expected to be 0.5% per annum, Explain why the population projection for 1991 is 1.05 < 10” x (1.005). ¢ Use the result from b to find the value of k correct to three significant figures. d_ Use the result from ¢ to predict the population for 2000. 7 Exponentials and Logarithms Given that y = log, 45 + log, 25 — 2 log, 75, express y as a single logarithm in base b. In the case when b = 5, state the value of y. (AEB) For certain planets, the approximate mean distance x, in millions of kilometres from the centre of the Sun, and the orbit T, in Earth years, are recorded, 57.9 108.2 227.9 | 778.3 0.24 0.62 1.88 11.86 Assuming a law of the form T = Ax”, draw a graph of InT against In.x. Estimate the values of A and n, giving your answers ta two significant figures, Use your graph to estimate the approximate mean distance in millions of kilometres of the Earth from the Sun. (AEB) The graph shows the function reo f(s) =e" forx > 0, By writing =e"? find an expression for x in terms of y. Hence write down f~'(x). Says e"? a cq Aslide into a ball pool at a children's play centre can be modelled by the curve y =3 x 107*/? where x and y are measured in metres. The slide has three supports, fixed to the ground at positions A, B and C as shown in the diagram. Q’ H : oe Calculate the height of the supports at A and G. b The support at Bis half the height of the support at A. How far along the ground is support B from support A. Complete Advanced Level Mathematics: Pure Mathematics 318 Exercise B Without using a calculator express as a single logarithm, atos( $5) +g + 2log2 Solve the following inequalities: a In(ax) <5 b (0.95)! >8 ‘The value V of a particular car, in pounds, at age { months from new can be modelled by the equation V = 12000e"“ + 2000, where k is a constant. a Use this model to write down the value of the car when new. b The value of the car is expected to be £8000 after 24 months. On this basis, calculate the value of k. (AEB) An employee joins a large firm on 1 January of a particular year. In order to provide a sum of money for the employee's retirement, £1500 is paid into special account on 1 January each year for the previous year's work. In addition, on each 1 January, interest is added to the account, the interest being 696 of the total amount in the account immediately prior to the annual payment of £1500. a Show that for an employee, who retires on 1 January after serving n years with the firm, the sum of money in the special account is equal to £25 000(1,06" — 1), b Find the least number of years required for the sum to exceed £100 000. (WIEC} By treating the following equation as a quadratic in e*, find the two values of x satisfying o* — 50° +6 =0. (WJEC) ‘The population of Iceland t years after 1990 can be modelled by the equation P = 0.251¢", where Pis measured in millions. a If the growth rate is estimated to be 0.8% per annum, estimate the value of 2. bh Inwhat year would the population reach half a million? Given than In(3x — 5) —In4 = 2Iny: a find the value of y when x = 2 express xin terms of y ina form not involving logarithms. (AEB) 7 Exponentials and Logarithms Applications and Activities Fibonacci numbers This sequence of numbers owes its name to the Italian mathematician Leonardo da Pisa (1170-1230), Each term of the sequence is the sum of the two previous terms. Since 1+1=2,1+2=3,2+3=5andsoon the sequence becomes 1, 1, 2, 3, 5, 8,13, ... If these numbers are tabulated, 6 6 |e | 43 and then plotted graphically, a curve is formed. Find the equation for the curve gonerated by the Fibonacci numbers. ws * The logarithmic spiral This spiral intersects all radii al a fixed angle and the distances from ils pale increase in a geometric sequence. To sketch a logarithmic spiral quickly use European DIN paper size AO. Fold the AQ size sheet of paper in descending"A' size order and draw a line from one carer to the opposite corner of each section in a clockwise or anticlockwise order. Jee Logarithmic spirals occur often in nature. Investigate! ‘Complete Advanced Level Mathematics: Pure Mathematics Summary iy =a then x = log,y. Common logarithms have base 10, and are written as logyo oF log. ‘Natural logarithms have base ¢, and are written as log, oF In. ‘The five basic properties of logarithms for all bases are: logab = loga +logb loga” = nloga logs = loga — logb ogi =o log, @ @ cis the exponential function and is used to model growth and decay. @ Relationships of the form y = ab* can be transformed from an exponential curve to a straight line by plotting logy vertically against x horizontally. ‘The gradient of the straight line is logb and the y-intercept is log a. @ Rolationships of the form y = ax! can be transformed from an exponential ‘curve to a straight line by plotting logy vertically against logx horizontally. The gradient of the straight line is b and the y-intercept is loga. Irheberrechiich 8 Sequences and Series What you need to know @ How to collect ‘like terms’. @ How to expand brackets, @ How to substitute values into expressions, @ How to express an algebraic fraction in terms of its partial fractions, Review 1 | Bycollecting together like terms, simplify the following expressions: a atated b atat(n—1)d © atartar d 1tx8 49-18 px) | 2 | Expand the following: a (n43)(n44) bo (r42)(r—1) e r(r+1) do (itx)? 3 | Evaluate: a 2n+3,whenn=3 b 3n-1,whenn=5 ¢ oné—1,whenn =4 d $n(n—1)(a—2), when a= e ar,whena=3,r=2 a(l =r") (-r) + When a =10,r=}.n=4. 4 | Express each of the following in terms of their partial fractions: 5 b 6 rei) (FF) +2) 7 a2 (n+ 1)(n +2) n(n +1) 321 ‘Complete Advanced Level Mathematics: Pure Mathematics: 322 8.1 Sequences and Series Sequences A sequence is a set of numbers occurring in order. Some examples of sequences are: the sequence of even numbers the sequence of perfect squares the sequence of triangular numbors. ‘The terms of a sequence are often represented by ordered lower-case letters. Thus u, denotes the first term and w, denotes the second, and the sequence becomes: Uys Ma, ty For the sequence of even numbers, uy = 2, uz = 4, uy = 6, and soon. A sequence can be described in several different ways, One method is to give an algebraic expression for the nth term. The nth term for the sequence of even numbers is 2n. This can be written: Uy = 20 Check that u, =n? and u, = $n(n +1) generate the sequence of perfect squares and the sequence of triangular numbers respectively, Example 1 Find the first five terms of the sequence whose nth term is given by _nln —1)(n-2) = 6 Solution ‘To find the first five terms put n = 1,2, 3, 4 and 5 respectively into the expression for u,. -1(-2) u, =18 Nt ) 0 (2 —1)(2 uy = '8 Sequences and Series w= MG =DG=2) 4 So the first five terms are 0, 0, 1, 4, 10. There are other types of sequences where the sign oscillates between. positive and negative. Example 2 Find the nth term of the sequence ~1,3,-5,7... Soiution Notice that without the change of sign this sequence is the sequence of odd numbers. For odd numbers, ti, = 2m — 1, ‘The change of sign in the sequence can be generated by using powers of ~1. (-1)'=-1 Odd powers give negative results. 4 Even powers give positive results. (-18 =-1 (-yt=1 So the nth term of the sequence ~1, 3. 5,7 is given by Uy = (1)"(2n — 1) Sometimes the rule for defining a sequence can be given in the form of a recurrence relation, This means the nth term u,, can be calculated only when the preceding terms are known. Example 3 ‘The Fibonacci sequence, 1, 1,2, 3,5,8,13,21,... can be defined by a recurrence relation. Find this rule, Solution To find the recurrence relation look at how the terms of the sequence are related to each other. uy=t 4, Urheberrechtlich geschittzte@@Sieria Complete Advanced Level Mathematics: Pure Mathematics: my =2=u, bay y= 9 =a tay Us = 5 = uy bay Ug = 8 = Uy bit So Ug = Uy-1 + Up-2 ‘The nth term in the Fibonacci sequence can only be found if the previous two terms, u,_, and a_, are known. Example 4 a Find the first four terms of the sequence defined by the recurrence relation u, = u,_; +2 with w, = 3. b Find the first five terms of the sequence defined by Uy = uy) — Up 2 With uy = 1 and uy = 2, Solution a Given uj, find u, using the given recurrence relation. Then calculate u, and u, using the same technique. uss up =u, +2 =5 uy =a, +2= uy =u, +2=9 So the first four terms of the sequence are 3,5, 7, 9. b Use the values of u; and u, to find uy. Then generate u, and u; using the same recurrence relation. uy=t uaz ay =3u; =u, =6-1=5 ‘Remember to put n = in the recurrence uy = Guy — uy = 15-2 =13 relation. uy = Suy uy = 39-5 = 34 So the first five terms of the sequence are 1, 2, 5, 13, 24. 324 Urheberrechilich gaschitt jes Material 8 Sequences and Series Series When the terms of a sequence are added together, a series is formed. 2444648 isa finite series 144+ 94+ 16 is an infinite series Ifa sequence has five terms, t,, U2, Us, Us and us, then the series based on this sequence would be Uy uy boy + uy bus This series can be written ina more concise form. Instead af appearing as a string of terms added together, the series is written using sigma notation. 2 y+ iy ty + ty tts = 3 uy mi Bis the Greok letter, sigma. It is used in mathematics to represent a summation. The letter ris a dummy variable, The r= 1 at the foot of the sigma indicates the term with which the summation should start, and the S.al the top indicates the term with which the summation should finish. 7 ‘Thus > u, means start with the fourth term, us and sum to the seventh Fd term, u;. Example 5 5 Wu, = 2r—1, find up. Solution ‘To sum this series, first substitute the values of r from 1 to 5 into the expression for the rth term of the sequence and then sum the results. ys u,= S@r-1) mF = (2-1) + (4-1) + (6-1) + (8-1) + (10-1) = 143454749 =25 7 Sigma notation is particularly useful when the series is infinite. In these cases there is no last term, so writing the series as a summation string would be impossible. In sigma notation, however, we can write: Do up say tay tay bee Urhebertechtlich geschiitz 2M teria Complete Advanced Level Mathematics: Pure Mathernatics ‘So the sum of the even numbers can be written: 24+446+8+-- Writing a series in sigma notation is dependant upon an algebraic expression for the nth term being known. Example 6 Express: a 143464104 15 in sigma notation b 7410+ 10-+ 16in the form 3 fle) mt Solution a Theterms in this series are from the sequence of triangular numbers. ‘The nth term, u, = 4n(n +1). Since 1 +3 +6+ 10 +15 begins with the first term and ends with the fifth term, 5 14346410415 = ED b The first stop is to find an expression for the nth term. Notice that the sequence 7, 10, 13, 16 increases in equal steps of 3. This suggests the expression for the nth term should contain a 3. Rewrite the sequence using this factor of 3. Note that r= 3 for the first term, u,=7 4, =10 uy = 13 =3x3-2 =4x3-2 =5x3-2 ‘This suggests that wo can write the nth term as uy =nx3-2=3n-2 ‘The series is finite with last term 16. Hence 3n-2=16 gn =18 n=6 6 Therefore 7410413 +16 = $7 (3r—2) mS 8 Sequences and Series 8.1 Sequences and Series Exercise Technique Find the first four terms of the soquences with the following nth terms: a oup=3n+4 bu, =4n-4 © u,=6n-1 do u,=(n+3)\(nt+4)e u,=2"-" fo usn'+1 Find u,, the nth term, for the following sequences: a 6,8,10,12,.. a 12,10,8,6, bb 14,17,20,23,... @ -3,-5,-7,-9,... © 1,6,11,16,21,... fF 0,3,8,15,24,... [3 | Use the following recurrence relations to find the noxt four terms of each soquence: A ty = Uy +3. do uy = upg — Uy-ge Wy = 2 = 1 bo ug=2uy yp t4yuy=1 ey = 2g y + Uy gy = 2, = 1 G Uy =3uy__,—1,u, =2 f up =} (Ups) +10 =4 Express the following series in sigma notation: a 14449416 do 345474+9411413 b 346411418 @ 042464124 20430442 c 44546474849 f 2464124+20+30+42 +56 Evaluate: 2 4 5 a yr b Sw +t) « yr a 3 ° A 2 d }(sr-2) e So ar+1) £ Sr+1)(r+2) mi a ms Express the following series in the given form of sigma notation: a 5+8+11+14, Sr em b sta+iti+i7+20, Ss) © 9411413445, ye do 9415421427, ye ret e 7+10+13+16+19, 5° fir) fa fF o+8+10+12+14+16+18, Sir) ms Complete Advanced Level Mathematics: Pure Mathematics 328 8.2 Arithmetic Progression An arithmetic progression is a sequence in which each term is produced by addinga fixed number. For example 2,5, 8,11,... is an arithmetic progression and 10,7,4, 1,—2,... is also an arithmetic progression. Notice that the number that is added throughout the sequence can be positive or negative. It is called the common difference. ‘The phrase ‘arithmetic progression’ is often abbreviated to AP. The first term is usually denoted by a and the common difference by d. Consider the AP 2,5,8,11,.... Here a = 2 and d = 3. This sequence can now be rewritten as 2 248 2+(2x3), 2+(3xI), 24+(4%3).. The nth term would be 2 +(n— 1) x3. WAP is 4 Learn this important result. Example 1 Find an expression for the nth term of the AP 10,7,4,1, Solution For this AP, a = 10 and d 4, =a+(a—1)d = 10 + (n—1) x (-3) =10-3n+3 =13-3n ‘Tho torms of an AP can bo added to form an arithmatic sories. a+ (a+d) + (a4 2d) + (at 3d) +--+ (a+ (n= A)d) + -- If this series is finite then it has a last term and it is possible to find an expression for the sum of the series. Example 2 Find the sum of the first 100 natural numbers. Solution ‘The first 100 natural numbers form an AP, with a= 1 and d= 1. The sum of this AP is a finite arithmetic series: T4243 4¢44¢---+ 100 How can this total be calculated? Gauss (1777-1855 ‘age nine he mentally cakulated the tota 5080. ‘8 Sequences and Series Let Sjop be the sum of the first 100 terms, Sig =1+243-4---+99+ 100 hl ‘Now write the sum again in reverse. Bio = 100 + 99 +98 +--+ 241 Pi Adding [1] and [2], term by term, 2 Sjqq = 101 + 101 + 101 +--+ 101+ 101 ‘We know that there are 100 terms on the RHS, so 2 < Sig = 100 x 101 => Sioa = }3(100 x 101) = 5050 ‘The same idea can be applied to every finite arithmetic series. Sp =at(atd)+(at+2d)+---+(a+ (n—-1)d) S$, =[a4+(n— 1d] +--+ (a42d) + (04d) +a 2S, = (2a + (n= 1)d] +++» + [2a + (n= 1)d) ‘There are n terms on the RHS, so BEER Ba inn. 4 Learn this important result. ‘The sum to m terms of an arithmetic series can be written in another equivalent form: | Example 3 For the AP 5,9,13,17,..., find: a thesixth term, ug b the sum of the first five terms, S;, Solution First identify the first term and the common difference. The standard results for nth term and the sum to n terms can then be applied. For the AP 5,9,13,17,...,a=Sandd =4 au =atin—a)d = uy =54(6-1) x4 =54+(5 x4) = 25 Urheberrechitich geschiit Complete Advanced Level Mathematics: Pure Mathematics b 8, = Flea + (m1) 84 =$ [2 <5) + (6-1) x4] = 3 (10416) =08 Example 4 Find the sums of the following arithmetic series: @ 24+548+114---+47 b 47441435 4---+(-43) Solution a Herea = 2andd=3. ‘The number of terms in the series, n, can be found by making the last term, 47, the nth term. Uy =a4+(n—1)d = 47 =2+(n-1)x3=3n-1 So 3n=47+1 > n=16 ‘The sum to 16 terms of the series can be found by using one of the formulas for S15. nla+)_ 16(2+47) =< = = 392 b Here a = 47 and d = —6, Making —43 the nth term, u, =a+(n=1)d = -43 =47 +(n-1)(-6) => -43=47-6n+6=53-6n So 6n=53+43 + n=16 This arithmetic series also has 16 terms. 9g agen 16(47 + (49) -— Why is this total so low? Recall that many of the terms were negative. Sig =474+41435+++-—25 31-37-43 This technique of identifying the first term, a, and common difference, d, for the AP is particularly useful when the information given relates to other terms in the series. 330 Utheberrechilich gaschiliztes Material 8 Sequences and Series Example 5 ‘The fifth term of an AP is twice the second term. The two terms differ by 9. Find the sum of the first 10 terms of the AP. Solution ‘The information given is that uy = 2m, and uy—u,=9 Using the fact that the nth term of an AP is given by u,,=a-+ (n~1)d, these two equations can be rewritten as simultaneous equations im a and d. tg = 2u, > ty — 2ury = 0 = (a+4d)—2(a +d) =0 = a+4d—2a-2d=0 > 2d-a=0 (i us—u,=9 = (a+4d)—(a+d)=9 = at4d-a-d=9 = ad = d= 2] Notice that equation [2] gives the value of d. Substituting this value of d into equation [1] gives (2*3)-a=0 3 a=6 ‘The sum of the first 10 terms can now be found. Use these values of a and, with n= 10 in the formula for the sum to n terms. S, = }n(2a +(n—1)d) So Sto = (12 + (9 x 3)) =5(12 +27) = 195 Example 6 Evaluate" (2r+1). ms Solution Set 1) =(641)4+(8+4) +--+ (8241) mt S7494-433 ‘This arithmetic series is generated by an AP where a= 7 and d= 2. Check Hint: Reduce r= 3 to that there are 14 terms in the series. The sum of the series can now be 1 =1in the dummy calculated. variable. Urheberrechilich geschiitzgg qlaterial Complete Advanced Level Mathematics: Pure Mathematics S, = Jni2a + (n—1)d] So Syq = (2x 7) + (13 x 2)] =7(14+ 26) = 280 ww So $7 (2r+1) = 280 mi Example 7 Pat saves £10 in the first month, £12 in the second month and increases the monthly savings by £2 each month. How long will it take Pat to save £5007 Solution Notice that the monthly savings form an AP in which a= 10 and d= 2, ‘Suppose Pat saves £500 in n months. This means the sum of the first n terms of the AP is 500. So = 3n(20+2(n—1))=500 4 Recall thata = 10 and d= 2. => = (204+ 2n—2)= 1000 => 18n + 2n* = 1000 = 2n* + 18n — 1000 = 0 ‘This is a quadratic equation in m. Solve it using the quadratic formula. py — 18 VIB =A Tx (1000) ~ 2x2 =183 or ~27.3(3 sf) ‘The negative answer can be ignored because the number of months cannot be negative. So £500 is saved in 18.3 months. Given that Pat saves monthly, the savings exceed £500 after 19 months. 332 Urheberrechilich ges chit 8 Sequonces and Series 8.2 Arithmetic Progression Exercise Technique 1 | Find (i) the next two terms and (ii) the nth term u,,, of each of the following arithmetic progressions: a 19,15,17,19, © 34,31,26,25,. b 8, 14,20,26, d Alyse 2 | Given the arithmetic progressions 3.7,11,15,...,u,: 6,11, 16,21,.. and 70, 66,62, 58,...,1,, find: a uy buy © My dvi e % fia ind the sums of the following arithmetic series: a 14549413 4---441 bo 2484144204+---4+38 © 447410419 +--.425 do 224274924..-447 | Use the method of Gauss (Example 2) to calculate: @ W+14+12+...+2 do 28427+26+...+17 bo 27 +244 214...49 @ 22417124... 18 © 564524484... 432 f -3-5-7-9...-21 5) Evaluate: w 1 a S(ar+2) b S@r-1) a at 20 ff © So (ar+s) dS (ar—10) St a Contextual 1 | The number of beams in a bridge structure form a sequence 3,5,7,9,.... What is the mth term of this sequence? 2 | Calculate the 20th term of the sequence —8,-2,+4,+10,.... 3 | Find the sum of the arithmetic series 15 + 18+ 21 +--+ 60. Check the result using the method of Gauss (Example 2). [4] ‘ind the sum of the first 16 terms of the arithmetic progression 114184254924... Complete Advanced Level Mathematics: Pure Mathematics S| The ninth term of an arithmetic series is three times the second term and the difference between the sixth term and twice the first term is 10. Find: athe first term b the common difference of the AP. G | The sum of the first five terms of an AP is 72, The sum of the first ten terms is 189. Find the sum of the first fifteen terms. 7 | when x is added to 6, 12, and 14 respectively and the results are squared, the new sequence is an arithmetic progression. Find x. 8 | Luke's piggy bank deposits increase by 5p each week. His initial deposit is 10p. How long will it be before Luke's savings are at least: a £10 b £100? In the week his savings exceeded £100, how much did Luke deposit? Q | An AP has three consecutive terms with a sum of 33 and a product of 935. Find the terms. 40) An AP has first term 10 and common difference 0.8. If the sum of the first terms is to exceed 250, find the value of n. BSequences and Series 8.3 Geometric Progression A geometric progression is a sequence in which each term is produced from the preceding term by multiplying by a fixed number. For example 3,6,12, 24,.... is a geometric progression, and so is 1, —4,1,—}, 4. Notice that the multiplier throughout the sequence can be positive or negative. It is known as the common ratio, In general the phrase ‘geometric progression’ is abbreviated to GP. The first term is denoted by a and the common ratio by F. AGP can then be written as a,ar,ar,ar®, ar’, ‘The common ratio can be found by comparing successive terms in the soquence, Beas Be Ue: Uy Wy Uy Uy GP 4 Learn this important result. Example 1 Find an expression for the nth term of the GP 3,6,12,24,.... Solution Laok closely at the sequence. uy, = 3, u; = 6, uy = 12, uy = 24 This GP has first term 3 and common ratio 2. apart uy = 3, uy =3 x2, uy = 3% 2, uy = 3 «23, 1 So the nth term is 3 x 2"? Example 2 Find an expression for the nth term of the GP 1,~1,4,— Solution Notice that for this GP, a = 1 and r= — 1 L u, = ar"? =tx(-prt =(-p"" Check this result by substituting values n=1,2,3,4and5 Complete Advanced Level Mathematics: Pure Mathematics ‘The terms ina GP can be added to form a geametric series. For example, the GP 3,6, 12,24 could be used to create the finite geometric series 3.464124 24. Itis also possible to find a rule to sum a geometric series, ‘The first term, a, and the common ratio, z, of the GP need to be known. LetS=3+6412424 Since this GP has a = 3 and r = 2 this can be written as S=34 (3x 2)+(3 x 27) 4 (3 x2") (1) Now multiply each term by the common ratio 2S = (3 x 2)+ (3 x 2?) +(3 x 2°) + (3x 2") 2] Subtracting equation [1] from equation [2]. 28-S=(3«2')-3 S(2 —1) = 3(2*-1) Check this result against the sum of the four terms of the geometric series, This procedure can be generalised to find the sum of any finite geometric sories. Let S, =a +ar+ar* +---4ar"* fa] ‘Then rS, =ar+ar +ar +-+-+ar" I) Subtracting [2] from [1], 5,-15,=a-ar" Sy(1—r) = afar") a Example 3 For the sequence 3,6, 12,24,..., find: a the next two terms b the nth term c the sum of the first five terms. Urheberrechtlich geschitztes Material 8 Sequences and Series Solution ‘The sequence 3,6, 12,24 is a GP with a= 3andr=2. a The next two terms are us and us, Using u, = ar"~", us = 3x2! = 3x16 =48 ug = 3x2? =3%32=96 b Theathtermis uy. un sar! =3 x2"! ¢ —Thesum of the first five terms is $5. aa—r) “Tr _ 30-2) 1-2 _ 30.32) =} _ 3(-31) ey =93 35 Notice that in Example 3c the formula for S; created a negative numerator and denominator, This was because the common ratio, r, was greater than. 1. In order to reduce problems with negative results an alternative form of the sum ton terms can be used. Example 4 A geomotric series has first term 8 and common ratio 1.4, Find the sum of the first 10 terms. Solution Since r > 1, use the alternative form for the sum of a GP. i = 556.51 (2 dp.) Urheberrechtli chee ggzpens Complete Advanced Level Mathematics: Pure Mathematics Finding the number of terms in a geometric series involves salving an exponential equation. Recall from Chapter 7 that this can be done using logarithms, Example 5 Find the number of terms in the geometric series 4+ 6 +9 +++ + 30.375. Solution ‘Since the series is geometric the ratio of successive terms will give the ‘common ratio. 39 Tn 6 =15) ‘This means a = 4 andr = 15. ‘Now examine the last, or ath, term, uy sar"! = 30.375 =4x15"" => ix 30.375 =1.5""" => 7.59375 =1.5""* Notice that the variable .n now appoars as a power. ‘Taking logarithms of both sides of the equation, log(7.593 75) = log(1.5"=1) = (n— 1) log(1.5) log(7.593 75) So n-1= a ‘That is, =6 So the series 4+ 6 +9 +... + 30.375 has six terms. 338 Urheberrechtlich gi chiitztes Material 8 Sequences and Series 8.3 Geometric Progression Exercise Technique [| Find (i) the next two terms and (i) the nth term of the following geometric progressions: a 1,2,4,, . b 2,6,18,54,... © 4,6,9,134,... d 0.1,0.3,0.9,2.7,.. oy Uqs 4B, 24,12, 20. Ye 2 | Given the geometric progressions 2, 10,50, and 1.3.9, 27,..., Hy, find: a ous bow © % d vw ey Fo We 3 | Given the first term and common ratio of a geometric series, find the sum of the terms indicated: Se Se a a=2r © a=6r ac 4| Find the sums of the following geometric series: a 244+ Op +256 do 2-6418=54+4 b 2-64+18-544-..413122 @ 20-30445 © W+24436+---418225 fF O1+054+25+4---4912.5 Contextual 1 | Find the ninth term of the geometric series 12 +8 +54432+---, correct to throe significant figures. Find the common ratio of a geometric series given that the third term is 6 and the seventh term is 486. 3 | Find the sum of the first ten terms of a geometric series with common ratio 2 and first term 14. 4| Write down the first three terms of 5~ 25(3)". State clearly: a the first term of the GP b the common ratio c the sum of this series, Geo a > 2001.1)" b ¥ soy =m mt Complete Advanced Levol Mathematics: Pure Mathematics 340 8.4 Convergence, Divergence and Oscillation Consider the following soquencas. 3,5,7,9,11, 1,2,1,2,1, 12 4b 2ebeeete Wns Asn increases, whal happens to u,, Vv, and w,? Notice that the first sequence is an AP with a= 3 and d= 2. The ath term, uy, = 2n +1. Asn gets large (nm + 00) ug gets large (ut, > co). This sequence is said to diverge. ‘The second sequence simply oscillates between 1, when nis odd, and 2, when n is even. This sequence is said to be oscillating. A sequence that repeats itself in a regular pattern is periodic. ‘The third sequence is not an AP or GP. Cheek this by trying to find a ference or common ratio. The sequence has nth term As.n gots large (n —+ se), w, gets small (1v,, —+ 0). This sequence is said to converge. The terms of the sequence get closer and closer toa limit of zero. A graph can provide a good illustration of the behaviour of the sequence as n increases. teh vy «@ . . ae . . . —— = id as n The sequence diverges to infinity. The sequence oscillates. The values grow in size as n increases. wy The sequence converges to a fixed value. Intthis case, w, — 0 asin increases. 8 Sequences and Series Example 1 A sequence is defined by the recurrence relation u,, ; = [6— up). Iu, = 2, describe the behaviour of the sequence. Solution Start by finding the first few terms of the sequence. uy uy = |6 —2,| = |6— 2) =4 uy = [6 —u,| = [6-4] =2 uy = [6 —u5|=|6-2|=4 ‘The sequence is oscillating between 2 and 4. So uy = 2 for odd n, and u,, = 4 for even a. Example 2 Describe the behaviour of the GP 20, -10,5,—2.5,1.25,.... Solution ‘This GP has a = 20 and r = —}. The nth term u, = 20 x (—})"*. ‘Asn gets large, u, —> 0. This sequence converges and oscillates. ‘The behaviour of this GP can be illustrated on a graph. The behaviour of a sequence can be ty very important if the terms are being combined to form a series. This can determine if the series will diverge or converge as a large number of terms are added together. Adding an infinite number of terms of a series is known asa sum to infinity. Example 3 Find the sum to infinity of the series 1-34 35- h+.... Urheberrechtlich geschi 3440 Complete Advanced Level Mathematics: Pure Mathematics Solution Notice that this series is created from a GP with a = 1 and r= —]. The nth term u, = 1% (=3)"). As n gets large, ua — 0. This means that the series increases by smaller numbers that oscillate between positive and negative. To find the sum to infinity of the series write down the sum of the first n terms. a(i=r) a -(-9") ayn Se ger aad s0-(-2") Asn = 00, (=)" +0. ‘This means that S,_.. =4 So1—t4+4-4+ i ‘This result can be generalised for any GP in which the common ratio-r is such that -1. laa- ral = ‘Substituting values of r from 1 to.n, where n is a large number, - 3 3 3.3 3.3 3.3 Lhe rs ~G-5)*G-a)G- aye" 3 3 3 +¢-sh)(h-a) Notice that, except for the first and last terms, all terms are repeated with one positive and one negative. These will cancel each other out. a ide es oar ee 0 lea male 2 n+2 Gheck this by summing successive terms of the Asn 00,2. — 0. So the series converges to the value 3. series using a calculator. mee ‘You should find that the 3 sum of the series never Sea as 2 exceeds 3. Urheberrechtlich geschi Complete Advanced Level Mathematics: Pure Mathematics 8.4 Convergence, Divergence and Oscillation Exercise Technique [1] Fina the sum to infinity for a geometric progression with: d [2] Describe the behaviour of the following sequences: a Uy.) = [8 u,| where uy = 10 b cos(30n) c Ing, (In 8)’, (In8)*, d In 1.8,(1n1.8)*, (In 1.8)',... 5 6 Ye = payin +a) f 10 "nin 1) Find the sums to infinity of the following geometric series: a 2434814274 d 100+80+64+ b 243 -814+27- e 20-8+3.2— © 1244+4+... f 20-18+16.2— 4 | For the following geometric progressions find the values of the unknown terms: Contextual 1 | Find the sum to infinity of the goometric series with first term 15. and common ratio 3. 2 | ‘The first term of a goometric series is 24 and the common ratio is — }, Find the sum to infinity. ‘The sum to infinity of a GP is 80 and the first term is 16. Find the common ratio. 344 Urheberrechtlich geschitztes Material ‘8 Sequences and Series [4] Given thet the common ratio of a geothetric ties isf and the sum to Infinity is 92, find the first term, 5 | The sum to infinity of a geometric progression is ten times the first term. Find the common ratio. [6] ‘The sum to infinity of a GP is 81 and the sum of the first four terms is 65. Find the first term and the common ratio. Gerry says, “The sum to infinity of a GP with first term 12 and common ratio 4 is 48". John immediately says that it must be wrong. a Explain why John is correct. b Given that the sum to infinity and the first term are correct, find the common ratio, Urheberrechtlich Complete Advanced Level Mathematics: Pure Mathematics 8.5 The Binomial Theorem and Power Series What does ‘binomial’ mean? ‘Bi’ is Latin, meaning ‘double’ soa binomial is Recall from Chapter an expression with two terms. Some examples of binomials are (a + b)*, that ‘polynomial’ is (3 +x), and (5 —y)', Greek for ‘many term The binomial (a +6)? can be rewritten as a sum of algebraic terms, (a+b) =a? +2ab +07 There are also algebraic expressions for binomials with powers greater than 2 (a+ by! =a' + 3a°b +3ab? +b* (a +by' = (a+b)(a+b)* = (a+b)fa’ + 307b + 3ab? + b*) =a(a' + 3a°b + 3ab* + b*) + b(a® + 3a°b + 3ab* +5") =a‘ +3a'b +3a°b? + ab! + ba* + 3a°b* + 3ab* + bt =a‘ +4a’b + 6a"b* + 4ab? +b* Consider the following structure: (a+b) =a+b (a+ by! =a? + 2ab 4b? (a+by =a? +3a7b + 3ab* +b" a’ +4a°b +6076? + 4ab? +b* (a+by Now write down the coefficients only of the terms in these expansions. As they are writlen down arrange them in a triangular form under each other. Write down the next row in this triangle. Check that each figure is created by adding the numbers directly above it. 4 6 4 4 POS POS PN 1 10 05 1 8 Sequences and Series So (a +b)' = a° + 5a‘ + 100°b* + 10a7b' + Sab! +b” Notice: © the position of the coefficients from Pascal's triangle @ that the powers of a decrease from 5 to 0 and the powers of b increase from 0 to5 @ that adding the powers of a and d in any term gives 5, which is the power on the original binomial. Its useful to remember these points when expanding a binomial. Example 1 Expand: a (24x)* b (3-x)° Solution a Nolice that the index, or power, here is 4. The fourth line of Pascal's triangle is 1 4 6 4 1 ‘These will be the coefficients of the terms in the expansion. Build up the expansion in two steps. Insert powers of the first term, °2', axe 4x2? 6x2? 4x2! 1x2? Insert powers of the second term, ‘x’. Tx atx” 4x2 «xt Gx22xx? ax2txx 1x 2% xxt Now evaluate any numbers raised to a power, recalling that a" = 1 for all values of a. The expansion now becomes (245) = 16 + 92x + 24n? + 8x7 + x4 b. The power here is 5. The fifth line of Pascal's triangle is required. 1 5 10 W 5 4 Insert powers of the first term, °3’. 1x3 5x3! 10x39 19x39? 5x3! 1x3" Insert powers of the second term, ‘~x'. Remember to include 1x 3%x (=x)? 5x34x(-x)' 10x 3" x (-x)? a ee te 10x 3% x (=x)* 5x3 x (=x) 1x3" x (=x)? theberrechtiich geschiitztes ‘gaggrial (Complete Advanced Level Mathomatics: Pura Mathematics Even powers of (—x) are positive and odd powers of (—x) are negative, so (3 —x) = 243 — 405x + 270x* — 90x! + 15x* — x° ‘This technique can be used to expand (a +x)" of (a — x)" for any positive integer n. The problem then becomes finding the values of the coefficients in the nth row of Pascal's triangle, Factorial notation A factorial is a product of consecutive natural numbers, starting at 1. So 3 factorial, written 3!=3 x 2x 1andG!=6x5x4x3x2x1 A table of factorial totals shows how concise this notation can be. How do factorials fit with the coefficients in Pascal's triangle? The terms of Pascal's triangle are known also as binomial coefficients. The third coefficient on the fourth row can be written using factorials as 4) at ig a) 4-3 xa Ina similar way the rth coefficient on the nth row of Pascal's triangle can be writen as Pete The general result for the expansion of (a+ x" is: 3 @e* Pte $x" 4 Learn this result. Example 2 Find the values of: - (0) » @ Solution 6 6 720 s (O-aga gaxa 8 " (2)- a soto. 2)° 27-2) 2st 2x 120 Urheberrechilich geschtitztes Material ‘8 Sequences and Series To calculate the value of binemial coefficients not all factorials need be remembered, The fraction can be cancelled down quickly since each factorial is « product of numbers beginning at 1. Example 3 Without using a calculator, find (3): Solution BX7KOXSXEXIK2K1 (@x2xi)x(Sxdu3x2xi) Notice that 5! is a common factor of both the denominator and the numerator. This means it can be cancelled. B\_8x7x6 a) 3x2xd Now look for other common factors. B) 8x7 (3)= *7 56 One advantage of using factorial notation is that particular terms in an expansion can be found without finding the whole series. Example 4 Find the term involving x” in the expansion of (1 + 2x)", Solution Recall the general result for expanding (a +x)", orayt eat sent (Mart ptt So the term involving x” in this expansion will be 12) (0499 (2): (2x) 12) _ 12x 11x10 _ a Bx2x1 220 So the term in x” is then 220 x 1° x 2°x? = 112 640x” Notice that this provides a quick method of identifying terms in the series. ‘The expansion of binomials using factorial notation for the binomial coefficients is often stated as a mathematical theorem. theberrect schitztes 'S4Giai Complete Advanced Level Mathematics: Pure Mathematics The binomial theorem Notice that (2) and both involve o!. ‘The expansion of the brackets can be written as a series using sigma notation. The binomial coefficients relate clearly to the powers of the terms aand b. The case where a = 1 and bis the variable x is particularly interesting, craters (*hee (t)ee(A)e ee ‘This can be used to find the approximate value of powers of numbers close to 1. Example 5 a Find an approximation correct to four decimal places of (1.01)°. b Find (0.99)* without using a calculator. Solution a This approximation can be made by using the binomial series for (atx® a satare (f+ ()e+(G)ee~ =14+6x4 15x? + 20K? +--+ Now write 1.01 as (1 + 0.01). This is better expressed as (1 +745). Substitute x = 1; into the series expansion, 6 15, 20 **T00 * zoo? * 00° = 140.06 + 0.0015 +0.00002 +.» = 1.06152 (1.01)* fee So, to four decimal places, an approximation for (1.01)° is 1.0615. ‘b Since 0.99 = 1 - 0.01, (0.99)! can be rewritten. as (1 —0.01)*, Use the binomial series for (1 —x)* with x = 45 (Q-xyteas (i) (ew + (Gor +(x) = 14x + 6x? — 4x? +204 8 Sequences and Series 4, 6 eo 100 * 300% 100" ~ 100" — 0.04 + 0.0006 — 0.000 004 + 0.00000001 = 0.960596 01 Power series ‘A power series is a polynomial. The terms of the series are powers of a variable, usually x , multiplied by a coefficient. Lox aS Gy Fayx + aX? Power series can be divergent or canvergent. When tho series is convergent there is usually a restriction on the range of values of the variable x. A power series can be created from a binomial expansion where the power is fractional and not an integer. The expansion is now written slightly differently to how it was before, because the binomial coefficient, ("), is nonsensical when n is a fraction. When n is negative or fractional, the binomial series for (1 +x)" is given by result. Notice that this series will be infinite if m is negative or fractional because the numerator will never contain the factor (n — n). This power series will converge provided ~1 < x < 1. This condition is particularly important because it means that the series is not valid for all values of x. Example 6 Find the first five terms in the binomial expansion o: range of values for which it is valid, Solution Applying the law of indices, 1 = (1 =x)" This isa I=x binomial with power —1. The series can be found by substituting n =—1 . into the binomial expansion (carefully checking the sign of x). i a(n =1)x? n(n = 1)(n-=2)x° rt aE 0 (1-2 $m MANA HAN aN? (14x)"=14 004 +e )(=2)(=3)(=4)(=x)* ry a“ Complete Advanced Level Mathematics: Pura Mathematics 2x? 6x? 2axt Check what happens Sates te tae when x =}andx =: mapa pad ala ce Notice the need for t restriction [x| <1, This series is valid for —1 < x < 1; that is, |x| <1. Example 7 Find the first four terms of the binomial series for \/1 + x. State the range of values of x for which the series converges. ‘Solution VI#x = (1-+x)!. This is a binomial, with power }. (1+x)" =14n0+2 1 So (1+x)! = 1440+ Sl4dx-be tae —-.. =14dx-be ede. ‘This series is valid for jx| <1. ‘The binomial series can be used to provide a power series expansion for rational functions. Example 8 Write ace as a series of ascending powers of x, up to and including the term in x". State the range of values of x for which the series is valid. ‘Solution ‘Once the rational function is written as a product the application of the binomial theorem can be seen. ‘Expand (1+ 2x)~* as a power series, then multiply this result by (1 —x). Check that (1+ 2x)* =1— ax 4 40x* — 160x7 +... Then (1—x)(1+ 2x)“ = (1 —x)(1 — Bx + 40x? — 160x) +--+) =1- 9x + 48x* — 200x7 + --- This series is valid for —1 < 2x < 1; that is |x] < }. '8 Sequences and Series: 8.5 The Binomial Theorem and Power Series Exercise Technique 1 | Find the expansion of the following binomials using Pascal's triangle: a (1+x)* b (3-x)! © (5-x)* d (2+x)5 2 | Find the values of the following binomial coefficients: *@ 8 @ «© G@) « () 3 | Expand the following using the binomial theorem: a (1+x) b (4-x)* «© (2-ax)9 do (w-3* 4 | Write the following as series of ascending powers of x, up to and including, the term in x": a (1-x)* ob @-x) © +x)? d (2-ay)* 5 | Find the range of values of x for which the series expansions of the following are valid: a (14x)! b (3-2x)) oc Wieix ds (43x)? {6 | Find the first four terms of the binomial series for the following functions: a (tx? ob WFR oc View d eS Contextual [1] Find the binomial expansion of (4 + x)*. When is this expansion valid? 2 | Find the term involving x’ in the expansion of (1 +.2x)'". 3 | Find the coefficient of the x” term in the expansion of (1 — 8x)"*. {4} the expansion of (x ~ 3)" has a term that does not contain x. State the value of this term. 5 | Find the value of (1.01)’ to four decimal places using the binomial expansion of (1+ x)". [6 | expand Y= as a series of ascending powers of x, up to and including the term in x*. State the range of values of x for which the expansion is valid. ‘Complete Advanced Level Mathematics: Pure Mathematics Consolidation Exercise A 1 | Find the sum of the arithmetic series 2+ 5 +8 +---+998. (NEAB) [2] the tenth term of the arithmetic progression is zero and the sum of the first 10 terms is 15. a Find the first term and the common difference. b How many more terms must be added for the sum of the arithmetic progression to be zero? | [3] A geometric series has third term 27 and sixth term 8. Show that the common ratio of the series is 8. Find the first term of the series. Find the sum to infinity of the series. Find, to three significant figures, the difference between the sum of the first 10 terms of the series and the sum to infinity of the series. (ULEAC) aa ee [4] rina the sum ofthe series with first term 11 and last term 40, in which ch term, after the first, rds the ious t by}. each torm, after the fist, exceeds the previous term by 3. eas [BB] the nth term of a sequence is ty, where tig = 98(8)" @ = 2,2,3,--+0 2 Pind the values of w, and u;. Giving your answers to three significant figures, calculate: b thevalue of uz, « Yu a Find the sum to infinity of the series whose first term is u, and whose nth term is u,. (UEEAC) [6] rind, in the simplest form, the first three terms in the expansion of (1 +21), in ascending powers of t, where |t] < J. Riis) [77] two sequences 1, t,43,...are dofined as follows: Sequence A: uy =2 tty = 3p Sequence B: m= 2 une 8 Sequences and Series For each of the two sequences, find the values of ua, ty. th, and ts. State for each sequence whether it is convergent, divergent or oscillating. (NEAB) [8] Given that |x| < 1, expand VT 2x as a series of ascending powers of x, up to and including the term in x’, simplifying the coefficients. (UGLES) [9] Find the term in x™ in the binomial expansion of (x + 2x7)", (OCSEB) Use the binomial theorem to give the expansion of (x + 7)". b Hence obtain the expansion of (x +2)*, expressing each term ina simplified form. ¢ Find the coefficient of x? in the expansion of (1+ x7)(x +3). (NEAB) Exercise B [1] Find the sum of the arithmetic sories 8-+7-+12-+---+79, [2 | sicten’s father gives her a loan of £10 800 to buy a car. The loan is to be repaid by 12 unequal monthly instalments, starting with an initial payment of £A in the first month. There are no interest charges on the: remaining debt but the instalments increase by £60 per month so that the second monthly payment is £(A + 60), the third monthly payment is £{A + 120) and so on. a Show that 4 = 570, b Find an expression in terms of n for the remaining debt immediately after Helen makes her nth payment. Give your answer in a fully factorised form, (NEAB) 8 [3] a1 Express G—ayegp pay it Battie fractions. ii Hence find the sum of n terms of the series a ee xb 3x7 RG b Asequence of integers uy, t2, wy is defined by uy = Sand Uy44 = 3u, — 2" for n > 1, Use this definition to find uz and us. {OCSEB) Complete Advanced Level Mathematics: Pure Mathematics 4} a Thesixth term of a geometric progression is 6.075; the fifth term is 4.05. Calculate: i the common ratio the first term. iii the 30th term (correct to three significant figures). b State, with a reason, whether the geometric series In3 + (Ins)? + (Inay! + is divergent or convergent. ¢ Theseries S is given by In 3 + In(3*) + In(33) + +--+ In(3®) = s In(3‘) it Show that $ is an arithmetic series whose sum is approximately 511. 4) Show that 57 In(ar) = min’ + Inn}. mi (OCSEB) [5] a The first term of an arithmetic progression is 9 and the seventh term is three times the second term. Find the common difference and the sum of the first 32 terms of the arithmetic progression. b The first torm of a geometric progression is 81 and the fourth term is 24, Find the common ratio and the sum to infinity of the geometric progression. (wyEC) 6 | A sequence is defined inductively as follows: uw, =2 Uy s1 = units digit of 2u, (that is, the remainder when 2u, is divided by 10) Write down the values of u, for i= 1,2,3,4,5 and show that u, = 4. State whether or not this sequence is periodic or convergent c Find the units digit of 2”, explaining how you obtain your answer. (OGSEB) ce 7 | The nth term, u,, of four sequences is defined below. For each sequence decide whether it is convergent, divergent to +50, divergent to —co or oscillating. For each convergent sequence, state the limit to which it tends asin 00, a ieee b m=8-4 ; 3n 4 eso, sin(inx) qa 8 Sequences and Series Applications and Activities Loan repayments < The sum of a geometric series can be used to find: @ the time taken to repay a loan given a debt, interest rate and repayment instalments: @ the repayment instalment given a debt, interest rate and repayment term, You borrow £1000 at 14% APR and pay back £300 per year, How long will it take to repay the debt? You barrow £1000 at 149% APR and want to repay the debt in three years. What should your monthly instalment be? Investigate credit terms for finance companies and check the repayment tables for the quoted APR. Summary Assequenee is a set of numbers occurring in order. When the lerms of a sequence are added together, a series is formed. A sequence can be described using an algebraic term for u, or by a recurrence relation. ‘A series can be written using sigma notation, which uses the Greek letter ¥ to represent summation over a number of terms. The nth term of an AP is [RBBQHENG)d, where a is the first term, and d is the common difference. ‘The sum ton terms of an arithmetic series can be written in equivalent forms Bas’ where / is the last term. The nth term of a GP is MHBBE? where ais the first term and ris the common ratio. 357, ‘Complete Advanced Level Mathematics: Pure Mathematics @ The sum to a terms of a geometric series with first term a and common ratio r is given by | @ A soquence is said to diverge if u, —» co as n> co, converge if u, —* a limit as n —+ oc, and be periodic if it repeats itself in a regular pattern. @ GPs with a common ratio r such that |r| < 1 create convergent infinite series whore S, ~ Sq, asin — 00, and @ The coefficients in a binomial expansion, (a +b)", can be written down ‘using Pascal's triangle, @ A factorial is a product of consecutive natural numbers; so 3! = 3 x 2x1 and 0! = 1 by definition. @ The binomial theorem states that, for positive integral values of n (rm € Z*) says: @ The binomial theorem can be used to write a binomial series for integer, negative and rational pawers. This power series will have a condition on the values for whieh it is convergent. 358 urh jeschittztes Material 9 Integration | What you need to know How to differentiate standard functions. How to use the laws of indices. How to find the value of an algebraic expression. How to sketch graphs. How to use sigma notation. Review Find: 2 fotin dX eosx) b A gst —ox+2) e A Gsinzx) c &(#-2) f Se-x +3e0850) Write each of the following expressions as Ax" or (Ax" + Bx"), where A and B are constants. 1 3x? xx? a ae ¥ a 5 aG 5 f yea +y%) Find the value of each of the following expressions for the given value of x: a 3x? +5x’ +x, whenx =2 bo 3xt—x' x, whenx ec wtoxti4,whenx=9 dot,whenx=8 -2 Sketch the graphs of: aoyet b do y=sinx e Simplify the following using sigma notation: a 14243 bo 14449416 +25 c 2444648 do 447410418 Complete Advanced Lavel Mathematics: Pure Mathematics 9.1 Indefinite Integration Integration can be described as a reverse operation. Consider some familiar reverse operations. Add ‘Subtract aus é uy Hy ‘Multiply Divide aN ee NN = Gube ‘Cube-root Nees i xy Mathematicians studying what we now call differentiation and integration realised that these processes were closely related. Leibniz (1646-1716) and Isaac Newton (1643-1727) independently discovered that integration is the reverse of differentiation. This fact is called the fundamental theorem of calculus. [isnot and statesman, He pulsed is work _ cai in 1684, Differentiation Integration ifforentiate: fs ulin Ba luce power by re > divide by ths number Tho principle of the reverse process will be used to integrate powers of x by increasing the power by 1 and then dividing the new term by this number ~ the reverse of differentiation. fe dy=1x" 4 Differentiating }x! gives x*. Check this by differentiating the answer; it should reverse the integration process. ‘dx’ means integrate ‘with respect to! the But note that gi(1x? +1) = 3 «4x? +0 =x? vanahlaw So [f(x)dx means 1D eee ons LGC) H ee Re integrate the integra and soon. f(x) with respect to: Integration | This means that any of the expressions }x*,4x* +1, 4x7 +2,... could be the answer to [xdx. Differentiation of each of these expressions gives x’. Since there is no definite answer the expression J x*dk is called an indefinite integral. The complete answer is fra = 127 +c, where c Is a constant. All indefinite integrals have ‘+c’ at the end where c is a constant. The following table illustrates the basic principles of integration. fexsfidr [rte [tatq=ato=1 zs S5as arte |&Gete)—s+0-5 ‘where & i any number Jade pise [kde + d=}xaxto~x as, frede Ptte |£Getqatxax+o=7% [where is any-constant fade yse [dgetaixat pose Joe ds qtte [£det da jxae tone except Jxlidx de% se |LGe? + aig? t0=x Jt ax+6)dx |p pd | LG = 2 + 6x40) = = +8rte a de Summary of the basic principles © RBREK ec, where kis any number © BBE ax, whore kis any numbor @ BRR, where nis any number except -1 © ABER iac+ seve Example 1 Find: a Ix(x—3)de b f(a —3Vx)dx z J 1 ) ae Solution a The technique here is to expand the brackets first, and then integrate each term separately. Jao 3)dx= for ax) dx =i -axitic she -Bi tc 4 Always add a constant for an indefinite integral, If the variable was ¢, not x, we would ‘integrate f° with respect tot’; fedatPre ‘Complete Advanced Level Mathematics: Pure Mathematics b To integrate the square root, first express it in powers of x. Ji--avaar= fir ax ax Rocall that yie= x! ar-axtiee ue frar= 2 : 4 Check the answer by =x-titec differentiation, ¢ First express the integrand as powers of x. [G)*-|G-2)* “|(-3) = Je =x yar +c¢ Remember to add a constant for an indefinite integral. = 3x =xt+hee x Example 2 ‘The gradient of a curve y = f(x) is given by $f = (2 — x)(2 — 3x). Find the equation of the curve, given that it passes through the point (3,2). Solution To find y from %, reverse the differentiation by using integration. y= Je =x)(2 - ax)dx = [(¢—ax+axtyae ax 3x? =a sp te ysa-atextte This equation represents a whole family of curves. The graphs can be drawn on a graphical calculator using different values of €. The graphs show that there are many functions with a gradient of (2 — x)(2— 3x), ‘However, there is only one graph that passes through the point (3, 2). Check that the value of ¢ must be -1. s Material 9 Integration | An alternative to the graphical method is to use algebra to find, or check, the value of c, Substituting the values of x = 3 and y = 2 into the expression for y, = = 2=12-36+27 +6 = -1=6¢ So the equation of the curve is y =x! — 4x* + 4x —1, Integration and mechanics In Chapter 5, mechanics was used as a context for differentiation, Recall that if's = distance travelled in time ¢ then the velocity is given by v = 4s, and the acceleration is given by a = 4. Using integration as the reverse process to differentiation it is now possible to work backwards. This means that we can find v if we know a, and ean find s if we know v. The new equations become: v= fear and s= frat Example 3 An aircraft taxies at a constant speed of 7.5m s~* to the start of a straight runway. It then begins to accelerate. Ils acceleration after { seconds is given by a = 21-1, a Find an expression for the velocity at time t seconds. b An observer estimates that 10 seconds after leaving the start of the runway the plane takes off. What distance has il travelled along the runway in that time? Solution Draw a diagram to illustrate the situation. y=7.5mst —— start of timing after t seconds 5 = distance in metres Urheberrechtlich geschitzte®@Bterial (Complete Advanced Level Mathematics: Pura Mathematics, == 2-2? We know that v = fadt, so we integrate the acceleration to find the velocity. v= fadt = fiet—jeyae saxdP-ixtite =t-itte ‘There is enough information in the question ta calculate c. At the start of the straight runway the speed of the aircraft is 7.5ms~'. So when t = 0, v = 7.5, Substitute these values into the expression for v. 75=0-0+¢ Bae veP-df+R b v=$=f-af4R So s=fvdt = f= ge +a =iP-axit+tric = ph ge+Brec The distance is measured from the start of the runway. So when ¢ = 0, = 0. Substitute these into the expression for s. 0=0-0+0+c pee s=iP-dh+te To find the distance covered in the first 10 seconds substitute t = 10 into this expression for s. s=4x<10’— 4x 10+ x 10 | Sos=2413m <4 Always include the appropriate unil. Note the exact answ Urheberrechttich 9 Integration | 9.1 Indefinite Integration Exercise Technique 1 | Use integration notation to describe each of the following statements: a integrate 2x" — 3x +1 with respect tox ‘b integrate e™ with respect tot ¢ integrate cos @ with respect to 0. [2] integrate cach of the following with respect tox: a3 b 2x=5 c xt 4x=7 d= x(2— 3x) e (x-3)(x+5) f (x+1y [3] rina: 1 12, a fer e feu i f100x"2dx 1 b si0dr f fatac i Jae e f(@+a)dt g fsrdr ko farddy a Syed h fae [4] Fina; a fat-sx+2)de b f(2t—s)*de Jx(2 +x) dx d fa-eyae Jot iix-a)de f fraxtdx [B] rina: [Es )= + \Ge= = [Gres a [oes ae ‘ JC) f Jie+oveae [6] itr) =ox, and f(a) = 4, find f(x). ° . ° [77] w scx) ax = g(x), what is the connection between g¢x) and f(x)? Contextual {1] ‘The velocity of a particle, at time t, is given by v = ¢+ 3. Find an expression for the distance, s, travelled by the particle at time ¢, given that s=Owhont =0. Complete Advanced Level Mathematics: Pure Mathematics [2] copy and complete the following, where each ‘s represents a missing term: a f(2x" — 10x" b ‘ d e f 8 [3B] teat) = foxtx +a) de: a write down g'(x) and solve g'(x) =0 b find g(x) given that g(—1) = 5. | The gradient of a curve at any point is given by $¢ = 4 ~ 2x. a Explain why the curve has a maximum value when x = 2. b Ifthe maximum value of the curve is 1 find the equation of the curve, {5| If g'(x) = 1 — 4x’, and (2, —13) isa point on the curve g(x), find g(x). [6] the gradient at any point (x,y) on a curve is given by df = — 4, Ifthe curve passes through the point (2,33), find the equation of the curve. (7 Attain starts from rest at station A and 24 minutes later stops at station B. Its velocity, t seconds after starting. is given by v = 0.6¢ — 0.004¢" ms~*. a Find an expression for the distance travelled from A. b Find the distance between the two stations. 8 | A particle leaves a fixed point O with a velocity of 20 ms”. It travels ina straight line and its acceleration after t seconds is given by a=(14-8t)ms* a Find an expression for the velocity in terms of t. b Find an expression for the displacement of the particle after leaving O. Urheberrechtlich geschtitztes Material 9 Integration | 9.2 The Area Under a Graph Integration is used for summation; usually of lengths, areas or volumes. The S-shaped symbol was devised by Leibniz because integration represents a sum, It was perceived as the sum of the areas of an infinite number of rectangles of height f(x) and infinitesimally small width dx. So each rectangle would be thinner than a pencil line and its area would be f(x) dx. We can find the area under the curve y = x?, between.x = 2 and x = 3, using Letbniz's idea that [ f(x) dv is the sum of the areas of many thin rectangles. Consider splitting the area into a series of thin rectangles. yer! yer? es Urheberrechtlich geschitt Complete Advanced Level Mathematics: Pure Mathematics. When the area is split into two rectangles the width of each is 0.5, The height of the first rectangle is f(2.25) and the height of the second rectangle as (2.75). The two distinct areas are then 0.5 x f(2.25) and 0.5 x f(2.75). Calculate these two values: 05 * f(2.25) = 0.5 x 5.0625 = 2.53125 05 * [(2.75) = 0.5 x 7.5625 = 3.78125 ‘The total area under the curve is estimated to be 2.53125 + 3.78125 = 6.3125 Repeat the process, this time splitting the area into four rectangles. The Width of each rectangle is then 0.25, and their heights are f(2.125), (2.375), f(2.625) and (2.875). Check that the total area is now 6.328 125, This process could be repeated many times, In each case the width of each rectangle gets much smaller. As the width becomes infinitesimally small (as dx —+ 0), the heights become f(x) for all values of x between x = 2 and x = 3. The total area of all the rectangles then becomes a better approximation to the exact area under the curve. The area can be calculated as ¥ esx where this exprossion represents the sum of the aroas of each ractangle with height f(x) and width éx, between x = 2 and x=3. In general, adding the areas of all these rectangles gives the area under the graph belwoon.x =a and x = bas [e (x) dx 4 Learn this result. We call ‘b’ the upper limit of the integral and ‘a’ the lower limit of the integral. An integral with limits is a definite integral. The shaded area under the curve y = x* is given by? x? dx. [pax = [28] < Square brackets are used to separate the ie limits from the integrated expression. So the area under the curve is 64 square units, Compare this to the result for four rectangles. Putting x = 3 and x = gives the area under! graph between x =0 ‘x =3,and x =Oand respectively. Subtrac the second from the | gives the area under | graph between x =2 3. ‘9 Integration | y= b= aer=2) The graph shows the curve y= (x — 1)(x— 2). a Find ff (x —1)(x— 2) dx and f(x — 1)(x— 2) dx. Comment on your answer. b What would you expect (3(x — 1)(x — 2) dx to equal? © Find the area of the shaded region. Solution a To use integration first express y as a polynomial inx. ¥ = (x-1)(e- 2) =x? - 3x42 1 1 [ie m2) ae = | (x? — 3x +.2)dx lo 0 = [be Be +243 = (}-}+2)-(0-0+0) ee) x8=}x442%x2)=()=342) Hate F ‘The value of the integral between x = 1 and x = 2 is negative. This is ‘because the region bounded by the curve, x = 1, x = 2 and the x-axis ies below the x-axis. b Since integration is a summation process we might expect that fe- 1-2) a= fie nox 2)dr+ fe —1)6e—2) ax o 0 h tdi} Urheberrechitic Complete Advanced Level Mathematics: Pure Mathematics 370 c Looking at the sketch of the function, we can avoid adding the areas algebraically. The graph shows that part of the curve is, below the x-axis, and for this region the integral is negative. Adding the integrals, as in b, would give too small a result for the area. Therefore, total shaded area =$+!=8—=1 square unit. Example 2 Yh ho) =hat g(a) = 6 + 2N2" (-8, 16) (6, 16) oO x Find the area of shaded region enclosed between g(x) = 8 + 2x) and (xy = 4x7, Solution ‘Tho atea is symmetrical, with the y-axis as the line of symmetry. So we can find the area of the whole shaded region by doubling the area of the part of the region lying in the first quadrant. Call the area under the curve (x) area A, and the area under the curve h(x) area B. v fs) = bt (x) = 8 + 2x29 area d= f+ 2x) de area B= [tax = if xa = foveal rita Mo = AU 512) — 0) = [ax+fat]f = 3 ue wa 4 = (64+! x 32)-(0+0) = 2 Urheberrechtlict A sketch of the curve will always give you: indication of whethe the curve crasses the acaxis, IF t doos, you ‘must calculate the ar of the regions above below the axis x=Oandx=8. Recall that J¥f(x) dx =k ff(x)d sschilztes Material 8 Integration | So the area of the shaded region = 2 x (urea A — area B) =2« (pip) -yE Area of the shaded region = 119.47 square units (2 d.p.). The graph shows a sketch of the curve y = x(x ~ 2)?. a Find the coordinates of P. b Find the shaded area. Solution a y=x(x-2)* = x(x? — 4x +4) =x) 4x? + dx = £nax"-eraa Pisa local maximum point, so { =O at P. Thal is, 3x* — 8x +4 = 0. This is a quadratic equation. The coefficient of x? is larger than 1, so factorise and solve it using PAFF: 3x? — Bx + 4 = 3x? — 2x 6x +4 = x(Bx — 2) — 2(3x — 2) = (Bx — 2)(x — 2) When 3x7 — ax +4 = 0, (3x —2)(x- 2) = 0. Sox =Zorx=2. ‘Substituting each of these coordinates into the original equation, we find that the coordinates of P are (7,32) and the coordinates of Q are Check these against the (2,0). sketch of the graph. chilegz ql Complete Advanced Level Mathematics: Pure Mathematics b The area of the shaded region can be found by subtracting the area of the region below the graph between x = 0 and x = 3 from the area of the rectangle with corners (0,0), (0,22), P (2,32) and @,0). u ‘The area of the rectangle is § x 8 = St ‘The area under the graph y = x(x — 2)* between x = 0 and x = is given by area flr at + anya = (pt -$e te =(ixM-4x£+2x4)- (0-040) = =u i ‘So the area of the original shaded region = {4 2 square units. 372 Urheberrechtlich geschitztes Complete Advanced Level Mathematics: Pure Mathematics 4| Use integration to find the area of the shaded region in each of the following: 5 | evaluate the following definite integrals: a [ie -9e +4 na b fr oax & I (10-2 + 6x2) dx a [ (1 10x4) dx ea -2 {6 | The graph shows a sketch of the curve y = (2 = x)(x ~ 4). a Write down the coordinates of y PandQ. b Explain why it is necessary to Y= e-4) evaluate two separate integrals in order to calculate the area of the shaded region. © Calculate the area of the shaded region. ‘7 | This diagram shows the curve y= x? — 8x4 3.and the line yoo. a Find the value of fia? — 3x +3) dx. Find the value of f¥(2x —1) dx. ¢ Hence find the area of the shaded region, 374 Urheberrechtlich geschitztes Material | 9 Integration | 4 Find the area ofthis shaded region. y= x(3-x) b Now find the area of this pattern, Evaluate the following integrals: a [ae b [ioa-nae © [Se d [ister Contextual / [4] Find the area of the shaded region in each of the following: © yex(s—x) ‘The diagram shows the vertical section through a tunnel 14 m long. The roof is an are modelled by the equation ¥ =6 —0.08x* — 0.0006x', a Find the area of the cross-section, b Find the volume of the tunnel. Urheberrechtlich geschiitztes S#Brial 9 Integration | 9.3 The Area Between the Vertical Axis and a Curve Exercise ' ‘Technique [1] tthe graph shows the line y= x41. a Find.x interms of y. ‘b Use integration to find the sum of an infinite number of horizontal rectangles that describe the shaded ‘The graph shows the curve y = }x4. Express xin torms of y and thon calculate the area of the shaded region, ‘The graph shows the curve y = x’. Express x in terms of y, Hence calculate the area of the shaded region, correct to two decimal places. [4] the curve shows the graph of y ==). Express x in terms ‘ofy. Hence calculate the exact area of the shaded region. wos yo Net Urheberrechtlich geschitztes SP@rial | Complete Advanced Level Mathematics: Pure Mathematics 9.4 The Area Between Two Curves ya fle) ‘The shaded region is enclosed between the graphs of y = f(x) and ¥ = a(x). Divide the area into vertical strips. Notice that when f(x) > g(x) the length of each strip is f(x) — g(x). This is the case for all values of x between x =a and x =b. Adding together the areas of all the thin rectangular strips gives the area of the shaded region. The area between the two curves is 2 (fx) = g(x)) dx. As 6x — 0 and the width of each strip ee extremely small, the area can be calculated by integration. Example 1 Find the area enclosed between the curves y =x" +2and y= 3x? —2, Solution Since y = x" +2 is greater than y = 3x* — 2 between x = —1 and x = 2, uso (F(x) = a(x) dx where f(x) =x" +2 and g(x) = 3x? — 2. 9 Integration | ‘The solid formed by the rotation volume = [sta First find an expression for y* in terms of x. yer 41 = Pa(t+iaxtsarta 1 So volume of revolution = | tees 2x4 + 1) de 1 =f (xt + 2x7 + 1) de Ly =ar(bbtixt tall, {+ 9 +1) — ( =h+®) = i-th Notice that this exact result can be approximated to 11.7 cubic units (3 s.f). Rotation about the y-axis Find the volume generated v= se) when the shaded area bounded f byy =f(x) and the y-axis, from thin ectangle of © y=atoy =b,is rotated about eee the y-axis. The width of the approximate roctanglo, dy, is a small measurement along the y-axis. Rotating the shaded area about the y-axis forms a solid. u fl ‘The rectangle becomes a : os thin horizontal dise, of depth éy and radius x. ( 2 * ‘The volume of the thin disc is ax* Sy. Urheberrechilich geschiitzte®BBioria ‘Complete Advanced Level Mathematics: Pure Mathematics ‘The volume of the solid of revolution is found boy adding together the volumes of al the discs. Volume of solid revolution = yy xx” dy, where dy is infinitesimally small, 7=* ‘We use integration fo sum the volumes of the discs. Compare this result with the formula for rotation about the x-axis. In particular notice the y"dx and x*dy terms. Example 2 Find the volume of the solid generated by rotating the shaded region about the y-axis. Solution Make a sketch of the solid. Itis a cylinder with a bowl shape removed, so we can find its volume by subtracting the volume of the bow! shape from the volume of the cylinder. volume of cylinder = n"h =axix2 = 2n cubic units ‘The coordinates of the curve at each edge of the shaded region are (0,1) and (1,2). so the bow! shaped solid is obtained by rotating the region between the y-axis and the curve between y = Land y = 2 through 360°, 9 Integration | ‘To use the equation that we know for volumes of revolution around the y-axis, we need to find the equation of the curve in terms of x*, yexte1 a y-tex? 2 volume -| at dy 5 = fxw-nar [yh =aldx4-2)-@x1-2)] = }rcubic units So the required volume = 2x —42 = 3neubic units. Example 3 Find the volume generated when the region bounded by the curve y = 9", the y-axis, and the lines y = 1 and y = 3 is rolated through four right angles about the y-axis, Solution First, sketch the curve and the a pe solid of revolution formed by 3 cS the rotation about the y-axis, 1 Veh ae rb volume of revolution = [, ax? dy. =a [stay == [vay 2 oly =a(p) =al@x 3f)- x 1)] 3x = Foal-1 = 9.88 cubic units (3 s.f) Urheberrechttich Complete Advanced Level Mathematics: Pure Mathematics 9.5 Volumes of Revolution Exercise Technique [4] rit the volume ot cach of the'selide firmed by nitating the following areas about the x-axis, showing clearly the necessary integration: av by, yak Mo) |. dal Df y=o-x? ae wae 3 : Cyt pax dyh yaye — ot Sketch the solid formed whon the area under the curve y= x? +1, from x= 1 tox = 2, is rolated through four right angles about the x-axis. Calculate the volume of this solid. Find the volume of the solid formed when the shaded area is rotated about the x-axis. Urheberrechilich geschiitztes Material | ous ‘9 Integration | Contextual A ball has a diameter of 6cm, Its volume can be calculated using integration. The equation of a circle of radius 9 is x* +? = 9. Find the volume of the hemisphere formed when the area under the curve between x= Oand.x 3 is rotated through one complete turn abaut the x-axis. Hence find the volume of the ball. Check this result using the formula for volume of a sphere. A small vase is formed when the shaded region is rotated through one complete tum about the x-axis. The arc AB is the part of the curve 4x? +1 between x = ~1 and x = 1. The arc BC is the part of the curve vx between x = 1 and.x = 9. Find the volume of the vase. 3 H A funnel is formed when the shaded region, bounded by the line y-=x~1, the y-axis, and the lines y = 0 and y = 4, is rotated through one complete turn about the y-axis. Find the volume of the funnel. Usheberrechitich geschiitztes ia 9 Integration | © Recall that # (sin(ax +b) = acos(ax +b) ‘This means [ cos(ax +b) dx = 3sin(ax +b) +c d__ fsin?xdx is not one of the standard integrals. Use the double angle formula from Chapter 3 to rewrite sin? x in terms of a multiple angle. cos 2x = 1~2sin? x = 2sin’x =1-cos2x = sintx = }—1eos2x Now foin?xax = Jig teos2x) ax - 1 fla cosan dx = }(x—}sin2x) +¢ = ix -dsinax+e Example 2 Find the area enclosed by the x-axis, y-axis and the curve y=1+sinx. Solution yaitsinx ar 2 Sketch the curve to establish the limits. eas fe + sin x) dx = [x coax)?” 7 F— c08($*) |-(0-cos0) 3n =F 0-041 3n st echilich geschitzte Merial ‘Complete Advanced Level Mathematics: Pure Mathematics Example 3 12 Evaluate [ (cos 4x — 6sin 2x) dx. Solution [cosa Gsin 2x) dx = [Jin 4x8 x 2(—cos 2x)" = [}sin4x + 3 cos 2x]5/ ~ [ien(5) +2-m(8)] ene 83 1 Ig _ v3 = -3 Example 4 Find the volume of the solid formed when the shaded region is rotated through 360° about the x-axis. y= 2sin 2 Since the solid has been formed by a rotation about the x-axis, use volume = [ ny? dx ‘We know that y = 2sin 2x, soy? = 4 sin? 2x. From the graph, the limits are a = 0 and b = 47. rolaiaa [rts siat axa Taking the constant multipliers out of the integrand. sin? 2x dx 392 Urheberrechtlich geschitztes Material 9 Integration | ‘The integrand contains sin’ 2x, which we can't integrate directly, We must use the multiple angle formula to write it in terms of double angles. cos 2A = 1~2sin*A = sin*A=}—1cos2A Putting A = 2x gives sin’ 2x =} ~ }cos4x Ber volume = 4x "sin? axdx lo = an fq —foosanyax 4 =a" (1 = cosax) dx = 2afe— }sin ax] = 2a[(j—fsing) — (0 —fsin0)] = tnx (f-0-0+0) = 42? cubic units Urhebertechiich geschitzteS %Bbaria 9 Integration | [77] Find the arons ofthe following shaded regions: b feos?xdx ‘The graph shows y = sinx. im a Explain why [ sinxde =0, 2 2 b What area is represented by f sin xdx? Evaluate this integral. © Use your answer to b to calculate the area of the shaded region. (2] A particle moves ina straight line. Its velocity at time ¢ is given by 2.cos3t. Find an expression for the displacement, s, after ¢ seconds, if 1395, Urheberrechtlich geschiitzt 9 Integration | Consolidation Exercise A [1] Fina: - a f(2x-3)Fdx ‘b the equation fora curve, y =f(x), for which & = 6x — 1 and which passes through the point (1,9). [2] the sketch shows the graph of y = a(3—). a Find Jx(3-x)dx. ‘b Hence calculate the area between the curve and the x-axis. ‘The sketch shows a graph of the curve y = 5x4 —x', a Find and calculate the coordinates of the stationary points. b Calculate the area of the shaded region enclosed between the curve and the oraxis, ‘The diagram shows part of the curve y = 7 + 6x —x7. Find: a the coordinates of the points P, Q, Rand 8. b the area of each shaded region. (UCLES) Bia Urheberrechtlich gesch Complete Advanced Level Mathematics: Pure Mathematics ‘The sketch shows the graph of y =x" ~ x together with the tangent to the curve at the point A (1,0). a Use differentiation to find the equation of the tangent to the curve at ‘Aand verify that the point B where the tangent cuts the curve again has coordinates (—2, -6). b Use integration to find the area of the region bounded by the curve and the tangent (shaded in the diagram), giving your answer as a fraction in its lowest terms. (UCLES) G | a Find the area of the region enclosed by the curve y = #4, the x-axis and the lines x = 1 and x = 3. b The area of the region enclosed by y = 44, the x-axis and the lines x = 2 and x = a, where a > 2, is 3.6 units squared. Find the value of a. (UCLES) ‘The diagram shows part of the curve y ~ JT. Calculate the volume formed when the shaded region is rotated through 360° about the x-axis. Find J(2cosx —3 sinx) dx. Find [(1— 6sin 3x) de. /2 Evaluate ["" 2 oos2td, + cosx. Find the area enclosed between the =Otox=a. an oe ‘Sketch the curve y = axes and curve from 398 Urheberrechtlich ge 9 Integration | “Och. ‘The shape of a glass paperweight is that of the solid formed by rotating about the y-axis the part of the curve 3x” + 2y* — 12y = 32 for which y 20 (in centimetres), a Verify that the paperweight is 8cm high. ‘b Calculate the volume of the paperweight. You may leave = in your answer. (SMP) Show that (cosx —sinx)* = 1—sin2x. 0 The diagram shows part of the curve y = cos x = sin.x. Find the volume generated when the shaded region is rotated through a complete revolution about the x-axis. (UCLES) Exercise B [A] rina | (ve+H Jas b Hence evaluate f (a+8 (NEAB) [2] a Acurve satisfies # = (ax -1)*. Given that the curve passes through (1,4), find its equation. b A particle moves in a straight line so that ¢ seconds after leaving a fixed point A, its velocity v (ms~') is given by y = @ — 4t+3, Find the displacement, s metres, for the particle at ¢ seconds, ifs = 0 when t=0. Utheberrechtlich geschitt ‘Complete Advanced Level Mathematics: Pure Mathematics [3] the sketch shows the curve y = x°. Find @ ifthe area of the shaded region is 1 square unit. [4] the diagram shows the curve 2°, and the lines x = 4 and y=8—2x. a Find the coordinates of A. Find the area of the shaded region. [5] a Onthe same graph, sketch the line with equation y = x +1 and the curve with equation y = 5x — x? +6. Values of x should be taken from xX = —2tox = +8, Shade in the region between the line and the curve, b Calculate the points of intersection of the line y =x +1 and the curve y= Sx-27 +6. ¢ — Use integration to calculate the area of the region that you shaded in a. (MEI) Accup has the shape made by rotating the graph of y = 3x%, for 0 1 =An(m41)(2n 41), show that the aroa of all the a rectangles can be wrilten 7 (h=1)(2h-1). b Investigate the total area under the curve y = x* between x = 0 and x = 1 for different values of A. ; : 1 ‘This can be investigated © Explain why [eax > galt 1)(2h=1). using a graphical By writing gts (h— 1)(2h — 1) ina different form, deduce its value as pe +h — 0. Comment on your answer. 10 Trigonometry Il What you need to know @ = The compound and double angle formulas. How to convert angle measurements between degrees and radians. @ The trigonometric ratios for the ‘special angles’. Review ‘Write down the compound angle expansions for the following: a sin(A+B) d_ cos(x +60") b cos(A—B) e sin(x—30°) cc tan2a f tan(x- 45°) 2 | 2 Convert the following angles into radians, leaving the answers in ‘terms of x: i 45" iv 220° ii 120° v 400" iii 315° vi —40° b Convert the following angles into degrees: eek i iv ii ii v i vi -% agen ob 3 | Without using a calculator, write down the exact value of the following: a sin 60° d sin? b tang e tann © cos90° F cos120° ‘Complete Advanced Level Mathematics: Pure Mathematics 10.1 The Factor Formulas In Chapter 3 we established the concept of an identity as an equation that is true for all values of the variable. One set of trigonometric identities is, known as the factor formulas. They convert expressions like sin A+ sin B into a product; similar to factorising the expression, Another common name for these identities is sum and product formulas. ‘These four identities have a straightforward derivation from the ‘compound angle formulas encountered in Chapter 3. To derive identity [1], recall the formulas: sin(A + B) = sinAcos B+ cosA sin B sin(A — B) = sinAcos B— cosAsinB Adding these two equations, sin(A + B) + sin(A— B) = 2sinAcosB Now let C=A+ Band D=A—B. ‘The LHS becomes sin C + sin D, and we have SoA=1(C+D) and B=1(C-D) ‘The RHS of the equation becomes 2 sin{} (C + D)] cos(} (C — D)] So sinC + sinD = asin?) cos?) z 2 Subtract the result sin(A— B) from Uso the same technique to derive identity [2]. sin(A+ B). 404 Urheberrechilich geschtitztes Material 10 Trigonometry i To derive identities [3] and [4] we need to use the compound angle formulas for cosine. cos(A + B) = cosAcos B— sinAsinB cos(A — B) = cosAcos B+ sinAsinB Identity [3] can be derived by adding these results. cas(A +B) + 0s(A — B) = 2cosAcos 5 Now let C= (A +8) and D = (A ~ B) as before. We find that: exec anv ~aer( 22) ero(222) Nole carefully the symmetries and patterns in the four factor formulas. They can often be more easily remembered if you say the wards as well as write the symbols, for example: ‘sine plus sine equals twice the sine of half the sum, cos of half the difference’. ‘The factor formulas provide a powerful mathematical tool for dealing with trigonometric functions. They can be used to solve equations, simplify expressions and prove more identities. Example 1 Find sin 105° — sin 15°, without using a calculator. Solution sin 105° — sin 15° = 2 cos}(105° + 15")sin} (105° — 15°) = 2.cos60° sin45° s2uixGa8 Example 2 Prove the following identities: a cos 20 +cos 30 + cos 40 = cos 30(1 +2c0s 0) sin3@+sin@ cos 30 + cos Solution a Notice that the LHS has three terms, one of which is cos 3¢. This also appears on the RHS so it would be sensible to apply one of the factor formulas on the other two terms. rhebertect schitates MO5ia ‘Complete Advanced Level Mathematics: Pure Mathematics Reorder so that the factor formula for th sum of two cosinas i =2cos 30c0s 0 + cos 30 straightforward to = 60530(2.c0s0-+1) = RHS cos 20+ cos 30 + cos 40 = cos30(1 + 2cos 0) LHS = cos 20 + cos 30+ cos 40 = cos 40 + cos20+cos 30 in 30 +sin 0 b IHS = cos 30+ 6088 _ 2sin20cos 0 ~ cas 30+ cos _ 2sin20cosé ~ 2e0s 20 cos 0 _ sin20 ~ cos20 = tan 20 = RHS sin30+ sind _ tan20 Example 3 Use the factor formulas to express the following as a sum or difference of two trigonometrical functions: a 2sin2AcosA b -2sin4dsin2A Sofution a Notice that 2sin 2A cos is a product of two trigonometric terms. Since it contains one sine and one cosine term, it must be the result of summing two sine terms. Compare it with the factor formulas. Ifsinx + siny = 2sin2A cosA, then x4+y=4a and x-y=2A ‘The solutions of these simultaneous equations are x = 3A and. yaa. So 2sin 2AcosA = sin 3A + sinA b —2sin 4 sim 2A contains two sine terms and a negative sign. The factor formulas suggest that this is a difference of two cosines. chilich geschiitztes Material 10 Trigonometry It If -2sin4Asin 2A = cosx — cosy, then x+y =8A and x-y=4A ‘These simultaneous equations have solutions x = 6A and y = 2A. So —2sin 4Asin 2A = cos 6A — cos 2A. Example 4 Without using a calculator, evaluate 2 cos 75° cos 15°. Soiution Notice the two cosine terms. The factor formulas suggest thal this is created from the sum of two cosines, If2c0s75° cos 15° =cosx-+cosy,then Identity 3, x+y = 150° and x-y=30° ‘These simultaneous equations have solutions x = 90° and y = 60°. So 20s 75° cos 15° = cos 90" + cos60° = 0+ When G < D the factor formulas will produce negative angles. Sometimes the ratios could simply be reversed as in Example 5; sinx +sin 5x = sin 5x + sin x. In the cases where this is not easily achieved, however, use the following useful results, which can be seen from the graphs for sine and cosine. Note the rotational symmetry ‘Note the reflection in the y-axis or ‘These results for negative angles can ‘simplify’ the factor formulas including negative angles. Complete Advanced Level Mathematics: Pure Mathematics Example 5 Solve: a sinx +sin5x b sin3x — sins: Solution a Notice the three distinct angles, x, 3x and 5x. The factor formula will help reduce this to a problem with fewer angles. The equation is easier to manipulate if rewritten as in 8x forO 2sin 3x cos 2x — sin3x = 0 > sin 3x(2e0s 2x ~1) =0 So sin 3x = 0 or 2cos 2x —1 sin 3x = 0 oF cos 2x =} sin 3x =O = 3x =0,2,2m,3R => x=0,5,8,x,... cos2x =4 = x= 5,82 ie, ‘The values of x within the given range are: 0, §, 5, 55, x. Instead of rearranging sin 3x ~ sin 5x = sinx to make all terms positive (giving the equation sin 8x = sin 5x + sinx, which we solved in a), we can apply the factor formulas to the original equation, and create a negative angle. sin 3x ~ sin 5x =sinx = 2cos 4x sin(—x) = sin x Using the result sin(=x) sinx, gives —2e0s 4x sinx = sinx => 2cos 4x sinx + sinx x(2c084x +1) rheberrechtl schitztes Material Complete Advanced Level Mathematics: Pure Mathematics 10.1 The Factor Formulas Exercise Technique Solve the following equations for 0° 9416 =F"(sin’ x + cos* 2) > 25 =F Recall that R > 0, so = R=V25=5 ignore —5 as a soluti 412 Urheberrechtlich geschitztes Material ‘Complete Advanced Level Mathematics: Pure Mathematics 414 ‘We would usually differentiate f(x) to find its maximum and minimum Remember to use values, but in this case they can be found without differentiating. radians when Consider the behaviour of the cosine function. It oscillates between a differentiating. maximum value of +1 and a minimum value of —1. So cos(x + 16.3") has a maximum value of +1 and a minimum value of —1, and since we have a stretch factor of 25, f(x) has a maximum value of 25 and a minimum value of =25. 4. aise ar 254144 = Fi (sin? a + cos? a) > 169 = RY So R= V109 = 13 So 12¢osx +5 sinx = 13cos(x ~ 0.395). ‘We know that 12 cosx +5 sinx = 10, Substituting the new expression we have just found, 13cos(x — 0.395) = 10 cos(x = 0.395) = 12 One possibility for x is given by x — 0.395 = cos” (18) = 0.603 So x = 0.693 + 0.395 = 1.088 x=109(3 sf) Ine ewe tes & Y= 12cosx+5sinx 3 Tiare Check that there is only one solution: cos(x — 0.895) = (38) has other solutions, including ~0.693 and 6,976, but they are outside the given range. Maximising and minimising rational functions ‘Sometimes the denominator of a rational function contains terms involving sin x and cos x. The technique of expressing these two terms as one trigonometric ratio can be useful in identifying the maximum and minimum values of the original function. Example 4 " y 1 ‘Find the maximum and minimum values ol Scan ecine as 1S Urheberrechtlich geschiitztes WPherial 10 Tigonometry I Periodic motion Periodic motion is any motion that repeats itself in equal intervals of time. Equations used to model this type of motion contain trigonometric terms, because the sine and cosine functions are periodic, Examples of this type of motion include: @ amass oscillating on the end of a spring: @ abuoy moving up and down on the waves on the surface of the water in a harbour; @ | the tip of a sewing machine needle moving up and down. Asspecial case of this type of motion is known as ‘simple harmonic motion’, ‘A.mass is suspended from the end of a spring as shawn in the diagram. ‘The mass is oscillating. The distance d cm between the fixture point and the mass is given by d= 17+ 12sin2t—5cos2t, where f seconds is the time after release. Find: a the maximum and minimum distances from the fixture point reached by the mass b the time at which the mass is first at its lowest point. Solution Notice that the expression for d contains two trigonometric terms. 12sin 2t — 5cos 2¢ can be rewritten in the form A sin(2t— a), where O Scos(—53.1°) =1 So cos(0—53.1°) =} ‘The principal solution is given by @ ~ 53.1° So0~ 53.1" = 360°n + 78.5°,n € Z. ‘The general solution for # is in two parts. cos'(2) = 78.5%. 6 = 360'n + 78.5" + 53.1" > 0 =360°n + 131.6" and @ = 360°n - 78.5" +53.1° = 0 =360'n-25.4°, wheren eZ Urheberrechilich geschitt 40 Trigonometry It 10.3 General Solutions of Trigonometric Equations Exercise Technique Find the general solutions, in degroes, of the following equations: a cos0=0.9 d sind b cos0 = 0.25 Gc sind=0.2 Find the general solutions of tho following equations: a cos0=] d b cosa = —4 e ce sing= f Find the general solutions of the following equations: a sinz0=1 b cos0=cos30 ¢ sind =sin30 d- cosad=-3 Find the general solution, in degrees, of the equation cos0+3sin 0 =2. Find the general solution of the equation cos 0 + cos 30+ cos50 = 0. 6 | Find the general solution of the equation sin(x + 30°) = cos(x +45°). Oand 0 < x <}, Give the value of « in radians to three decimal places. Determine the greatest and least values of the following: 1 in 0 — oy b en Pane tees (sind — 4008041 State a value of ( in radians for which the least value of the expression in boccurs. (AEB) GJ a Express the function 3cos x° +4sin x” in the form Rsin(x +2)", stating the values of Rand 2. b Write down the maximum value of 3.cosx° +4 sinx’. c Solve the equation 3 cosx° + 4sinx° = 0 for 0° < x < 360°. (NEAB) 7 | Express /3sin 0 — cos in the form Rsin(@ ~ 2), where R > 0 and 0° Oand 0° 0 and 0 < a <§, state the value of Rand find the value of 2 in radians to three decimal places. c Determine the greatest and least values of oT and state a value of x at which the greatest value occurs. (AEB) 427, Complete Advanced Level Mathematics: Pure Mathematics 5 | Express 2.cos @ + 2sin 0 in the form Fcos(0 — 2), where R > 0 and 0 <2 <}n, giving the values of R and 2 in exact form. Hence, or otherwise, show that one of the acute angles 0 satisfying the equation 2cos 0+ 2sin0 = V6 is $4, and find the ather acute angle. (UCLES) [6 | {in this question, give your answers correct to the nearest 0.1°, where appropriate] a Find the general solution, in degrees, of the equation 5 cos 20 = 3. b Solve each of the following equations, far 0 < 0 < 180°: i Scos#+2sin*@=4 ii Ssin0+30os0=5 (UCLES) (7) By squaring both sides of the identity sin’ @ + cos? @ = 1, prove that, 4(sin‘ 6 + cos* @) = 3 + cos 48. Find the general solution in radians of the equation 4(sin* @ + cos* @) = 2 —cos 20. (AEB) Urheberrechtlich geschdtztes Material 10 Trigonometry i Applications and Activities The mathematical representation of a wave The general equation for a wave represents every point x on the wave at every time f, y= Asin(kx - ot), where A is the amplitude of the wave, k is the wave number and o is the angular frequency (velocity). Then k is the number of waves contained in the interval of 2x. Now consider sound as an example of a wave. If two waves ’, and y; given by Vi =Aysin(kyx — o,f) and yy =A, sin(kzx — ot) act at the same time, the resultant sound (from the principle of superposition) is given by y; + 2. Investigate the behaviour of yy + y2. and particularly the analysis of the resultant wave in the real world. How is this interpreted? Simple harmonic motion (SHM) SHM is motion in which the acceleration of a body is directly proportional to its displacement from the equilibrium position but in the opposite direction. Investigate SHM when the displacement of the object performing the motion is written in the form x = Acosut +Bsin wt, where w is the angular velocity, ¢ is time and A and B are constants to be determined. What is an advantage of writing displacement in this form? Complete Advanced Level Mathematics: Pure Mathematics: Summary @ = The factor formulas, used to convert sums and differences of sines and cosines into products, are: aa" aa." @ The function asin x + dcosx ean be expressed as: Rsinx+2) —Rsin(x—2) —-Reos(x+2) —-Reos(x —2) depending on the sign values of @ and b, Rand x can be found by first expanding using the appropriate compound angle formula, and then equating coefficients to produce two simultaneous equations. Squaring and adding them gives R = Va? +B? and dividing them gives an equation for tana. This technique is useful for 1. solving equations of the form asinx +beos x = ¢; 2. finding maximum and minimum values of rational functions with trigonometric denominators; 3. analysing periodic motion. @ | Trigonometric equations of the form: sin® = kand cos # = k, where -1 ¢k <1 and tan® =k, where KER can be solved to give both a principal solution and a general solution. The principal solution can be found using a calculator, The general solution formula can then be used to generate other valid solutions as required. Remember that the general solution can be written in both degree and radian form. 1. Ifcos 0 =k, then 0 = 360°n + 2 or 0 = 2xn +2. 2, Msind = 4, then @= 180°n + (~1)"«or 0 = xn + (—1)"2. i | a. Iftand =k then @ = 180°n +20ré =an+a. jes Material 11 Differentiation I What you need to know @ How to differentiate rational powers of x, and sine and cosine functions. @ How to use the chain rule to differentiate composite functions. @ How to locate stationary points on a curve and determine their nature by Joking at the sign of $f on either side of the stationary point. @ How to find the equation of the tangent and the normal to a curve at a given point. @ How to solve equations involving e* and Inx. @ How to write down the Cartesian equation of a circle. @ How to use the binomial theorem to express (11+ ax)" as a series of ascending powers of x, where nis rational. Review Differentiate each of the following with respect to x: a oy=x?-ox+11 e b y=10ve f c ysx-xt 8 do ysaxtt h 2 | Use tho chain rule to find # for each of the following: a y=(x+11)* d yp=v2-8 b y=(4x-1)* e cys vers fy =i mei ae 3 | Find the coordinates of the stationary points on the graph of y =4}x*— x’ — 3x? + 8x +3 and determine their nature. | Find the equations of the tangent and the normal to the curve y =4 at the point where x = 2. 5 | Find the exact solutions of the following exponential and logarithmic equations: a a4 b In(sx) d In(x-1)=6 431, ‘Complete Advanced Level Mathematics: Pure Mathematics {6 | Write down the Cartesian equation for each of the following circlas: a radius 4, centre (0, 0) b radius 6, centre (0, 3) ¢ radius 7, centre (4, ~2) d_ radius V3, centre (—1, 6) 7 | Uso the binomial theorem to express each of the following as a series of i ascending powers of x, up to and including the x? term: i‘ a (1¢x)? b (1-2)! ° es do Vitae Urheberrechilich geschiitztes Material 11 Differentiation Il 11.1 Differentiating Products and Quotients In Chapter §, the techniques for differentiating polynomials, functions involving rational powers of x, and sine and cosine functions were developed. Many other mathematical functions are formed by multiplying or dividing two or more of these types of functions. Some of these products and quotients can be differentiated by first expressing the function as a polynomial, and then differentiating term by term. However, there are many products and quotients, such as ¥ = (2x4 3)(x~ 1)! andy = 924", where it is either difficult or impossible to express the overall function as a polynomial. Functions of this type can be differentiated using two standard results, or algorithms, which are known as the product and quotient rules, The product rule Suppose y = u(x)r(x) is the product of two separate functions of x; and v. Any small change, 6x, in the value of x will give rise to corresponding small changes, 6u and Sv, in the values of functions u and v respectively. These in turn result in a small change, dy, in the value of y, such that oy =(y + dy) -y = u(x + dx) (ax + 3x) — u(x) (x) Itis possible to rewrite this expression for dy in the form, Sy = ule + Sxpe(x + dx) — ule + dx)er(x) + v(x)u(x + dx) —u(x)v(x) It follows then that dy = ulx + dx)[v(x + 5x) — v(x)] + v(x)[ule + dx) = UG] Difforentiating from first principles, a (+x) (# oe = vs) + Kim [ron (te 5) ze) ee ba is = lim [ute + oxy] x fim [PE Bw eee + (x) > jm [HEE Y= He) sere eee) = ula) x 2 ven) x ‘Complete Advanced Level Mathematics: Pure Mathematics The product rule for differentiating functions of the form y = u(x)v(x) can be stated as Using the abbreviations u’ and v’ for 4! and 4 respectively, the product rule is more commonly stated as a Example 1 Using the product rule, differentiate y = (2x + 3)(x = 1)* with respect tox. Solution = uw where u = (2x+3) and v= (x ~ 1), Thenu! = 2 and (x — 1)", Using the product rule, dy. uv’ +00" = (2x +8) x a(x — 1)? + (x — a) x2 = 4(2x +3) (x — 1)3 + 2(x -1)4 = a(x —1)*[2(2x +3) +(x—-1)) = 2x —1)*(5x +5) = 10(x+1)(x—1)° ‘An advantage of the product rule is that it is usually possible to get a factorised expression for {¢. This is particularly useful when trying to locate and determine the nature of any stationary points. Example 2 Find the gradient of the curve y = x* cos x when x = x. Solution Let y = uv where u =x" and v = cosx, Then ul = 2x and v’ = —sinx, Using the product rule, ow! =x? x (-sinx) + cosy x 2x = =x? sinx + 2xcosx = x(2co8x — xsinx) 434. 11 Differentiation II When x = x, the gradient of the curve y = x? cosx is WY s(2cosn— nsin) pees =-2n The quotient rule Suppose y = #1 is the quotient of two separate functions of x; u and v. ‘This can be rewritten as the product of u and 4, such that tae = v v Using the product rule to differentiate y =u x 1gives ‘V idu nG) ea ‘So we have the quotient rule for differentiating functions of the form y= oe This is more commonly written as EEE nt Example 3 Use the quotient rule to differentiate the following functions with respect tox: sin Solution a Lety =#,whereu =3x+1 andy =x—2, Then u'=3andv’ Urhe n geschitt 114 Differentiation Il 11.1 Differentiating Products and Quotients Exercise Technique [1] use ine product and quotient rules to differentiate each of the following with respect tox: rome b (x —3)(x+2)* at (-a, -6) ce y= Hh at (1,2) Contextual 1 | Find the equations of the tangent and the normal to the following curves at the points indicated: a y= #44 at (2,-4) by © y=xt osx at (7, -2*) dy = (x —2)(x+ 4)? at (-3, -5) =r at (-1,) Urheberrectilich geschittztes ABZ ia 11 Differentiation Il 1.16123 1.10467, re 1.09862 | 1,38630 Notice that as dx — 0, the factor “=! converges to a limiting value, which is different for each value of a. For example, fey = 0.69315 x 2" Jay 1.09861 x 3° ‘This suggests that there is a value of a between 2 and 3 for which Jim (5-4) = 1. For this particular value ofa, it follows that (a) = a". Usea calculator to complete the above table of values of “= fora = 2.5, 2.7 and 2.8. By further trial and improvement, find the value of a, correct to three decimal places, for which fim (=4) = 1. ‘This value of a is an irrational number. Its value is e = 2.718 2818, correct to seven decimal places, It fallows that te 1 n( ox a1 ‘This means that the derivative of the exponential function y = e* is GR 6 teem nis importantes, ‘The exponential function is the only function that remains unaltered when differentiated, An alternative way of establishing this result is to use the series expansion of e* introduced in Chapter 7: Urheberrechtlich geschitt (Complete Advanced Level Mathematics: Pure Mathematics. Differentiating this infinite power series term by term, d 2x ax ae 05 ates atart 2 oe =145+Rtgt =e It can also be shown that the derivative of any exponential function of the form y =a" is ‘This means that Jim (#=") = Ina. ‘The chain rule can be used to differentiate exponential functions of the form y = e/"), where f(x) is some function of x. Let y where u = f(x). Then $ =e! and #2 = f"(x) Using the chain rule, 4 Learn this important result, ay _ dy du du “dx =e! x f(x) =/'(xjo"! <4 Derivative of the power x original exponential function Example 1 Differentiate each of the following with respect to x: a Lety =e" where u = 3x+1. Then # =e” and ! = 3. Using the chain rule, Urheberrechtlich gesch Material Complete Advanced Level Mathematics: Pure Mathematics 442 Example 3 Asa result of a slump in the housing market, a property initially valued at £60 000 on 1 January depreciates. Its value ¢ weeks later can be modelled. by V = 600000" where its value is £V and k is a constant to be determined, Exactly one year later, the property remains unsold and is valued at only £50000, Solution b Using the quotient rule, dy vu’ ar je" — (ax +2)" — (e 3xje" ey? 13x ae Let.x = uv, where u =e” and v = cos 3¢. Then u’ = 4! = 2e" and w = = -3sin3e. Using the produet rule, dx at = ~3e" sin Bt + 20" cos 3t =e" (2cos3t—3sin 3t) uv! + vu' Find an expression for k. Find the market value of the property after 26 weeks to the nearest pound. Find an expression for 4. Then find the rate at which the property’s value is depreciatingat the start and end of the year to the nearest pound. Afler one year, when f = 52, V = 50000, Substituting into the expression for V gives 50000 = 60 0000" = ead ‘Take logarithms of bi => 52k =In(j) sides. = azin(@) k= 0.0035; When t = 26 a mogative value is expected for = zak V =60000e" depreciation. =60000e!!"4) = 654772 11 Differentiation It © Given that V = 60 000e", where k = 41n(3), differentiate V with Tespect tot. av aa H ay = 80 000ke = 82 In Genin’ At the start of the year, t = 0, The rate at which the property's value is depreciating is found by putting ¢ = 0 into this expression. ‘Check that Sf -epage = 210.37 ‘To the nearest pound, the rate of depreciation is £210 per week. ‘Carry out a similar calculation by substituting ¢ = 52. Verify that after cone year, the rate of depreciation is £175 per week Logarithmic functions ‘To differentiate the natural logarithmic function y= In.x from first principles, consider the small change, dy, in the value of this function that results from a small change, x, in the value of x. Since éy =(y +é6y)-y = In(x + 5x) —Inx, where y + dy = In(x + 4x) is the value of the function corresponding to x + 4x, il follows that = jm [Bos tos) Unfortunately there is no easy way of expanding the In(x +x) term in this expression, An alternative approach must be taken in order to find 4. Recall that the natural logarithmic function y = Inx is the inverse of the exponential function y =e. Iy =Inx, then x =e. Differentiating both sides with respect toy. dx. yn Urheberrechtlich geschiitzte 14 Differentiation II Example 4 Differentiate the following with respect tox: = In(x? + 3x — 2) d y= In(cos x) ey = logy (x? +1) ce yexlinx Soiution a yaln(? +3x—2)> =, by =In(cosx) > —=88 = —tanx © Lety— uv, whore a =x andy =Inx. Then u! =1and v/ =3. Using the product rate, 9 = uv + vu! =*(3) +Inx % =1¢+Inx d= Lety =! where u = Inx and v = e%. Then u’ =} and v’ =e". dy _vu' — uv! Using the quotient rulo, g-"y e Let y = logy u, where u = (x? + 1). Then $f = 5phyg and ff = 2x. dy _ dy du ‘Using the chain rule, G7 du ax ee ~ulnt0 _ © (+ 1)In 10 ax Urheberrechilich geschatztefBeria ‘Complete Advanced Level Mathematics: Pure Mathematics: 11.2 Differentiation of Exponentials and Logarithms Exercise Technique [1] pitterentiate each of the following with respect to x: a f y=xo* b Bg y=e*?sinx c hy d i y=e'sin x e ioy=y [2] itterentiate each of the following with respect tox: y= In(ax) f y=In(sinx) In(x*) 2Inx In(x*) y= In(4x +1) In(1 +x) = In(1 — x) y= In(x* +1) iy =logio(3x -4) a b © d e [3] For each of the following curves: i find expressions for #° and # ii locate and determine the nature of any stationary points. 2e + 1)e* (x +2) + hy © y=e'sinx(-2 0, Urhe rosia ich geschiit 11 Differentiation 1! d- Lety = tan u, where u =3. Then # = i dy dy dul 1 a Using the chain rule, GE = GE x Gt = lye 1 = ay a(i+a __a a ex Example 5 ‘Use the product and quotient rules to differentiate each of the following: a yexsin tx b yaaa Solution a Lety = uv, wherew =xand v = sin“! x. Then u’ = 1 and v’ = Ls. Using the product rule Baw’ +n x +sin™' x bo Lety =4,whereu =tan-*x and y= v= 2x. 2. Thom u! = phy and val —uv' ro atx? ot - G3) = axtan“ x - (exe Using the quotient rule, oe = T= 2xtan' x (1 +x Urheberrechilich geschitzteqg@iterial Complete Advanced Level Mathematics: Pure Mathematics 11.3 Further Trigonometric Differentiation Exercise Technique Differentiate each of the following with respect to x: a ystanzx e by =secdx f © y= cot(x’) 8 dy =cosec*sx h Use the product and quotient rules to differentiate each of the following with respect to x: a y=x*tanx by =5xsecx c y= 2xsin''x do y=x*tan'x Contextual [11] Fina the gradiont of each of the following curves at the point indicated and the equations of the tangent and the normal at that point. ay =tan({) at (x, V3) b oyssin (1—x)at(hg © y= xsocx al (x, -2) 454 Urheberrechilich geschitztes Material ‘11 Differentiation tl 11.4 Using the Second Derivative Stationary points Recall from Chapter 5 that the stationary points on a graph can be located by solving the equation 4! = 0. Their nature can be determined by looking al the gradient of the graph on either side of each stationary point. It is also possible to use the second derivative to help decide whether a particular stationary point is a maximum or a minimum point. ‘The gradient of a curved graph is itself a function of x. By drawing the graphs of y = f(x) and ils gradient function y = /"(x), itis possible to relate features that appear on the two graphs. For example, sketch the graphs of the function f(x) =x* = 3x* - 9x + 10 and its derivative f’(x) = 3x* — 6x — 9. Check the nature of the stationary points as you draw the graphs. Check your results using a graphical calculator. sf (-1,15) ae point of fla) =39-at-ow + 10 inflexion at(1,—1) nt = 0 minimum point at(3,-17), gradiont = 0 sradient of the gradient function alae) <0 at maximum point S\ -\O P(x) = ax? -ax-9 * gradient of the gradient function. 4 ea ala)>s al minimum point gradient of the gradient function 4 (a ii (a) =9 at point of inflexion theberrect Shep Complete Advanced Level Mathematics: Pure Mathematics Notice that at the maximum point on the curve y =x" ~ 3x* ~ 9x + 10, the function changes from being an increasing function of xto a decreasing function of x. This ‘negative’ change in the gradient, from positive through zero to negative, as the curve passes through (~1,15), means that the ‘gradient of the gradient function’, or second derivative, is negative at the maximum point. So at the maximum point on the curve, {L(12) < 0. SS nt rt At the minimum point on the curve y =x" — 3x? — gy + 10, the function changes from being a decreasing function of x to an increasing function of x. The corresponding change in the gradient is from negative through zero to positive as the curve passes through (3, ~17). This indicates that the ‘gradient of the gradient function’, or second derivative, is positive at the minimum point. So at the minimum point on the curve, #(%8) > 0. PS eet simp eit There is also a point of inflexion at (1,—1) on the curve y =x" —3x" - 9x +10. This is where the tangent to the curve crosses from one side to another. This is not a stationary point. Instead itis the point at which the gradient of the curve is at its most negative. The ‘gradient of the gradient function’, or second derivative, is zero at this point of inflexion. In fact £4 = 0 at all points of inflexion, whether they are stationary or not. Itis possible, however, for the second derivative to equal zero at points that are not points of inflexion. Consider the graph of y ~ x*. This has a stationary point at x = 0. Its second derivative 14 = 12x7 equals zero when x = 0, but this stationary point is clearly a minimum. YE pet Check this with a graphical calculate Check also the grap ry He mond $3 = atx = 0 y= —x". This isa maximum at x = 0 where {5 11 Differentiation tt Summary @ = oand £3 < oat maximum points @ X= oand > oat minimum points @ iff <0 and {4 = 0, the sign of the gradient, #, on either side of the stationary point must then be found to determine its nature - de not. assume that it is a stationary point of inflexion. Example 1 Find the coordinates of the stationary points on the curve y= (x +3)(x—2)*, Use the second derivative to determine their nature, Sketch the curve. Solution Lot y = uv where u = (x +3) and v = (x ~2)'.'Then uf = Land v= (x2), Using the product rule, ve u/ +00! = (x43) x 4(c = 2)" 4 (x-2)' x1 =4(x+3)(x— 2)" + (x—2)' = (x —2)°[4(x +3) +r 2)) = (x —2)"(5x +10) = 5(x+2)(x— 2) Al stationary points on the curve, 42 = 0. B(x + 2)(x—2)7=0 > x= -2Zorx=2 Verify that when x = —2, y = 256 and that when x = 2, y = 0. The stationary points are located at (~2, 256) and (2,0). Their nature can he determined by evaluating the second dorivative at x = —2 and x = 2 respectively. Use the product rule again to find ©, Let &! = fg, where f = 5(x +2) and g = (x —2)°. Then f’ =5 and g =Hx- 2), It follows that ay Fand g have been used ae Btw here instead of wand w * ; to avoid confusion with = 15(x + 2)(x — 2)? + 5(x — 2) (heannleworkh = S(x — 2)"[3(x + 2) +(x —2)] = 20(x —2)"(x +1) Urheberrechtlich gesch ‘Complete Advanced Level Mathematics: Pure Mathematics When x = —2, £4 = 20(—-4)*(-1) = -320 <0. So (—2, 256) is a maximum point. When x = 2,8 = 20(0)*(3) = 0. Looking at the gradient of the curve on either side of x = 2: @ = whenx = 1, $= 5(3)(-1)' = -15 @ = whenx =3, $= 5(5)(1)' =25 ‘The gradient changes from negative to positive as the curve passes through x = 2, so (2, 0) is a minimum point. This information can now be used to sketch the curve. (2,256) M4 <4. Check this result using a graphical caleulator, ym bet 3) (x= 2) General points of inflexion Stationary points of inflexion have two forms. The gradient of the curve can be positive on both sides of a stationary point of inflexion or negative on both sides, as shown in the diagram. Check that a tangent drawn to the curve at a stationary point of inflexion crosses from one side to the other. However stationary points of inflexion are special cases. The gradient of a curve does not have to be zero at a point of inflexion. In fact, there must be al least one non-stationary point of inflexion on any smooth continuous curve between two stationary points. This situation can best be described graphically. Complete Advanced Level Mathematics: Pure Mathematics vasa B (——#oa “ol © y=s4 Sy moan $Y <0 ‘The gradient reaches a maximum value at point B. This means $4 = 0, Since the gradient ofthe gradient function is changing from positive to negative as it passes through B, a I ane = dy *. y This result is true for all circles centred at the origin, because the radius squared term in the equation of the circle is zero when differentiated. This can be confirmed by considering the radial line from the centre Oto some point (x,y) on the circumference of the circle. Notice that the gradient of this line y is¥, Because it also intercepts the circle al right angles, il is the normal to the circle at this point. This means that Ws) the gradient of the tangent to the circle at this point is —*. Example 1 Acircle C is centred at (6, -2) and has a radius of 5 units. a Write down the Carte b Find an exprossion for $. ¢ Find the gradient of the circle at the two points where x = 9, Solution a Recall that, for a circle of radius r, centred at (a, b), (x-ay +(y—b) =P ‘The Cartesian equation of this cirele is therefore (x6)? + (y +2)? =25 b Differentiating both sides of the equation with respect to x, d d d ae OF + +27 = F285) Verify that the equivalent form is x+y? — 12K + dy +18 Alternatively, differenti at4y?- 12x 4y +18 term by term, using t chain rule for the tert iny. 11 Differentiation It Using the chain rule, é (x6)? =2(x-6) and fest a 2 dy poeta 4 = ay +2) So x5) +2442) 0 Then ay +2) = -26¢- 6) © Substitute x =9 into (x — 6)? + (y + 2)? = 25. This gives 37+ (y +2)? = 25 Qty? +444 = 25 yr +4ay—12=0 (y+6)(y-2)=0 + y=-Bory =2. AL (9, -6), At (8,2), Notice that in this example, 4 = Sis undefined when y ==2. This corresponds to the two paints on the circle where the tangents: are parallel to the y-axis, Unlike the equation of a circle, many implicit functions have equations that include product terms. These are expressions such as xy, xy? and x°y. These terms can still be differentiated implicitly but require the use of the product rule, Urheberrechtich geschittzteq@eyerlal Complete Advanced Lave! Mathematics: Pure Mathematics Example 2 For each of the following, find an expression for in terms of x and y. a ox? 43xy-4x=9 bo oyt+axt-y?=0 Solution a Differentiate both sides of the equation with respect to x. d d d ad ae) +O) gy) = Use the product rule to differentiate the second term. Let u = 3x and v=y. Then u! = 3andv’ = Now Slow) = wr! p= ai sy It follows that differentiation of both sides of the equation gives at at +3y-4=0 Rearranging to find §, ax <4 -28—sy dy _4~-2¥~-3y dx ax b Differentiate both sides of the equation with respect to x. d Sty) tar 8 yt )=0 ‘Using the product rule, duyy= ay + 2xy? and using the chain rule Suy-at dy dy So rag Oe tae = (axty- - = —(2xy? + 8x) = w0e- 1 =—axGy4 +4) > yt) Ea nty? +4) dy _ xy? +4) dx y(1—x?) Urheberrechilich geschiitztes Material Complete Advanced Lavel Mathematics: Pure Mathematics ‘Now factorise and simplify the equation. dy diy _ 2-854 (4- 4x) 25 =0 2y-x dy 2 0(#53) + (4-40 =0 ‘Verify that this equation can be rearranged to give dy _16y—4x—4 de 2(1—x) a(1—x) So (~2, -1) is a maximum point on the curve, Whenx =4andy =2,¢8=2=3>0 So (4, 2) isa minimum point on the curve. Notice that the equation of the curve in this example can be rearranged into the form y = f(x). Show that y = #4, Its first and second derivatives can then be found using the quotient rule. Use a graphical calculator to draw the graph of y = ey Confirm that the curve has a maximum point at (~2, —1) and a minimum point at (4, 2). y. _asite Yana 470 Utheberrechitich eschat es Material 14 Differentiation Il 11.5 Implicit Differentiation Exercise Technique [71] For each of the following circles: write down its Cartesian equation find an expression for in terms of x and y ili calculate the gradient of the circle al the point P, whose coordinates are indicated: — a radius 5, centre (1, 2); P (5. 5) b radius 13, centre (3, —4); P(—2, 8) c radius V9, centre (5, 0); P(—1, v3) [2] Find an expression for # in terms of x and y for euch ofthe following: a 3x72? tay =0 do d4y—Sx42y7+6=0 b x? +4y— aay =0 e ww+7y-3x'y-4=0 c 6x*+2y*?-y+4a=0 f sx-3y? +207 +1=0 Find the gradient of each of the following curves at the point indicated: a y-2xt- ‘tay + 24 = Oat (2,2) b ay—axt-y? +1 at (2,5) By? =3x=x7y=47 = Oat(=1,4) For each of the following curves: i find an expression for # in terms of x and y fi find the coordinates of the stationary points(s) on the curve iii find an expression for in terms of x,y and iv determine the nature of the stationary point(s): a 3x? +y*—3xy-9=0 b 5x4 +24 -2y-12=0 G oxtla+x'y+2=0 Find the equations of the tangent and the normal to each of the following ‘curves at the points indicated: a 2x43y—xy? +4 =0al(-1, -2) box 42y? + axy? — 12 = Oat (2,1) cy? 410K + xy — 15 = Dat (6, ~3) 471 14 Differentiation Il Example 1b is particularly important. The parametric ‘equations of an ellipse centred at the origin are x = acas 0, y = bsin@, Eliminating the parameter @ gives the Cartesian ‘equation for the ellipse wie ‘The Cartesian equation of a curve can usually be obtained from parametric ‘equations. Eliminate the parameter, t, from the two parametric equations, x= f(t) and y = g(t). How is the gradient of such a parametrically defined curve found? First differentiate both x and y with respect to the parameter , to find 4f and { respectively. Then, using the chain rule 9 A Mth cant tena dx FEB) Py-6P = ax- 20 = Cy-2x=40 or Py-2x-4"=0 Urheberrechilich geschiit Complete Advanced Level Mathematics: Pure Mathematics 476 The equation of the normal is y-6t=-}@-0) Multiply both sides = ayaa Pet ff = gy¢PxsO+12t or ay+ex—t—12r=0 At the point (8, 12), 1 = 2. The equation of the tangent at this point, found by substituting { = 2 into Cy — 2x = 4f°, is 4y—2x=32 or 2y-x-16=0 At the point (~1, ~6), t = —1. Substituting ¢ = —1 into the general equation for a normal to the curve 2y + Px =f? + 12t, gives W+xs-13 or B+x+13=0 Stationary points and second derivatives ‘The second derivative 4 of a parametrically defined curve is found by differentiating the first derivative & with respect to. (that is, ¢¢ ¢2)), Bul, for most parameter curves, {¢ will be a function of the parameter, t, and not x. This means the chain rule must be used to change the variable by which ¢ is being differentiated, so that Having found an expression for“, it can be used to determine the nature of any stationary points on the parameter curve. Example 5 Find an expression for # and for each of the following parametrically defined curves: a x=214+3 b x=4cosd yet y=sin? 0 Solution a Ya2ang=7 » &-(@)/(a) = 2e 14 Differentiation II dfdy) dt ai lax) “av =£(23) ~ at ae) “ax Beet Bee ot ~ 28 b $)=~4sind and 4; = 2sinOcos0 ae (30) / (a) _ 2sinOcos@ =4sin0 ay w) do Example 6 Find the coordinates of the stationary points on the curve defined in terms of a parameter t by x = (? + 1 and y = f° — 121. Use the second derivative to determine the nature of these points. Solution i = 2tand = 3? — 12 = (Hye) At stationary points on the curve $f = 0. af-12_4 Solve this equation by a ‘equating the numerator > af =12 to zero, So Urheberrechttich gesch 14 Differentiation It Solution 4 = 2tand = 4-20 Gi ae ae dx dt * 8S aE At stationary points on the curve, & = 0. 4-2 atte andied t=2 So the stationary point occurs at (5, 5). To determine its nature we need to evaluate the second derivative. ay a a) at de de di\ de When t = 2, ££ < 0, so (5, 5) isa maximum point. To sketch the curve we can evaluate x- and y-coordinates for different values of the parameter, ‘Together with what we know about the stationary points, we can produce asketch of the curve; v, Urheberrechtlich gesch Complete Advanced Level Mathematics: Pure Mathematics 11.6 Parametric Differentiation Exercise Technique Eliminate the parameter to find the Cartesian equations of the following d= x=4cos@,y =3sin0 e@ x=1+2c086,y =3 +4c080 f x=acos0,y=bsind Find the coordinates of the points on each of the following curves where the gradient has the value indicated: Sot b Find the equations of the tangent and the normal to the curve with parametric equations x = e, y = 1 +e”, at the point where f = 1. ‘The curve C has paramotric equations x = f* and y = 2¢— 1. Finda general equation for the tangent at some point (f*, 2t — 1) on the curve. Hence, write down the equation of the tangent to the curve at (16, 3). Find the coordinates of the stationary points on each of the following parametrically defined curves. Find also an expression for the second derivative and use it to determine the nature of these stationary points. Hence, or otherwise, sketch the curve. a x=3t+1 bo ox=P41 y=0-6r ystste Complete Advanced Level Mathematics: Pure Mathematics So the linear approximation to the function gives a value for the definite integral that is correct to two decimal places. This approach is particularly useful when evaluating integrals that are difficult to evaluate exactly. The binomial theorem can only be used to find the series expansions of functions of the form (a +x)", where n is a rational power. Functions such as sin.x, cosx, o* and Inx can also be expressed as a series of ascending powers of x using the Maclaurin series. The Maclaurin serles Suppose f(x) is some function of x that can be written as a series of : ascending powers of x, such that he Maclauria Series expansion $0) = do + 04x + ax? Hagx" +... ta to Mae tena where a, is the coofficiont of the x” term. Assume that f(x) is a function that can be continuously differentiated to find f'(x), f"(x) and its higher derivatives. It follows that F(8) =) + 2K + Sagx? + 4ayx? +... f(x) = 2a; + Bax + 12a4x" + 20052 +... JF" (x) = Gay + 24a4x + 6Oa5x* + 120a5x" +... f(x) = 240, + 1200,x + 3600,x" + 8400,x" +... and so on. Substituting x = 0 into these expansions, F(9) =a = a = flo) FO) =a, = a =/00) $"(0) = 2a, = 4 LO) s°(0) = 60, = 4-f0 f.%(0) = 24a, = 4tx ay > ay oo More ganarally; «=O where f\")(0) is the value of the rth derivative of f(x) at x = 0. Substituting these values for the coeffictents Gp, 44, 2» 45, @4y +++ into the series for f(x) gives Ca rt 4 Learn this result. (Complete Advanced Level Mathematics: Pure Mathematics Example 5 Use the Maclaurin series expansion to find the cubic approximation of In(1 +x). Hence find the approximate value of In(1.1). Solution fle) =I +x) + f(0)= In = Fey=(txy" = f(=1 fe) =-(1+a)* = f(=-1 "(x)= 21 +x) = f"(0) = 2, and so on. ‘The Maclaurin series for In(1 +x) is therefore ae In(1 +x) =x— ai and the cubic approximation is In(1 x) x= 4x? 44x? Substituting x = 0.1 gives In(1.1) = 0.1 ~} (0.1) + 4(0.2)° = 0.1 — 0.005 + 0,000 33 = 0.0953 (4d.p.) By evaluating the higher derivatives of In(1 +x) when x = 0, it is possible to show that the Maclaurin series for In(1 +x) is ‘This expansion is only valid for —1 , Find the two values of the constant a for isich fx) has a stationary value when x = 2. (OCSEB) [7] A.curve has equation x" +" 4 2x 4 5y = 9. Find an expression for $¢ in torms of x and y. Hence show that the gradient of the curve is never positive. (OCSEB) 11 Differentiation It Applications and Activities Making a cone Cut a sector of angle @ from a circular piece of card of radius 20cm and attach the two straight edges together 20 cm to make a right-circular cone of height hem and > base radius rcm. a Find an expression for the perpendicular height, 4, of the cone in terms of its base radius, b Use this to find an expression for the volume, V, of the cone in terms ofr only. ¢ Use the product rule to differentiate V with respect to r. Find the exact value of r for which &¢ = 0, and the corresponding value of V. Show that this is the maximum possible volume for a cone made from this particular piece of card. d__ Find an expression, in terms of 0, for the longth of the circular edge of the piece of card from which the cone is made. Equating this to the circumference of the base of the cone, find the value of # that corresponds to the maximum possible volume of the cone. Repeat these calculations for a cone made from a circular piece of card of radius Rm. In what way is the angle @ that corresponds to the maximum possible volume of the cone dependent on the radius R? Summary © The second derivative 4 is used to determine the nature of stationary points: a P, at) and ty>0 = minimum point we ity de 7 and Gg <0 > maximum point @ When # = 0 and {5 = 0, the gradient must be considered on either side of the stationary point to determine whether it is a maximum point, a minimum point ora point of inflexion. @ Anon-stationary point of inflexion is a point where the gradient of the curve is at its maximum or minimum value locally, The tangent to the: curve at these points crosses from one side of the curve to the other. 491 Complete Advanced Level Mathematics: Pure Mathematics. @ = Types of inflexion can be determined by considering the first and third derivatives: @ The product rule for differentiation is: @ = The quotient rule for differentiation is: echilich geschiitztes Material 11 Differentiation I! @ A function inx and y that cannot be written in the form y = f(x) is an implicitly defined function. The method used to find the gradient of an implicit function is implicit differentiation, and relies on the chain rule. The product rule and/or the quotient rule may also be needed. @ The first and second derivatives, #¢ and #4, can be found for parametrically defined curves using the chain rule. @ = Maclaurin’s theorem can be used to find series expansions for standard functions. For values of x close to 0: where f")(0) is the rth derivative of f(x) evaluated at x = 0. Urheberrechtlich geschiitztaqgguterial ‘Complete Advanced Level Mathematics: Pure Mathematics 12 Integration Il What you need to know How to integrate polynomials and basic trigonometrical functions by reversing differentiation. How to use integration to calculate areas and volumes. How to differentiate standard functions and use the product rule, How to manipulate algebraic fractions. How to express rational functions as partial fractions. How to differentiate implicit functions, Review Integrate the following with respect to x: a xt +5x?-x+10 b (x-4)? ¢ sinx d- cosax e x f sinkw Sketch the curve y = x* +1. a Find the area under the curve between x = 0 and x = 1. b Find the volume generated when this area is rotated about the x-axis, through one complete turn. Differentiate the following functions with respect to x: a sin(4x +1) e Inx ho In(x? +7) b ef fF Inax i xe* c ef g In(2x+13) j xsin2x a et Express each of the following as a proper fraction: x+1 x a y-1 ‘om Express each of the following as partial fractions: x-7 3x b ax? — 7-18 (x +1)(x +3) (e+ 1G? +1) x 7x +10 Find ¢ for each of the following using implicit differentiation: a yP=x41 b ya=x*+2x-3 Complete Advanced Lavel Mathematics: Pure Mathematics Jp sect 2x de. Example 1 Find: a fsinsxdx b fsin(4 — 2x) dx € feos (3x) de d Solution a Jsinsxax = —seos5x+ 6 < [sinks dx = —Zeasix +6 : 1 b [sins 2xyax = ~ Cent) 40 = Jeos(4 2x) te c [feos (3-x) dx= [Eysn @-x), = (-sin2) - (—sin3) =-sin2 +sin3 = 0.768 (34.p.) d [sec ar ax = [ftan 2x]5/* = (flan) - ({tano) = (x1) -@x0)=3 Remember to use double angle formulas when integrating sin® x or cos? x (see Chapter 9). Sometimes the compound angle formulas can be used to transform an integral into a ‘simpler form’. MRE coe ssn eens TERAREBEsA coed — sin in ERA ces + sin asin Example 2 Find: a fsinSxcos 2x dx bff! sinxsin 3x dx hy {2} ia (4) s Material 12 Integration I! Solution a Adding formulas [1] and [2], sin(A +B) +sin(A—B) = 2sinAcosB Now let A = 5x and B = 2x ‘Then sin(5x +2x) + sin(5x — 2x) = 2sin 5x cos 2x = sin7x + sin3x = 2sin 5x cos2n => } (sin 7x + sin 3x) = sin 5x cos 2x ‘The product sin 5x cos 2x has now been transformed into the sum of two trigonometrical functions, which can be integrated, foinsxcosaeax = fptsinzx + singy) ae = 3f(sin 7x-+ sin 3x) dx = }[-}eos 7x — feos 3x] + ¢ = — fyeos 7x — }cos3x +e b Subtracting formula [4] from formula [3], cos(A +B) ~ cos(A —B) = —2sinAsing Let A= x and B = 3x, Then cos(x + 3x) — cas(x — 3x) = — 2sinysin3v =} [cos 4x — cos(—2x)] = sinxsin 3x Recall that cosine is an even function, so that cos(~2x) = cos 2x. So sinxsin $x = —} (cos 4x — cos 2x) 4 4 Now i sinxsingw dx = [" =1(cos 4x — cos 2x) dx lo a =a torax-eos2n ax = f[f}sinax— sin 2x5!" = -4[(Qsinz ~ Jsin§) ~ ({sino ~ Jsino)] =-H(xo-3x1-0+0) =i) =4 SMa echilich geschitt ‘Complete Advanced Level Mathematics: Pure Mathematics The exponential function Remember that ane important feature of the exponential function is that its gradient function is equal to the function itself. By applying the chain rule to the exponential function, derivatives can be found as follows, and by thinking of integration as the reverse of differentiation we can write down another set of standard integrals, Example 3 Find: a fe*dx b fetdx ce foldx di fhot dx Solution a forex Ge" be Using result [2], with a = —1. = e+e b fone ste" +6 << Using result [2], with a = 2. c Joraca Get ventdtte a fle are aero =e) = (C48) =e? +16? =e? —e°) = 1.468 (3 dp.) 498 Urheberrechtlich geschitt Material ‘Complete Advanced Level Mathematics: Pure Mathematics Definite integrals involving logarithms. ‘The diagram shows the graph of y = 1. The areas of the shaded regions are equal. Since we find the area under the curve by integrating between limits, we would expest [7 !dx =— [oy 1 dx. a [at =In2-In1 =Inz i dx =[lax}-3 axe eee = In(—1) — In(—2) But we get stuck here because we cannot find the logarithm of a negative number on a calculator; the logarithmic function is only defined for positive values of x, To avoid unnecessary complications with logarithms of negative numbers, we use the modulus function when integrating with the logarithmic function, In this example, mg : [ = dx = [Injxi}=} = In|-1| = In|~2) =In1—Inz =-In2 In general: p= Inif(x)l +e Urheberrechilich geschiit Remembor that the modulus function te ‘us to take the absolu value. Check Example 4. Ni that the modulus function is needed f parts but d, because possible for f(x) <0 s Material 12 Integration It ‘The diagram shows part of the curve y = ;5,.. Find the area of the shaded region. Solution o 18 shaded area = [°° dx --f ie ~2[ln|a — ax] = ~2[ln|-2| — In|-3}) = ~2[ln7 — In} = =21n() ‘The minus sign signifies that the area is below the x-axis, so tho shaded area is 2In(2). Inverse trigonometric functions Another set of important integrals can be found by studying the results of differentiating inverse trigonometrical functions. Begin with sin“'() and tan™*(8), where a is a constant. Using general functions like these will allow us to establish another sot of goneral integrals. =sin (2 ta y=aiv*(2) Then siny == Recall from Chapter 11 how to differentiate sin y Differentiating with respect to x, implicitly with respect tox, dy_1 OV ie dy 4 Se ads aeosy BM oria Urheberrechilich geschitt 12 Integration I Example 6 Find: late » [alee Solution 4 Using the standard sesult [ [ots dx = sin”! (®) +6, witha = 1 leer sin (2) +e=ain-* x+e b Using the standard result [1 ay ae=F tan (% )+ewith a= vi Jreee= [pe ee (FD + Example 7 aluato [| —3— ax tvaluate | Solution Rearrange the integral in standard form by extracting a factor of 3 oulside the integral sign and changing the 4 to 2°, x= =3[ akg ar (ao “als ( JI. Remember to work in = afsin-1(2) — sin“ (0)] radians. -3[2- |=5 Recall the special angles: Urhebertechtlich ges chittzte giggeeia (Complete Advanced Level Mathematics: Pura Mathematics 12.1 Standard Integrals Exercise Technique [1] Fina: a fsinaxdx d fcos(1 + 2x)dx [2] evatuate: a ff*sinzeaa © fil" soct ede [3] Fina: a fsin2xcosxdx cj? sinaesinx dy [4] Fina: b feosaxdy « fsin(ax—1)dx e fsect axdx f (1 + sec? 2x) dx b !cos(1 +20) d0 d fi! sect axde b fcos2xcosx de d Gsinxcos2xdx a fe™dx Je dx g s2e™dx b fe*dx Jo dx h fie“ dx ce fade Je dx i fae de [5] rina; a f(2x-+3e**) de Ja-el*)dx of (f(e—3e™) de b f(a-e)dx G20" dx s fi@elt*ax 6 Ja+etfds [6] ere: a fide b fidx © ftde od fase e seirde fF Sree og Seigde oh frye Find: a fr de Syke x c sade do fpr de Sats dx f ficiede Find: a feotxdx Jit de © fame [9] Fina: a falar Jae ce falede S fa ds Sytade Jae f Sytzde 12 Integration I Contextual [11] skotch the curvey =1+ sin2x for is a proper fraction and can be written in the form, where k is a constant. x ax FO) aye So Jae-if[ Fee 4 Recall | 5 ae — nisin) +e. = in +x?) +0 Improper fractions cannot be written in the form 2, where kis a constant. To integrate an improper fraction first write it as the sum or difference of proper algebraic fractions. Then try to use the list of standard integrals. Simple improper fractions All improper fractions can be expressed as the sum or difference of a polynomial and a proper fraction. Example 1 Find: x x+4 a ‘ [rae ik fae . lAe® Solution a First express ;4, as the sum ofa polynomial and a proper fraction. Do this by writing the numerator in terms of the denominator x _-(1-x)+1_-(1-) theberrect chitztes GO Complete Advanced Level Mathematics: Pure Mathematics Now [7 ae= J(u) wwe [ae =-x-] =x-In|i-a|+e (x) b First write 4s a sum involving a proper fraction. ax+2 [ae 4 The form [ Ft =x+2injx— 1) +e xt 7x _ x7 +x7)— 7x 8 Fee Tea TF x7 +x) 7x Te 7x? _. A aye nor pti (e-sn) =te—7(2, =i 7a =e -2[ eae a theform {£2 ax F(x) | = fx? -JIn(7 +52) +e Partial fractions If the denominator can be factorised, partial fractions can be used to simplify the rational function (see Chapter 6). jo dr Use | £2 a= infin) +e Utheberrechtlich geschittate ‘The integral is now i the standard form where f(x) =1—x. by Check this result by differentiation. Complete Advanced Level Mathematics: Pure Mathematics ¢ Express 1x in partial fractions. First factorise; Mex? =x +1)1.T 1, 4 west =a te4+; Check that t= 5-24 Jaga -[(S-ytxa7)* = frtac— [Pas [ae _— pi Fa Example 3 a dx =1inb. a 26x? + 5x4 91 b Evaluate |, anya 15) Solution a Notice that the denominator is the difference of two squares. That is, 4—x? =(2+.x)(2 —x). Use this to write partial fractions, 1 1 Gheck thet Gaara) * aa) tie fie d= f (sees aes eftoeif =tfgpse af e “fia = H[ln|2+ai]', — in 2—x//5 =}(In3 —In1) —}(In1—In3) =Hng-0-0+}In3 fetsinear = te"(2sint— cost) +c constant of integratio ‘Check this answer using the product rule, Differentiating 4e"(2sint — cost) should give the original integrand, o” sint. 12 Integration lt 12.4 Integration by Substitution Integration by substitution is a technique used to transform difficult integrals into simpler ones in which the standard integrals can be used. How could J(1 + 3x)° dx be found quickly? We could expand (1 + 3x) using the binomial expansion and integrate the resulting polynomial term by term. An alternative is ta make the substitution t = 1 +3x. Then Ja+an’ar becomes [fae. Although the integral has been simplified we cannot integrate ¢* with respect tox, We need a df rather than a dx after the integrand. Let = [itas then iy) ‘Tha chain rule gives ar dx _ ode di di dt Intograting both sides with respect to t gives, Jeger Comparing this to equation [1], we see that dx is effectively replaced by 4 dt. Since f= 1+ 3x in this case, {f= 3. So 4! =}, and dx is replaced by }dt. So Jo+ ax)® dx =| x de “fs nae =pxdte Substitute f = 1+ 3x. =h( tax +e Notice that the substitution makes the integrand simpler. The difficult part is changing the variable with respect to which the integration is carried out from x to t. 521 12 Integration i Find the values of the new limits: Do not worry that the a cate i © when e=2)1=1-2=—4 So fa-wtax "Px (yee Integration by substitution can be used to establish other standard integrals. For example to find [ax +b)" de, where a, band n are real numbers, use the substitution f= ax +b. Then #! =a, and 4 =3, eae fugl Now ic0y dx = Je xh = if a re" a aaa ‘Therefore | PI. sors Look back again at Example 1 where (ax + b)" = (6x +1)!, Here a= 3, b=landn=4. So foc+ntar= 260 =f Grtilec ‘More complex integrals require a higher level of algebraic skill to make sure that the integral has been expressed entirely in terms of the new variable only. Example 4 Evaluate |°, x/7 Px dx. Urheberrechttich Complete Advanced Level Mathematics: Pure Mathematics Solution Lott = 14x. Then gf =1and $}=1. When x = —1,f=0 and when x =0,t=1, Replace dx with de. Viteae = [ic-nvex sae fp Notice that the integran - fe bes xviPais written in = 9e" 30% =(}x1-}x1)- (0-0) Example 5. Use the substitution u? = 5 —x to find f?(x+4 1)(5 — x}! dx. Solution Let u* = 5 ~x, and differentiate implicitly with respect tox. ‘Then 2u{f = —1 and so $f = —2u. When x =1,u?=5-1=4 > u=2 When x = 5,u7=5-5=0 + u=0 Also,.x =5-u? > x+1=6 Now ficene =xidx = | (6 -u*)(u*)! x -2udu =-2 [ie- uu? du =-2 [low ul) du = ~2[2u" —4u5}2 = =2[(0 -0) = (2 x 2° =} x 2)] =-2(-16 + 2) Choosing suitable substitutions Try finding [xe™ dx. Using a systematic approach, integration by parts may be your first choice, but the second integral is too difficult to integrate. A good strategy is then to try a substitution. Uthebertecthtlich ges chatzte 12 Integration I Spotting the substitution ¢ = g(x), where g(x) is some function of x, often depends on recognising a common factor that appears in the integrand and in g'(x). Step @ Let tbe some function of x so that a factor is created that will cancel or simplify the integrand. Let t=x? 4 Differentiating ¢ = x* wrtx gives a factor of x, which appears in the integrand. ae, Step @ Write dx in terms of dt. at 1 eat 2 k= pat Now fre? a= fret Zar <4 Notice that the x will cancel. =}fetdr Step @ Integrate with respect to t. fret ax =4year =lete Step @ Substitute for ¢ throughout to express the integral in terms of the original variable, x. fre? arate" +e Other useful hints @ = For integrals containing e*, try | = e*. @ For integrals containing Va" = x", try x = a sin9. For example, with Va—x#, use x = 2sin 0. @ For integrals containing Va" +x°, try x = a tan, For example, with Vo+x!, use x =3tand. Example 6 Find: a Joint 0cos a0 b [ese Solution a Leté=sind. < @ Differentiating t = sin 0 gives us a factor of cos 0. Thon # =cos@ = dé@=bydt. @ Write di) in terms of dt. Urheberrechtlich geschttztes 48a Complete Advanced Level Mathematics: Pure Mathematics Notice that the cos @ terms cancel. Now sin? ecos.a0 ~ fe xcosd x—Ladt cos 0 = [rae =]P+e¢ <@ Integrate wrt t. =Jsin’ 0+¢ 4 @ Substitute for t. b Letp=e', << @ Exponential functions in the integrand can be simplified. Then ¥=e" 3 dv=3dp ay = Ae™ -1 y =e" —1 <@ Simplify the constant. So the general solution is y = Be** ~ }, where B is a constant. Example 2 Find the solution of the differential equation (1+ x) {= 1 — sin® y, for which y= { when x= 0, (OCSEB) 538. Urheberrechtlich geschttztes Material 12 Integration II Solution 1d 4 Tosinty de 145 4D Separate the variables. 1 dy 1 ; la eli <4 @ Integrate both sides wet x. tany =In|1+x\ +e When x= 0," =3 Thus tang=In1+e¢ 4 @ Find the constant. 1=0+6 + 1=¢ So tany=In|1+x|+4 y= tan“ (In|1 +x) +1) Modelling with differential equations ‘When there is sufficient information to describe how one variable changes with respect to another, the situation can often be modelled by a differential equation. Newton's law of cooling ‘This states that the rate of change in temperature of an object is proportional to the difference between its temperature and that of its surroundings. Newton's law can be modelled by a differential equation. For hot water cooling, let 0 be the temperature of the water at time f ‘To minutes, and the surrounding temperature be 18°C. The difference some assumptions have: between them is then @— 18, Then Newton's law gives to be made, In this case it is assumed that the 0 @—18) temperature of the a surroundings is constant. Writing this as an equation, #. K-18), where ke R. chitzte3Bterial Urheberrechtli Complete Advanced Level Mathematics: Pure Mathernatics 540 ‘The negative sign indicates that @ is decreasing, Ifit were to be omitted the constant k would work out to be negative, Population growth Scientists have crudely suggested thal the populations of certain organisms grow at a rate that is proportional to the size of the population. Lo be the population at time f. Then dN Fran Writing this as an equation, dN dt AN, where keR. Example 3 According to Newton's Law, the rate of cooling of an object is proportional to the temperature difference between the object and its surroundings. Warm water is poured into a basin, and cools from 41.1°C to 40.0°C in 5 minutes, The temperature of the room is a constant 17°C, a Write down a differential equation to mode! the temperature of the water. b Solve the differential equation, and find the temperature of the water after 10 minutes. & Find the time taken for the water to cool to 37°C. Solution a Let 0 be the temperature in degrees Celsius of the water at time (, ‘Then according to Newton's Law the rate of change of @ with respect to time is proportional to 0 — 17. a (0-17) wo a ~—k(0— 17), where k is a constant —k <4 Separate the variables, 17 the ‘modulus function is required. 0-17 =Ae™™, where A is a constant 0=17+A0# Complete Advanced Level Mathematics: Pure Mathematics 12.6 Differential Equations Exercise Technique [1 | Find the general solution to the following differential equations: Baan 4? d B= Wet 208 g 2+raxrh b antec 9 Baxdtsinx oh fot tata e Baxte f xf=1—xe% i 1ogaat+ [2] Find the general solution to the following differential equations, a a 2x? e =a i #-ys1 b socx t= £ foxy i (e+ a)R-2=0 g-4 8 kay ag} ho ¥=xe" 3 Find [xo% dx, a b Use yaur result to a to find the general solution to e~* $f = =i | The variables x and y satisfy the differential equation # + ytanx = 0. a Find the general solution. b Find a particular solution where y = 1 when x = 5. 5 | a Find the general solution to = b Find a particular solution where y = 4 hia x Contextual 1 | An object cools from 100°C to 70°C in 34 minutes. Assume Newton's law of cooling applies. The surrounding air temperature is 20°C, a State Newton's Law of cooling. b If Tis the temperature of the body at time { minutes, write down a differential equation that deseribes haw the body cools. © Solve the differential equation to find Tin terms of f and use it to find the temperature after 45 minutes, d= How long will the body take to cool to 50°C? 2 | The rate of increase of the number of bacteria present in a culture is proportional to the number present at that time. a If Nrepresents the number of bacteria after f hours write down a differential equation connecting N and t. Complete Advanced Level Mathematics: Pure Mathematics. Consolidation Exercise A Integrate the following with respect to x: aid e sin5xsinx i cotax b 4cos(2x +7) f 1+0™ iogh c sin2x+cosie og de k gb ae ceacae hose 1 os [2] use integration by parts to find the indefinite integral {x*Inxdx. (SMP) [3B] evatuate jg xo ax. (ULEAG) : £ +. " [4] show that [itp d= Ine, where a is rational number to be determined. a) [B] Find the vatues of o and bit [295 dx = —tinb. [6] use the substitution 1 = 1 +3sin to evaluate [25 do. (SMP) jo v1 + 3sin [7] Find the value of ff x(x — 3)” dx. (ULEAG) [8] Find y in terms of x given that = yo and that y = 1 when.x = 0. (ULEAG) A balloon is expanding and at time ¢ secands its surface area is scm. ‘The expansion is such that the rate of increase of s is proportional to. Jy When the surface area is 900 cm’, it is increasing at a rate of 60cm’ s"*. a Show that ¢5 = 1890, b Solve this differential equation given that s = 400 when f = 0, ¢ Find to the nearest second the time at which s = 900cm. (NEAB) Al time t hours the rate of decay of the mass of a radioactive substance is proportional to the mass xkg of the substance at that time, At time {= 0 the mass of the substance is Akg. a By forming and integrating a differontial equation, show that = Ae™, where kis a constant, b Itis observed that x = }A at time ¢ = 10, Find the value of t when x = 14, giving your answer to two decimal places. (ULEAC) 12 Integration I Exercise B 1 | The curve shown has parametric equations x = deos 0, y = 5 sind, 0<0<2, The straight line PR is a tangent to the ellipse at P, where @ = 5. Find the area of the shaded region, bound by the curve, the x-axis and the tangent PR. [2] Evaluate the following integrals exactly: a [ sinxcos3x dx b f o o [3] a Use integration by parts to find fxe™* dx. b Y= (xt) cosx ‘The graph shows part of the curve y = (x-+1) cosx. Calculate, correct to two decimal places, the area of the shaded region. a tox + 1’ B Ala Express Sp ycay ay it the form A + gay Use this to ax? 42x41 tnd | per : ax? toxt4 A, Bx+C b Expross “ay gy inthe form + pg Then find [ee xi +4) 2 x(13 - 3x) c Bvaluate ye By means of the substitution u = 6 —x or otherwise, show that Jeg dr = -In|o—a| +0. Urhe Complete Advanced Level Mathematics: Pure Mathematics b Asamathematical model for my regular 6 km morning jog, I assume that my speed is proportional to the distance that I still have to go to reach the end. i If start offat a speed of kmh", and after f hours have travelled xkm, explain why 4 = £(6 — x), where kis a constant. ii Show that k= 4. ¢ By solving the differential equation #¢ = 7,45, find the time taken for me to complete 4 km of the jog, and state my predicted speed at the time. d__ By considering the predicted time taken for me to complete the jog, comment on the suitability of the model. (SMP) G | Water flows out of a tank through a pipe at the bottom, and at a time t minutes the depth of water in the tank is x metres. At time t = 0 the depth of water in the tank is 2 m. After 10 minutes the depth is 1.5 m. It is desired to find the time T, correct to the nearest minute, at which the depth is 1m. a Inasimple model the rate at which the depth of water in the tank docreases is taken to be constant. Find T using this model. b In amore refined model the rate at which the depth of water remaining in the tank is decreasing at any instant is proportional to the square root of the depth at that instant. i Explain how the more refined model leads to the differential equation —4f = kx, where k is a positive constant. Find the general solution of the differential equation in i. Find Tusing the more refined model. (UCLES) 7 | A mathomatical model for the population growth of a certain country assumes that the country has a constant birth rate of 22 per thousand and aconstant death rate of 7 per thousand. The population of the country at ? the end of 1995 was 4.80 million and the population of the country at time t years later is denoted by x. a Show that 4° = 0.015x. b Find x in terms of t. Use the mathematical model to estimate: the population of the country at the end of 1998 d_ the year in which the population of the country will exceed 6 million. (WJEC) 12 Integration I Applications and Activities Cooling Fill a container with hot water. Note the room temperature and place a thermometer in the hot water and wait until it has adapted to the temperature of the water. At this point begin to note the water temperature every 2+ minutes. Record your results in a table similar to the one below. = o 10 [125] 15 [175] 20 [225] 25 [275 | Write down Newton's law of cooling Write down a differential equation that will model the temperature of the water. ¢ Solve the differential equati the table to find 0 in terms of t. d__ Investigate how well your differential equation models this situation, @ Investigate Newton's law of cooling for different shaped containers. 2s| 5 [75 ce and use the first and third entries in Integration spotting ‘The fundamental theorem of calculus states that differentiation is the reverse of integration, In this activity an integral in the form [f(x)g(x) dx must be made by selecting two function fram the box below, oe The answer must contain a combination of terms from the second box, below. One example is f} «x? dx = 1x? +0. A more complex example is [xe" dx =e" + ¢, because fle" +0) = 2xe". How many different integrals complete with answers can you spot?

Você também pode gostar